新GRE作文ARGUMENT官方题库+翻译

This page contains the Argument topics for the Analytical Writing section of the GRE?revised General Test. When you take the test, you will be presented with one Argument topic from this pool.

Each Argument topic consists of a passage that presents an argument followed by specific task instructions that tell you how to analyze the argument. The wording of some topics in the test might vary slightly from what is presented here. Also, because there may be multiple versions of some topics with similar or identical wording but with different task instructions, it is very important to read your test topic and its specific task directions carefully and respond to the wording as it appears in the actual test.

---------------------------------------------------------------------------------------------------------------------------- 1)

Woven baskets characterized by a particular distinctive pattern have previously been found only in the immediate vicinity of the prehistoric village of Palea and therefore were believed to have been made only by the Palean people. Recently, however, archaeologists discovered such

a"Palean"basket in Lithos, an ancient village across the Brim River from Palea. The Brim River is very deep and broad, and so the ancient Paleans could have crossed it only by boat, and no

Palean boats have been found. Thus it follows that the so-called Palean baskets were not uniquely Palean.

Write a response in which you discuss what specific evidence is needed to evaluate the argument and explain how the evidence would weaken or strengthen the argument.

一种具有独特花纹的编织篮子以前只在史前村庄Palea的临近地区发现过,因而被认为是Palea居所独有的。然而最近,考古学家在一个与Palea隔着一条叫做Brim河的村庄Lithos发现了一个这样的Palea篮子。Brim河很深很宽,所以古代Palea人只能坐船穿过它,但没有证据表明Palea人拥有船只。这表明那种所谓的Palea篮子并不是那里所独有的。

1) the author fails to consider the possibility that the Palean may arrived in Lithos through other means

2) the Palean basket could arrive other place through many possible methods such as trade, or boats

possessed by other culture.

3) In Palean times, the Brim River may not be as deep and broad as it is currently.

4) This may also result from migration. 5) 没有证据表明palea和brim是同时期存在的,不同时期情况更复杂

---------------------------------------------------------------------------------------------------------------------------- 2)

The following appeared as part of a letter to the editor of a scientific journal.

"A recent study of eighteen rhesus monkeys provides clues as to the effects of birth order on an individual's levels of stimulation. The study showed that in stimulating situations(such as an

encounter with an unfamiliar monkey),firstborn infant monkeys produce up to twice as much of the hormone cortisol, which primes the body for increased activity levels, as do their younger siblings. Firstborn humans also produce relatively high levels of cortisol in stimulating situations (such as the return of a parent after an absence).The study also found that during pregnancy, first-time mother monkeys had higher levels of cortisol than did those who had had several offspring."

Write a response in which you discuss one or more alternative explanations that could rival the proposed explanation and explain how your explanation(s) can plausibly account for the facts presented in the argument.

最近一项对恒河猴的调查揭示了出生顺序对于个体刺激水平的影响。调查显示在受到刺激的环境下(例如遇到一个不熟悉的猴子),头胎小猴的皮质醇分泌会翻倍,这样增加了它及其兄弟姐妹的身体灵活度。人类的头胎孩子在遇到刺激(例如父母外出回家)时也会分泌出相对浓度的皮质醇。研究同时还发现在怀孕期间,第一次为人母的猴子的皮质醇要比生过几次孩子后的猴子水平高很多。

1) 任何猴子不是一类,不能归于一谈

2) 产生皮质醇的原因可能是由于害怕等因素而非出生顺序

3) 产生皮质醇可能是由于害怕等等因素而非出生顺序

4) 样本的数量和真实性、代表性作者没有说明 5) 母猴体内皮质醇水平跟小猴体内皮质醇水平没有直接关系

-------------------------------------------------------------------------------------------------------------------------------- 3)

The following appeared as a letter to the editor from a Central Plaza store owner.

"Over the past two years, the number of shoppers in Central Plaza has been steadily decreasing while the popularity of skateboarding has increased dramatically. Many Central Plaza store

owners believe that the decrease in their business is due to the number of skateboard users in the plaza. There has also been a dramatic increase in the amount of litter and vandalism throughout the plaza. Thus, we recommend that the city prohibit skateboarding in Central Plaza. If

skateboarding is prohibited here, we predict that business in Central Plaza will return to its previously high levels."

Write a response in which you discuss what questions would need to be answered in order to decide whether the recommendation is likely to have the predicted result. Be sure to explain how the answers to these questions would help to evaluate the recommendation.

过去两年中,中心广场的顾客数量稳步下降与此同时滑板却急剧增加。很多中心广场的店主认为顾客数量下滑是由于玩滑板人数的增加。同时整个广场废弃物和公物损毁也大大增加。因此,我们建议应该在中心广场禁止玩滑板。如果禁止的话,我们估计中心广场的生意将会恢复到以前的水平。

1) 过去“两年”,时间太短使得数据不具说服性

2) 没有证据表明顾客数量的下降与skateboarder的增加有直接关系

3) 仅仅是店主的看法不具有参考价值,应该调查顾客等更多人的看法

4) 废弃物和公务损坏的增加也会是顾客数量下降的原因

5) 没有证据表明废弃物和公务损坏的增加是skateboarder造成的

171)

The following appeared as a letter to the editor from the owner of a skate shop in Central Plaza.

"Two years ago the city voted to prohibit skateboarding in Central Plaza. They claimed that skateboard users were responsible for the litter and vandalism that were keeping other visitors from coming to the plaza. In the past two years, however, there has only been a small increase in the number of visitors to Central Plaza, and litter and vandalism are still problematic.

Skateboarding is permitted in Monroe Park, however, and there is no problem with litter or

vandalism there. In order to restore Central Plaza to its former glory, then, we recommend that the city lift its prohibition on skateboarding in the plaza."

Write a response in which you discuss what questions would need to be answered in order to decide whether the recommendation and the argument on which it is based are reasonable. Be sure to explain how the answers to these questions would help to evaluate the recommendation.

两年前,市里投票禁止了在中央广场玩滑板。他们认为是滑板爱好者的废弃物和损毁公物的行为使游客远离了广场。然而在过去两年中,中央广场的游客却只有少数增加,而且废弃物和损毁公物的行为依然存在。滑板运动在M公园就被允许,但是那里却没有任何的废弃物和损毁公物的行为。为了能让中央广场重新焕发生机,我们建议市里应该取消广场禁止滑板的限令。

---------------------------------------------------------------------------------------------------------------------------------------------

4)

The following appeared in a letter from a homeowner to a friend.

"Of the two leading real estate firms in our town—Adams Realty and Fitch Realty—Adams Realty is clearly superior. Adams has 40 real estate agents; in contrast, Fitch has 25, many of whom work only part-time. Moreover, Adams' revenue last year was twice as high as that of Fitch and included home sales that averaged$168, 000, compared to Fitch's$144,000.Homes listed with Adams sell faster as well: ten years ago I listed my home with Fitch, and it took more than four months to sell; last year, when I sold another home, I listed it with Adams, and it took only one month. Thus, if you want to sell your home quickly and at a good price, you should use Adams Realty."

Write a response in which you examine the stated and/or unstated assumptions of the argument. Be sure to explain how the argument depends on these assumptions and what the implications are for the argument if the assumptions prove unwarranted.

在本市的两家最大的房地产经纪公司--Adams Realty和Fitch Realty--之中,Adams显然更优秀一些。Adams有40名房地产经纪人,而Fitch只有25个,且很多是兼职工作。而且,Adams去年的收入是Fitch的两倍,其平均房价为$168000,而Fitch仅为$144000。在Adams销售的房屋卖得也更快:十年前,我把我的房产交给Fitch,它用了四个多月才卖出去;去年,我在Adams卖了另一处房产,仅用一个月就售出了。因此,要想让你的房产卖的更快更好,你应该选择Adams。

1) A的收入是F的两倍,但没有数据显示A售出房产的数量是F的两倍,所以很可能A收取了较

高的中介费

2) A的房价比F高没有考虑到房产的净价值,可能A的房产本来就价值高,所以售价较高

3) 十年前的数据不能跟去年的数据比较,不具可比性

---------------------------------------------------------------------------------------------------------------------------- 5) The following appeared in a letter to the editor of the Balmer Island Gazette.

"On Balmer Island, where mopeds serve as a popular form of transportation, the population increases to 100,000 during the summer months. To reduce the number of accidents involving mopeds and pedestrians, the town council of Balmer Island should limit the number of mopeds rented by the island's moped rental companies from 50 per day to 25 per day during the summer season. By limiting the number of rentals, the town council will attain the 50 percent annual reduction in moped accidents that was achieved last year on the neighboring island of Seaville, when Seaville's town council enforced similar limits on moped rentals."

Write a response in which you discuss what questions would need to be answered in order to decide whether the recommendation is likely to have the predicted result. Be sure to explain how the answers to these questions would help to evaluate the recommendation.

b岛的人口在夏季达到了100000,而助力车则是岛上一种非常普遍的交通方式。为了减少由助力车和行人引发的事故,b岛市政府应该限制岛上夏季助力车租赁公司的租车数量,从原来的每天50量减少到25量。通过限制出租数量,市政府将会实现助力车事故减少50%的目标,就像去年邻近s岛市政府在实施了同样的政策后所取得的效果。

1) 没有证据显示岛上的人车交通事故是因为助力车造成的,原因可能主要是由于行人的不遵守交通规则造成。

2) 即使是降低了50%的数量,也不意味着会降低50%的事故啊,可能车少了开的就会更快,反而会增加事故的严重性

3) 岛上助力车租赁公司的租车数量占的比例多吗?减少会明显改变岛上助力车的总量吗?

4) 没有证据显示助力车过多造成了事故,作为主要的交通工具,可能正是因为助力车在夏季过少, 才导致大多数人不得不走路,而更多的人走到了助力车道,造成交通事故,加大助力车的投入可能还会减少事故的发生

5) 在s岛上产生的效果不一定在b岛产生

159)

The following appeared in a letter to the editor of the Balmer Island Gazette.

"The population on Balmer Island increases to 100,000 during the summer months. To reduce the number of accidents involving mopeds and pedestrians, the town council of Balmer Island plans to limit the number of mopeds rented by each of the island's six moped rental companies from 50 per day to 30 per day during the summer season. Last year, the neighboring island of Torseau

enforced similar limits on moped rentals and saw a 50 percent reduction in moped accidents. We predict that putting these limits into effect on Balmer Island will result in the same reduction in moped accidents."

Write a response in which you discuss what questions would need to be answered in order to decide whether the prediction and the argument on which it is based are reasonable. Be sure to explain how the answers to these questions would help to evaluate the prediction.

b岛的人口在夏季达到了100000,而助力车则是岛上一种非常普遍的交通方式。为了减少由助力车和行人引发的事故,b岛市政府应该限制岛上夏季助力车租赁公司的租车数量,从原来的每天50量减少到25量。去年邻近T岛市政府通过限制出租数量,助力车事故减少50%的。我们预计B岛的相同限制也同样会起到效力并使助力车事故降低50%。

173)

The following appeared in a letter to the editor of the Balmer Island Gazette.

"The population on Balmer Island doubles during the summer months. During the summer, then, the town council of Balmer Island should decrease the maximum number of moped rentals

allowed at each of the island's six moped and bicycle rental companies from 50 per day to 30 per day. This will significantly reduce the number of summertime accidents involving mopeds and pedestrians. The neighboring island of Torseau actually saw a 50 percent reduction in moped accidents last year when Torseau's town council enforced similar limits on moped rentals. To help reduce moped accidents, therefore, we should also enforce these limitations during the summer months."

Write a response in which you examine the stated and/or unstated assumptions of the argument. Be sure to explain how the argument depends on these assumptions and what the implications are for the argument if the assumptions prove unwarranted.

b岛的人口在夏季翻了一番。b岛市政府应该限制岛上夏季助力车租赁公司的租车数量,从原来的每天50量减少到25量,这将有助于减少由助力车和行人引发的事故。去年邻近T岛市政府通过限制出租数量,助力车事故减少50%的。为了减少助力车事故,因此我们也应该采取相同的政策。 --------------------------------------------------------------------------------------------------------------------------------------- 6)

Arctic deer live on islands in Canada's arctic regions. They search for food by moving over ice from island to island during the course of the year. Their habitat is limited to areas warm enough to sustain the plants on which they feed and cold enough, at least some of the year, for the ice to cover the sea separating the islands, allowing the deer to travel over it. Unfortunately, according to reports from local hunters, the deer populations are declining. Since these reports coincide with recent global warming trends that have caused the sea ice to melt, we can conclude that the

purported decline in deer populations is the result of the deer's being unable to follow their age-old migration patterns across the frozen sea.

Write a response in which you discuss what specific evidence is needed to evaluate the argument and explain how the evidence would weaken or strengthen the argument.

北极鹿生活在加拿大极地区域的岛屿上。它们全年都通过冰块在岛屿间移动来寻找食物。它们的栖

居地局限在那些温暖得足以维持它们所需的植物生长,并且在一年的至少某些时候冷到足以让岛屿间的海面结冰以使它们能够在岛屿间旅行的地方。然而,根据当地猎人的报告,鹿的数量正在下降。由于这一报告正好与最近导致海洋冰面融化的全球变暖趋势同时发生,我们可以得出结论:北极鹿的数量的下降是它们无法按它们原有的迁移习惯穿越结冰海面的结果。

1) 猎人的报告是可靠?之前的鹿大概有多少头,之后的鹿有多少头?是否仅仅是因为鹿迁移出了猎人居住的区域

2) 也可能有其他原因,比如天敌数量增加等等,导致鹿的数量下降,并不一定是温度升高所致

3) 猎人的猎杀也会造成鹿的数量的下降,猎人很可能在推卸责任

4) 还有,而且没有证据表明冰川已经融化到无法迁移的程度,即使已经融化到无法迁移了,那么气候变暖也可能会导致岛上的植被增加,可能已经可以满足鹿的需求,反而不用迁移了,无法迁移并不一定意味着鹿的数量会减少

---------------------------------------------------------------------------------------------------------------------------- 7)

The following is a recommendation from the Board of Directors of Monarch Books.

"We recommend that Monarch Books open a caféin its store. Monarch, having been in business at the same location for more than twenty years, has a large customer base because it is known for its wide selection of books on all subjects. Clearly, opening the cafe would attract more customers. Space could be made for the cafe by discontinuing the children's book section, which will probably become less popular given that the most recent national census indicated a significant decline in the percentage of the population under age ten. Opening a cafe will allow Monarch to attract more customers and better compete with Regal Books, which recently opened its own café."

Write a response in which you discuss what questions would need to be answered in order to decide whether the recommendation is likely to have the predicted result. Be sure to explain how the answers to these questions would help to evaluate the recommendation.

我们建议m书店在电内开设一个咖啡厅。M书店在目前的店址上已经经营了20多年,并由于其广泛的图书种类而拥有了庞大的客户群体。很明显,新开设的咖啡厅会吸引更多的客户,空间可以通过撤出儿童书籍柜台来获得,因为最近一次全国调查显示10岁以下儿童的比率显著下降,所以儿童书就可能没以前那么畅销。开设新咖啡厅将会使m吸引更多客户并更好的与最近刚开设了咖啡厅的r书店展开竞争。

98)

The following is a recommendation from the business manager of Monarch Books.

"Since its opening in Collegeville twenty years ago, Monarch Books has developed a large

customer base due to its reader-friendly atmosphere and wide selection of books on all subjects. Last month, Book and Bean, a combination bookstore and coffee shop, announced its intention to open a Collegeville store. Monarch Books should open its own in-store cafe in the space currently devoted to children's books. Given recent national census data indicating a significant decline in the percentage of the population under age ten, sales of children's books are likely to decline. By replacing its children's books section with a café, Monarch Books can increase profits and ward off competition from Book and Bean."

Write a response in which you examine the stated and/or unstated assumptions of the argument. Be sure to explain how the argument depends on these assumptions and what the implications are for the argument if the assumptions prove unwarranted.

自打20年前在C开设以来,M书店就因其友善的氛围和广泛的图书种类而拥有了一大批顾客。三个月,B&B,一家综合咖啡书店,宣布其要在C开设新店。我们建议m书店在电内开设一个咖啡厅,空间可以通过撤出儿童书籍柜台来获得,因为最近一次全国调查显示10岁以下儿童的比率显著下降,所以儿童书就可能没以前那么畅销。开设新咖啡厅将会使m吸引更多客户并更好的与B&B书店

展开竞争。

99)

The following is a recommendation from the business manager of Monarch Books.

"Since its opening in Collegeville twenty years ago, Monarch Books has developed a large

customer base due to its reader-friendly atmosphere and wide selection of books on all subjects. Last month, Book and Bean, a combination bookstore and coffee shop, announced its intention to open a Collegeville store. Monarch Books should open its own in-store cafe in the space currently devoted to children's books. Given recent national census data indicating a significant decline in the percentage of the population under age ten, sales of children's books are likely to decline. By replacing its children's books section with a café, Monarch Books can increase profits and ward off competition from Book and Bean."

Write a response in which you discuss what specific evidence is needed to evaluate the argument and explain how the evidence would weaken or strengthen the argument.

自打20年前在C开设以来,M书店就因其友善的氛围和广泛的图书种类而拥有了一大批顾客。三个月,B&B,一家综合咖啡书店,宣布其要在C开设新店。我们建议m书店在电内开设一个咖啡厅,空间可以通过撤出儿童书籍柜台来获得,因为最近一次全国调查显示10岁以下儿童的比率显著下降,所以儿童书就可能没以前那么畅销。开设新咖啡厅将会使m吸引更多客户并更好的与B&B书店展开竞争。

--------------------------------------------------------------------------------------------------------------------------------------- 8)

The following appeared in a memo from the director of student housing at Buckingham College.

"To serve the housing needs of our students, Buckingham College should build a number of new dormitories. Buckingham’s enrollment is growing and, based on current trends, will double over the next 50 years, thus making existing dormitory space inadequate. Moreover, the average rent for an apartment in our town has risen in recent years. Consequently, students will find it

increasingly difficult to afford off-campus housing. Finally, attractive new dormitories would make prospective students more likely to enroll at Buckingham."

Write a response in which you discuss what specific evidence is needed to evaluate the argument and explain how the evidence would weaken or strengthen the argument.

为满足我们学生的住房需求,Buckingham学院应该建造一幢新的宿舍。Buckingham的报名人数正在增加,而且按照现有趋势,报名人数将会在未来50年中增加一倍,从而导致现有住宅不能满足要求。而且,我们镇上公寓的平均租金在近几年间上涨了。因此,学生将会越来越发现难于负担校外住宿费用。最后,一幢引人注目的新宿舍将会使未来的学生更愿意报考Buckingham。

--------------------------------------------------------------------------------------------------------------------------------------- 9)

Nature's Way, a chain of stores selling health food and other health-related products, is opening its next franchise in the town of Plainsville. The store should prove to be very successful: Nature’s Way franchises tend to be most profitable in areas where residents lead healthy lives, and clearly Plainsville is such an area. Plainsville merchants report that sales of running shoes and exercise clothing are at all-time highs. The local health club has more members than ever, and the weight training and aerobics classes are always full.Finally,Plainsville's schoolchildren represent a new generation of potential customers:these schoolchildren are required to participate in a

fitness-for-life program,which emphasizes the benefits of regular exercise at an early age.

Write a response in which you examine the stated and/or unstated assumptions of the

argument.Be sure to explain how the argument depends on these assumptions and what the implications are for the argument if the assumptions prove unwarranted.

NW是一家专卖保健食品及其他相关产品的连锁店,准备将其下一家店铺开在Plainsville。这家店一定会很成功:因为我们的商店在那些居对健康生活高度关注的地区是盈利最多的。很明显,plainsville就是这样的地方。Plainsville的商家报告说运动鞋和运动衣的销售处于历史高点。当地一家康体俱乐部现在的会员比以往任何时候都多,减肥训练和有氧运动班总是满员。另外,我们还可以预见到新生代的顾客群:Plainsville的在校学生被要求参加一个叫做"终生健康"的项目,它强调从小开始经常锻炼的好处。

--------------------------------------------------------------------------------------------------------------------------------------- 10)

Twenty years ago,Dr.Field,a noted anthropologist,visited the island of Tertia.Using an

observation-centered approach to studying Tertian culture,he concluded from his observations that children in Tertia were reared by an entire village rather than by their own biological parents.Recently another anthropologist,Dr.Karp,visited the group of islands that includes Tertia and used the interview-centered method to study child-rearing practices.In the interviews that Dr.Karp conducted with children living in this group of islands,the children spent much more time talking about their biological parents than about other adults in the village.Dr.Karp decided that Dr.Field's conclusion about Tertian village culture must be invalid.Some anthropologists

recommend that to obtain accurate information on Tertian child-rearing practices,future research on the subject should be conducted via the interview-centered method.

Write a response in which you discuss what questions would need to be answered in order to decide whether the recommendation and the argument on which it is based are reasonable.Be sure to explain how the answers to these questions would help to evaluate the recommendation.

20年前,知名人类学F博士访问了T岛,并从他的观察中得出结论:T岛的孩子是由整个村子抚养长大而不是他们的亲生父母。然而,另一位人类学家K最近对生活在T岛以及周边一些岛屿的儿童采访显示,他们在谈话中谈到他们呢双亲的时间远远多于谈到村庄里其他成年人。K博士的这次研究证明F博士关于T岛乡村文化的结论是错的,一些人类学家建议要想以进一步获取准确的T岛的儿童成长规律,未来应该采用采访为主的方法。

21) The following appeared in an article written by Dr.Karp,an anthropologist.

"Twenty years ago,Dr.Field,a noted anthropologist,visited the island of Tertia and concluded from his observations that children in Tertia were reared by an entire village rather than by their own biological parents.However,my recent interviews with children living in the group of islands that includes Tertia show that these children spend much more time talking about their biological parents than about other adults in the village.This research of mine proves that Dr.Field's

conclusion about Tertian village culture is invalid and thus that the observation-centered approach to studying cultures is invalid as well.The interview-centered method that my team of graduate students is currently using in Tertia will establish a much more accurate understanding of child-rearing traditions there and in other island cultures."

Write a response in which you discuss what specific evidence is needed to evaluate the argument and explain how the evidence would weaken or strengthen the argument.

20年前,知名人类学F博士访问了T岛,并从他的观察中得出结论:T岛的孩子是由整个村子抚养长大而不是他们的亲生父母。然而,我最近对生活在T岛以及周边一些岛屿的儿童采访显示,他们在谈话中谈到他们呢双亲的时间远远多于谈到村庄里其他成年人。我的这次研究证明F博士关于T岛乡村文化的结论是错的,因此这种以观察为主的文化研习方式也是无效的。由于我那些在T目前调研的研究生们正是采用我的采访为主的方法,所以他们将会对T岛孩子抚养传统已经其他岛屿文化给出更加精确的了解。

23)

The following appeared in an article written by Dr.Karp,an anthropologist.

"Twenty years ago,Dr.Field,a noted anthropologist,visited the island of Tertia and concluded from his observations that children in Tertia were reared by an entire village rather than by their own biological parents.However,my recent interviews with children living in the group of islands that includes Tertia show that these children spend much more time talking about their biological parents than about other adults in the village.This research of mine proves that Dr.Field's

conclusion about Tertian village culture is invalid and thus that the observation-centered approach to studying cultures is invalid as well.The interview-centered method that my team of graduate students is currently using in Tertia will establish a much more accurate understanding of child-rearing traditions there and in other island cultures."

Write a response in which you examine the stated and/or unstated assumptions of the

argument.Be sure to explain how the argument depends on these assumptions and what the implications are for the argument if the assumptions prove unwarranted.

20年前,知名人类学F博士访问了T岛,并从他的观察中得出结论:T岛的孩子是由整个村子抚养长大而不是他们的亲生父母。然而,我最近对生活在T岛以及周边一些岛屿的儿童采访显示,他们在谈话中谈到他们呢双亲的时间远远多于谈到村庄里其他成年人。我的这次研究证明F博士关于T岛乡村文化的结论是错的,因此这种以观察为主的文化研习方式也是无效的。由于我那些在T目前调研的研究生们正是采用我的采访为主的方法,所以他们将会对T岛孩子抚养传统已经其他岛屿文化给出更加精确的了解。

--------------------------------------------------------------------------------------------------------------------------------------------- 11)

The council of Maple County,concerned about the county's becoming overdeveloped,is debating a proposed measure that would prevent the development of existing farmland in the county.But the council is also concerned that such a restriction,by limiting the supply of new housing,could lead to significant increases in the price of housing in the county.Proponents of the measure note that Chestnut County established a similar measure ten years ago,and its housing prices have

increased only modestly since.However,opponents of the measure note that Pine County adopted restrictions on the development of new residential housing fifteen years ago,and its housing prices have since more than doubled.The council currently predicts that the proposed measure,if passed,will result in a significant increase in housing prices in Maple County.

Write a response in which you discuss what questions would need to be answered in order to decide whether the prediction and the argument on which it is based are reasonable.Be sure to explain how the answers to these questions would help to evaluate the prediction.

Maple County理事会担心Maple County会发展过度该理事会正在讨论一个限制县里现有农田开发的建议措施。但是理事会同样担心这种对新住房供给的限制可能会导致该县房价的猛涨。该措施的支持者们指出十年前Chestnut County制定了一个相似的措施后来那里的房屋价格只是有限得上涨了一些。但是该措施的反对者们也指出十五年前Pine County对其新建住宅的发展进行了限制后来那里的房价上涨了不止一倍。理事会预测该建议如果被通过将导致Maple County的房价猛涨。[说明写一篇文章阐述一些需要回答的问题以判断论证过程以及建立在此论证过程基础上的预测是否合理。一定要解释这些问题的答案如何帮助我们评价这个预测。]

--------------------------------------------------------------------------------------------------------------------------------- 12)

Fifteen years ago,Omega University implemented a new procedure that encouraged students to evaluate the teaching effectiveness of all their professors.Since that time,Omega professors have begun to assign higher grades in their classes,and overall student grade averages at Omega have risen by 30 percent.Potential employers,looking at this dramatic rise in grades,believe that grades at Omega are inflated and do not accurately reflect student achievement;as a result,Omega graduates have not been as successful at getting jobs as have graduates from nearby Alpha University.To enable its graduates to secure better jobs,Omega University should terminate student evaluation of professors.

Write a response in which you discuss what specific evidence is needed to evaluate the argument and explain how the evidence would weaken or strengthen the argument.

15年前,Omega大学实施了一项新措施,鼓励学生对所有教授的教学效果进行评价。从那以后,Omega的教授开始给予学生更高的分数,Omega的学生成绩总平均上升了30%。未来的雇主显然认为Omega的分数贬值了,无法反映出学生的真实能力。因此,Omega的毕业生找工作时没有邻近的Alpha大学毕业生成功。为使Omega毕业生找到好工作,我们应立即停止学生对教授的评价。

--------------------------------------------------------------------------------------------------------------------------------------------- 13)

In an attempt to improve highway safety,Prunty County last year lowered its speed limit from 55 to 45 miles per hour on all county highways.But this effort has failed:the number of accidents has not decreased,and,based on reports by the highway patrol,many drivers are exceeding the speed limit.Prunty County should instead undertake the same kind of road improvement project that Butler County completed five years ago:increasing lane widths,resurfacing rough highways,and improving visibility at dangerous intersections.Today,major Butler County roads still have a 55 mph speed limit,yet there were 25 percent fewer reported accidents in Butler County this past year than there were five years ago.

Write a response in which you discuss what specific evidence is needed to evaluate the argument and explain how the evidence would weaken or strengthen the argument.

为了提升高速公路的安全性,p市去年把所有高速公路的限速由55降到了45。但是这样的努力尝试失败了,事故发生的数量没有下降,并且根据高速巡警的报告很多司机都超速驾驶。所以p市应该采用b市5年前相同的道路改善计划:增加车道宽度,重新铺设不平路面以及改善危险交叉路口的能见度。今天,b市的主要道路依然是55限速,但上报的事故率却比5年前减少了25%。

--------------------------------------------------------------------------------------------------------------------------------------------- 14)

The following appeared as part of an article in a business magazine.

"A recent study rating 300 male and female Mentian advertising executives according to the average number of hours they sleep per night showed an association between the amount of sleep the executives need and the success of their firms.Of the advertising firms studied,those whose executives reported needing no more than 6 hours of sleep per night had higher profit

margins and faster growth.These results suggest that if a business wants to prosper,it should hire only people who need less than 6 hours of sleep per night."

Write a response in which you examine the stated and/or unstated assumptions of the

argument.Be sure to explain how the argument depends on these assumptions and what the implications are for the argument if the assumptions prove unwarranted.

最近一项根据每天平均睡眠时间,对300名m市广告公司男女高管调查显示出了高管们所需的睡眠时间与他们公司成功与否的关联。根据对这些广告公司的研究,那些公司高管报告说每天睡眠不足6小时的公司利润更高并且发展更快。这些结果表明如果公司想蓬勃发展,他们就必须只雇佣那些每天睡眠不足6小时的管理人员。 118)

The following appeared as part of an article in a business magazine.

"A recent study rating 300 male and female advertising executives according to the average number of hours they sleep per night showed an association between the amount of sleep the executives need and the success of their firms.Of the advertising firms studied,those whose executives reported needing no more than six hours of sleep per night had higher profit margins and faster growth.On the basis of this study,we recommend that businesses hire only people who need less than six hours of sleep per night."

Write a response in which you discuss what questions would need to be answered in order to decide whether the recommendation and the argument on which it is based are reasonable.Be sure to explain how the answers to these questions would help to evaluate the recommendation.

最近一项根据每天平均睡眠时间,对300名m市广告公司男女高管调查显示出了高管们所需的睡眠时间与他们公司成功与否的关联。根据对这些广告公司的研究,那些公司高管报告说每天睡眠不足6小时的公司利润更高并且发展更快。这些结果表明如果公司想蓬勃发展,他们就必须只雇佣那些每天睡眠不足6小时的管理人员。

--------------------------------------------------------------------------------------------------------------------------------------------- 15)

"The following memorandum is from the business manager of Happy Pancake House restaurants.

Recently,butter has been replaced by margarine in Happy Pancake House restaurants throughout the southwestern United States.This change,however,has had little impact on our customers.In fact,only about 2 percent of customers have complained,indicating that an average of 98 people out of 100 are happy with the change.Furthermore,many servers have reported that a number of customers who ask for butter do not complain when they are given margarine

instead.Clearly,either these customers do not distinguish butter from margarine or they use the term'butter'to refer to either butter or margarine."

Write a response in which you discuss one or more alternative explanations that could rival the proposed explanation and explain how your explanation(s)can plausibly account for the facts presented in the argument.

Happy Pancake House在美国西南部的餐厅用人造黄油代替了天然黄油。但是这种改变对顾客的影响非常小因为只有大约2%的顾客投诉,这说明100个人中有98人对于这种替换是乐于接受的。而且,很多服务生报告说很多仍然要了天然黄油的顾客在被给予人造黄油的时候并没有投诉。显然,这些顾客要么分不清天然黄油和人造黄油,要么是用"天然黄油"这个词汇来指天然黄油或人造黄油。

1) 没抱怨不代表不知道,可能很大一部分人都放弃抱怨了

2) Recently,时间太短而不具有说服力,可能时间长了顾客就会逐渐发现这种改变

3) 2%是谁统计的?统计方法的合理性?

4) 为什么没有数据表明改变之后餐厅的销量是否减少?

51)

The following memorandum is from the business manager of Happy Pancake House restaurants.

"Butter has now been replaced by margarine in Happy Pancake House restaurants throughout the southwestern United States.Only about 2 percent of customers have complained,indicating that 98 people out of 100 are happy with the change.Furthermore,many servers have reported that a number of customers who ask for butter do not complain when they are given margarine

instead.Clearly,either these customers cannot distinguish butter from margarine or they use the term'butter'to refer to either butter or margarine.Thus,to avoid the expense of purchasing butter and to increase profitability,the Happy Pancake House should extend this cost-saving change to its restaurants in the southeast and northeast as well."

Write a response in which you discuss what questions would need to be answered in order to decide whether the recommendation is likely to have the predicted result.Be sure to explain how the answers to these questions would help to evaluate the recommendation.

在整个美国西南部的所有Hp餐厅都采用人造黄油替代了天然黄油。只有2%的顾客投诉,这意味着100人中有98人对于这次改变都是愉悦的。另外,很多服务生报告说很多投诉要天然黄油的顾客在被给了人造黄油以后便没有了投诉。很明显,要么是这些顾客无法辨清人造或天然黄油,要么是他们用天然黄油来指代人造或天然黄油。因此,为了避免购买天然黄油的花销并且提升利润率,Hp餐

厅应该将这种节约开销的方式推广到美国东南部以及东北部的所有餐厅。

130)

The following memorandum is from the business manager of Happy Pancake House restaurants.

"Butter has now been replaced by margarine in Happy Pancake House restaurants throughout the southwestern United States.Only about 2 percent of customers have filed a formal

complaint,indicating that an average of 98 people out of 100 are happy with the change.Furthermore,many servers have reported that a number of customers who ask for butter do not complain when they are given margarine instead.Clearly,either these customers cannot distinguish butter from margarine or they use the term'butter'to refer to either butter or

margarine.Thus,to avoid the expense of purchasing butter,the Happy Pancake House should extend this cost-saving change to its restaurants throughout the rest of the country."

Write a response in which you examine the stated and/or unstated assumptions of the

argument.Be sure to explain how the argument depends on these assumptions and what the implications are for the argument if the assumptions prove unwarranted.

在整个美国西南部的所有Hp餐厅都采用人造黄油替代了天然黄油。只有2%的顾客投诉,这意味着100人中有98人对于这次改变都是愉悦的。另外,很多服务生报告说很多投诉要天然黄油的顾客在被给了人造黄油以后便没有了投诉。很明显,要么是这些顾客无法辨清人造或天然黄油,要么是他们用天然黄油来指代人造或天然黄油。因此,为了避免购买天然黄油的花销并且提升利润率,Hp餐厅应该将这种节约开销的方式推广到美国东南部以及东北部的所有餐厅。

131)

The following memorandum is from the business manager of Happy Pancake House restaurants.

"Butter has now been replaced by margarine in Happy Pancake House restaurants throughout the southwestern United States.Only about 2 percent of customers have complained,indicating that an average of 98 people out of 100 are happy with the change.Furthermore,many servers have reported that a number of customers who ask for butter do not complain when they are given

margarine instead.Clearly,either these customers cannot distinguish butter from margarine or they use the term'butter'to refer to either butter or margarine.Thus,we predict that Happy Pancake House will be able to increase profits dramatically if we extend this cost-saving change to all our restaurants in the southeast and northeast as well."

Write a response in which you discuss what questions would need to be answered in order to decide whether the prediction and the argument on which it is based are reasonable.Be sure to explain how the answers to these questions would help to evaluate the prediction.

在整个美国西南部的所有Hp餐厅都采用人造黄油替代了天然黄油。只有2%的顾客投诉,这意味着100人中有98人对于这次改变都是愉悦的。另外,很多服务生报告说很多投诉要天然黄油的顾客在被给了人造黄油以后便没有了投诉。很明显,要么是这些顾客无法辨清人造或天然黄油,要么是他们用天然黄油来指代人造或天然黄油。因此,Hp餐厅应该将会提升利润如果它这种节约开销的方式推广到美国东南部以及东北部的所有餐厅。

133)

The following memorandum is from the business manager of Happy Pancake House restaurants.

"Butter has now been replaced by margarine in Happy Pancake House restaurants throughout the southwestern United States.Only about 2 percent of customers have complained,indicating that an average of 98 people out of 100 are happy with the change.Furthermore,many servers have reported that a number of customers who ask for butter do not complain when they are given

margarine instead.Clearly,either these customers cannot distinguish butter from margarine or they use the term'butter'to refer to either butter or margarine.Thus,to avoid the expense of purchasing butter and to increase profitability,the Happy Pancake House should extend this cost-saving change to its restaurants in the southeast and northeast as well."

Write a response in which you discuss what specific evidence is needed to evaluate the argument and explain how the evidence would weaken or strengthen the argument.

在整个美国西南部的所有Hp餐厅都采用人造黄油替代了天然黄油。只有2%的顾客投诉,这意味着100人中有98人对于这次改变都是愉悦的。另外,很多服务生报告说很多投诉要天然黄油的顾客在被给了人造黄油以后便没有了投诉。很明显,要么是这些顾客无法辨清人造或天然黄油,要么是他们用天然黄油来指代人造或天然黄油。因此,为了避免购买天然黄油的花销并且提升利润率,Hp餐厅应该将这种节约开销的方式推广到美国东南部以及东北部的所有餐厅。

--------------------------------------------------------------------------------------------------------------------------------------------- 16)

In surveys Mason City residents rank water sports(swimming,boating,and fishing)among their favorite recreational activities.The Mason River flowing through the city is rarely used for these pursuits,however,and the city park department devotes little of its budget to maintaining riverside recreational facilities.For years there have been complaints from residents about the quality of the river's water and the river's smell.In response,the state has recently announced plans to clean up Mason River.Use of the river for water sports is,therefore,sure to increase.The city government should for that reason devote more money in this year's budget to riverside recreational facilities.

Write a response in which you examine the stated and/or unstated assumptions of the

argument.Be sure to explain how the argument depends on these assumptions and what the implications are for the argument if the assumptions prove unwarranted.

调查显示m市的老百姓把水上运动(游泳,划船及垂钓)作为最受欢迎的娱乐活动。贯穿m市的M河很少被用于水上运动,然而城市公园部门很少投入资金去维护沿河的活动设施。这么多年居民们一直投诉河水的质量及味道。为了回应这一投诉,当局最近宣布了清理M河的计划,因此M河的水上活动使用将会增加。所以市政府应该增加对河岸活动设施的资金投入。

--------------------------------------------------------------------------------------------------------------------------------------------- 17)

The following appeared in a memorandum from the manager of WWAC radio station.

"To reverse a decline in listener numbers,our owners have decided that WWAC must change from its current rock-music format.The decline has occurred despite population growth in our listening area,but that growth has resulted mainly from people moving here after their retirement.We must make listeners of these new residents.We could switch to a music format tailored to their tastes,but a continuing decline in local sales of recorded music suggests limited interest in music.Instead we should change to a news and talk format,a form of radio that is increasingly popular in our area."

Write a response in which you discuss what specific evidence is needed to evaluate the argument and explain how the evidence would weaken or strengthen the argument.

为了扭转观众流失的局面,WWAC的老板们决定必须要摒弃目前的摇滚乐风格。尽管我们广播区域内人口增加,收听人数依然下降。但增加的人口主要是一些退休后移居本地的。我们必须要抓住这些新的听众,我们可以转变音乐风格来迎合他们,但最近本地音乐销量的持续走低表明他们对音乐没什么兴趣。所以,我们可以变为新闻谈话类的形式来提升我们电台在本地的人气。

19)

Two years ago,radio station WCQP in Rockville decided to increase the number of call-in advice programs that it broadcast;since that time,its share of the radio audience in the Rockville listening area has increased significantly.Given WCQP's recent success with call-in advice

programming,and citing a nationwide survey indicating that many radio listeners are quite interested in such programs,the station manager of KICK in Medway recommends that KICK

include more call-in advice programs in an attempt to gain a larger audience share in its listening area.

Write a response in which you discuss what questions would need to be answered in order to decide whether the recommendation and the argument on which it is based are reasonable.Be sure to explain how the answers to these questions would help to evaluate the recommendation.

两年前,R地区的WCQP电台决定增加在其广播节目的电话参与节目。此后,其在R地区的听众覆盖率急剧增加。鉴于WCQP这种措施的成功,同时也根据一项全国范围的调查显示很多的观众比较喜欢这样的电话参与节目,M地区的Kick电台的经理建议Kick也应该增加类似的电话参与节目以来扩大在其地区的听众占有率。

93)

The following appeared in a memorandum from the manager of WWAC radio station.

"WWAC must change from its current rock-music format because the number of listeners has been declining,even though the population in our listening area has been growing.The population growth has resulted mainly from people moving to our area after their retirement,and we must make listeners of these new residents.But they seem to have limited interest in music:several local stores selling recorded music have recently closed.Therefore,just changing to another kind of music is not going to increase our audience.Instead,we should adopt a news-and-talk format,a form of radio that is increasingly popular in our area."

Write a response in which you discuss what questions would need to be answered in order to decide whether the recommendation and the argument on which it is based are reasonable.Be sure to explain how the answers to these questions would help to evaluate the recommendation.

为了扭转观众流失的局面,WWAC的老板们决定必须要摒弃目前的摇滚乐风格。尽管我们广播区域内人口增加,收听人数依然下降。但增加的人口主要是一些退休后移居本地的。我们必须要抓住这些新的听众。但最近很多本地音乐店的销量持续走低表明他们对音乐没什么兴趣。因此,转换另外一种风格的音乐想让不能增加听众人数。所以,我们可以采用一种最近本地十分流行的新闻谈话类形式。

109)

The following appeared in a memorandum from the general manager of KNOW radio station.

"Several factors indicate that radio station KNOW should shift its programming from rock-and-roll music to a continuous news format.Consider,for example,that the number of people in our

listening area over fifty years of age has increased dramatically,while our total number of listeners has declined.Also,music stores in our area report decreased sales of recorded

music.Finally,continuous news stations in neighboring cities have been very successful.The switch from rock-and-roll music to 24-hour news will attract older listeners and secure KNOW radio's future."

Write a response in which you examine the stated and/or unstated assumptions of the

argument.Be sure to explain how the argument depends on these assumptions and what the implications are for the argument if the assumptions prove unwarranted.

若干因素表明KNOW电台应该把它的节目从摇滚乐转成连续的新闻节目。举例来说,我们覆盖范围内的老年人数量显著增加,而总的听众数量最近减少了。而且,我们地区音像店中音乐的销量下降。最后,邻近城市的连续新闻节目非常成功。从摇滚乐转型到24小时新闻节目将会吸引更多的老年听众并将会保障KNOW电台的未来。

110)

The following appeared in a memorandum from the manager of KNOW radio station.

"Several factors indicate that KNOW radio can no longer succeed as a rock-and-roll music

station.Consider,for example,that the number of people in our listening area over fifty years of age has increased dramatically,while our total number of listeners has declined.Also,music stores in our area report decreased sales of rock-and-roll music.Finally,continuous news stations in neighboring cities have been very successful.We predict that switching KNOW radio from rock-and-roll music to 24-hour news will allow the station to attract older listeners and make KNOW radio more profitable than ever."

Write a response in which you discuss what questions would need to be answered in order to decide whether the prediction and the argument on which it is based are reasonable.Be sure to explain how the answers to these questions would help to evaluate the prediction.

若干因素表明KNOW电台应该把它的节目从摇滚乐转成连续的新闻节目。举例来说,我们覆盖范围内的老年人数量显著增加,而总的听众数量最近减少了。而且,我们地区音像店中音乐的销量下降。最后,邻近城市的连续新闻节目非常成功。从摇滚乐转型到24小时新闻节目将会吸引更多的老年听众并使KNOW电台更加盈利。

--------------------------------------------------------------------------------------------------------------------------------------------- 18)

The following is a memorandum from the business manager of a television station.

"Over the past year,our late-night news program has devoted increased time to national news and less time to weather and local news.During this period,most of the complaints received from viewers were concerned with our station's coverage of weather and local news.In addition,local businesses that used to advertise during our late-night news program have canceled their

advertising contracts with us.Therefore,in order to attract more viewers to our news programs and to avoid losing any further advertising revenues,we should expand our coverage of weather and local news on all our news programs."

Write a response in which you examine the stated and/or unstated assumptions of the

argument.Be sure to explain how the argument depends on these assumptions and what the implications are for the argument if the assumptions prove unwarranted.

去年,我们的深夜新闻节目投入了更多的时间播放全国新闻,较少的时间播放天气预报和地方新闻。在同一时期,我们从观众那里获得的投诉大部分是关于电视台对于天气和地方新闻的播放的。而且,若干曾经在我们深夜新闻节目播放广告的地方企业刚刚取消了和我们的广告契约。因此,为吸引更多的电视观众观看我们的新闻节目并避免失去未来的广告收入,我们应该在我们所有的新闻节目中增加天气和地方新闻的播报。

20)

The following is a memorandum from the business manager of a television station.

"Over the past year,our late-night news program has devoted increased time to national news and less time to weather and local news.During this time period,most of the complaints received from viewers were concerned with our station's coverage of weather and local news.In addition,local businesses that used to advertise during our late-night news program have just canceled their advertising contracts with us.Therefore,in order to attract more viewers to the program and to avoid losing any further advertising revenues,we should restore the time devoted to weather and local news to its former level."

Write a response in which you discuss what specific evidence is needed to evaluate the argument and explain how the evidence would weaken or strengthen the argument.

去年,我们的深夜新闻节目投入了更多的时间播放全国新闻,较少的时间播放天气预报和地方新闻。在同一时期,我们从观众那里获得的投诉大部分是关于电视台对于天气和地方新闻的播放的。而且,若干曾经在我们深夜新闻节目播放广告的地方企业刚刚取消了和我们的广告契约。因此,为吸引更

多的电视观众观看我们的新闻节目并避免失去未来的广告收入,我们应该在我们把天气和地方新闻的播报时候恢复到以前的水平。

--------------------------------------------------------------------------------------------------------------------------------------------- 22)

According to a recent report,cheating among college and university students is on the

rise.However,Groveton College has successfully reduced student cheating by adopting an honor code,which calls for students to agree not to cheat in their academic endeavors and to notify a faculty member if they suspect that others have cheated.Groveton's honor code replaced a system in which teachers closely monitored students;under that system,teachers reported an average of thirty cases of cheating per year.In the first year the honor code was in place,students reported twenty-one cases of cheating;five years later,this figure had dropped to fourteen.Moreover,in a recent survey,a majority of Groveton students said that they would be less likely to cheat with an honor code in place than without.Thus,all colleges and universities should adopt honor codes similar to Groveton's in order to decrease cheating among students.

Write a response in which you discuss what questions would need to be answered in order to decide whether the recommendation and the argument on which it is based are reasonable.Be sure to explain how the answers to these questions would help to evaluate the recommendation.

通过最近一次报告,大学作弊现象显著增加。然而G大学却通过采取诚信制度的方式使该校学生作弊现象减少。该制度要求同学同意在学业中不作弊,并且但他们怀疑别人作弊是告知老师。G大学的诚信制度取代了原有的老师严密监管的老体制,在老体制中平均每年有30起作弊被上报。而在实施新制度后的第一年,学生上报了21起。五年后,这一数字更是下降到14起。另外,最近一项调查显示,G大学绝大多数学生表示有了诚信制度以后比以前更不可能作弊了。因此,所有的大学都应该采取与G大学类似的诚信制度来降低学生作弊现象。

119)

Evidence suggests that academic honor codes,which call for students to agree not to cheat in their academic endeavors and to notify a faculty member if they suspect that others have cheated,are far more successful than are other methods at deterring cheating among students at colleges and universities.Several years ago,Groveton College adopted such a code and discontinued its

old-fashioned system in which teachers closely monitored students.Under the old system,teachers reported an average of thirty cases of cheating per year.In the first year the honor code was in place,students reported twenty-one cases of cheating;five years later,this figure had dropped to fourteen.Moreover,in a recent survey,a majority of Groveton students said that they would be less likely to cheat with an honor code in place than without.

Write a response in which you discuss one or more alternative explanations that could rival the proposed explanation and explain how your explanation(s)can plausibly account for the facts presented in the argument.

最近证据表明G大学的诚信制度的方式有效的使该校学生作弊现象减少。该制度要求同学同意在学业中不作弊,并且但他们怀疑别人作弊是告知老师。多年前G大学的诚信制度取代了原有的老师严密监管的老体制,在老体制中平均每年有30起作弊被上报。而在实施新制度后的第一年,学生上报了21起。五年后,这一数字更是下降到14起。另外,最近一项调查显示,G大学绝大多数学生表示有了诚信制度以后比以前更不可能作弊了。

120)

Several years ago,Groveton College adopted an honor code,which calls for students to agree not to cheat in their academic endeavors and to notify a faculty member if they suspect that others have cheated.Groveton's honor code replaced a system in which teachers closely monitored

students.Under that system,teachers reported an average of thirty cases of cheating per year.The honor code has proven far more successful:in the first year it was in place,students reported twenty-one cases of cheating;five years later,this figure had dropped to fourteen.Moreover,in a recent survey,a majority of Groveton students said that they would be less likely to cheat with an honor code in place than without.Such evidence suggests that all colleges and universities should

adopt honor codes similar to Groveton's.This change is sure to result in a dramatic decline in cheating among college students.

Write a response in which you discuss what questions would need to be answered in order to decide whether the recommendation is likely to have the predicted result.Be sure to explain how the answers to these questions would help to evaluate the recommendation.

7年前,G大学采取了诚信制度的方式,该制度要求同学同意在学业中不作弊,并且但他们怀疑别人作弊是告知老师。G大学的诚信制度取代了原有的老师严密监管的老体制,在老体制中平均每年有30起作弊被上报。而新的诚信体制却被证明成功的多,在实施新制度后的第一年,学生上报了21起。五年后,这一数字更是下降到14起。另外,最近一项调查显示,G大学绝大多数学生表示有了诚信制度以后比以前更不可能作弊了。因此,所有的大学都应该采取与G大学类似的诚信制度来降低学生作弊现象。这些证据表明所有的大学都应该采取类似于G大学的诚信制度,而且这样的制度一定会使大学作弊现象急剧下降。

138)

The following appeared as an editorial in the student newspaper of Groveton College.

"To combat the recently reported dramatic rise in cheating among college students,colleges and universities should adopt honor codes similar to Groveton's,which calls for students to agree not to cheat in their academic endeavors and to notify a faculty member if they suspect that others have cheated.Groveton's honor code replaced an old-fashioned system in which teachers closely monitored students.Under that system,teachers reported an average of thirty cases of cheating per year.The honor code has proven far more successful:in the first year it was in place,students reported twenty-one cases of cheating;five years later,this figure had dropped to

fourteen.Moreover,in a recent survey conducted by the Groveton honor council,a majority of students said that they would be less likely to cheat with an honor code in place than without."

Write a response in which you discuss what specific evidence is needed to evaluate the argument and explain how the evidence would weaken or strengthen the argument.

为了应对最近大学作弊上升的现象,大学应该采用与G大学类似的诚信制度。该制度要求同学同意在学业中不作弊,并且当他们怀疑别人作弊是告知老师。G大学的诚信制度取代了原有的老师严密监管的老体制,在老体制中平均每年有30起作弊被上报。而新的制度却成功的多,在实施新制度后的第一年,学生上报了21起。五年后,这一数字更是下降到14起。另外,最近一项调查显示,G大学绝大多数学生表示有了诚信制度以后比以前更不可能作弊了。

--------------------------------------------------------------------------------------------------------------------------------------------- 24)

A recently issued twenty-year study on headaches suffered by the residents of Mentia investigated the possible therapeutic effect of consuming salicylates.Salicylates are members of the same chemical family as aspirin,a medicine used to treat headaches.Although many foods are naturally rich in salicylates,food-processing companies also add salicylates to foods as preservatives.The twenty-year study found a correlation between the rise in the commercial use of salicylates and a steady decline in the average number of headaches reported by study participants.At the time when the study concluded,food-processing companies had just discovered that salicylates can also be used as flavor additives for foods,and,as a result,many companies plan to do so.Based on these study results,some health experts predict that residents of Mentia will suffer even fewer headaches in the future.

Write a response in which you discuss what questions would need to be answered in order to decide whether the prediction and the argument on which it is based are reasonable.Be sure to explain how the answers to these questions would help to evaluate the prediction.

最近一项20年关于M地区百姓头痛的研究正在探寻水杨酸的潜在治疗作用。水杨酸酯和用来治疗

头痛的药物--阿司匹林是同一类化合物。尽管很多食品天然富含水杨酸酯,食品加工公司仍然在食品中加入它来作为防腐剂。这种水杨酸酯在商业上的使用被发现与我们为期20年研究的参加者所报告的头痛发病的平均数量下降是相关的。但研究结束时,食品加工公司却刚刚发现水杨酸酯也可以被用作食品香料,因此很多公司准备这样添加。依据以上的研究,一些健康专家预期Mentia居患头痛症的数量将会持续稳步下降。

26)

The following appeared in the summary of a study on headaches suffered by the residents of Mentia.

"Salicylates are members of the same chemical family as aspirin,a medicine used to treat headaches.Although many foods are naturally rich in salicylates,for the past several decades,food-processing companies have also been adding salicylates to foods as

preservatives.This rise in the commercial use of salicylates has been found to correlate with a steady decline in the average number of headaches reported by participants in our twenty-year study.Recently,food-processing companies have found that salicylates can also be used as flavor additives for foods.With this new use for salicylates,we can expect a continued steady decline in the number of headaches suffered by the average citizen of Mentia."

Write a response in which you discuss what specific evidence is needed to evaluate the argument and explain how the evidence would weaken or strengthen the argument.

水杨酸酯和用来治疗头痛的药物--阿司匹林是同一类化合物。尽管很多食品天然富含水杨酸酯,过去几十年中食品加工公司仍然在食品中加入它来作为防腐剂。这种水杨酸酯在商业上的使用被发现与我们为期20年研究的参加者所报告的头痛发病的平均数量下降是相关的。最近,食品加工公司发现水杨酸酯也可以被用作食品香料。根据这种水杨酸酯的新用途,我们可以预期Mentia居患头痛症的数量将会持续稳步下降。

28)

The following appeared in the summary of a study on headaches suffered by the residents of Mentia.

"Salicylates are members of the same chemical family as aspirin,a medicine used to treat headaches.Although many foods are naturally rich in salicylates,for the past several

decades,food-processing companies have also been adding salicylates to foods as preservatives.This rise in the commercial use of salicylates has been found to correlate with a steady decline in the average number of headaches reported by participants in our twenty-year study.Recently,food-processing companies have found that salicylates can also be used as flavor additives for foods.With this new use for salicylates,we can expect a continued steady decline in the number of headaches suffered by the average citizen of Mentia."

Write a response in which you examine the stated and/or unstated assumptions of the

argument.Be sure to explain how the argument depends on these assumptions and what the implications are for the argument if the assumptions prove unwarranted.

水杨酸酯和用来治疗头痛的药物--阿司匹林是同一类化合物。尽管很多食品天然富含水杨酸酯,过去几十年中食品加工公司仍然在食品中加入它来作为防腐剂。这种水杨酸酯在商业上的使用被发现与我们为期20年研究的参加者所报告的头痛发病的平均数量下降是相关的。最近,食品加工公司发现水杨酸酯也可以被用作食品香料。根据这种水杨酸酯的新用途,我们可以预期Mentia居患头痛症的数量将会持续稳步下降。

---------------------------------------------------------------------------------------------------------------------------------------------

25)

The following was written as a part of an application for a small-business loan by a group of developers in the city of Monroe.

"A jazz music club in Monroe would be a tremendously profitable enterprise.Currently,the nearest jazz club is 65 miles away;thus,the proposed new jazz club in Monroe,the C-Note,would have the local market all to itself.Plus,jazz is extremely popular in Monroe:over 100,000 people attended Monroe's annual jazz festival last summer;several well-known jazz musicians live in Monroe;and the highest-rated radio program in Monroe is'Jazz Nightly,'which airs every weeknight at 7

P.M.Finally,a nationwide study indicates that the typical jazz fan spends close to$1,000 per year on jazz entertainment."

Write a response in which you discuss what specific evidence is needed to evaluate the argument and explain how the evidence would weaken or strengthen the argument.

在Monroe建立爵士乐俱乐部将会是非常盈利的产业。当前,最近的爵士俱乐部也在65英里以外;因此,我们筹建的俱乐部C Note将会占有全部的本地市场。而且,爵士乐在Monroe非常流行:去年夏天10万多人参加了Monroe的爵士音乐节,若干知名爵士音乐家居住在Monroe,Monroe获评价最高的广播节目是平时每天播出的Jazz Nightly。最后,一项全国性研究表明典型的爵士爱好者每年花费近1000元用于爵士娱乐。

100)

The following was written as a part of an application for a small-business loan by a group of developers in the city of Monroe.

"Jazz music is extremely popular in the city of Monroe:over 100,000 people attended Monroe's annual jazz festival last summer,and the highest-rated radio program in Monroe is'Jazz

Nightly,'which airs every weeknight.Also,a number of well-known jazz musicians own homes in Monroe.Nevertheless,the nearest jazz club is over an hour away.Given the popularity of jazz in Monroe and a recent nationwide study indicating that the typical jazz fan spends close to$1,000 per year on jazz entertainment,a jazz music club in Monroe would be tremendously profitable."

Write a response in which you examine the stated and/or unstated assumptions of the

argument.Be sure to explain how the argument depends on these assumptions and what the implications are for the argument if the assumptions prove unwarranted.

爵士乐在Monroe非常流行:去年夏天10万多人参加了Monroe的爵士音乐节。而且Monroe获评价最高的广播节目是平时每天播出的Jazz Nightly。同时,若干知名爵士音乐家居住在Monroe。但是当前,最近的爵士俱乐部也在65英里以外。鉴于目前M地区爵士音乐的流行以及一项全国性研究表明典型的爵士爱好者每年花费近1000元用于爵士娱乐。因此,在Monroe建立爵士乐俱乐部将会是非常盈利的产业。

102)

The following was written as a part of an application for a small-business loan by a group of developers in the city of Monroe.

"Jazz music is extremely popular in the city of Monroe:over 100,000 people attended Monroe's annual jazz festival last summer,and the highest-rated radio program in Monroe is'Jazz

Nightly,'which airs every weeknight.Also,a number of well-known jazz musicians own homes in Monroe.Nevertheless,the nearest jazz club is over an hour away.Given the popularity of jazz in Monroe and a recent nationwide study indicating that the typical jazz fan spends close to$1,000 per year on jazz entertainment,we predict that our new jazz music club in Monroe will be a tremendously profitable enterprise."

Write a response in which you discuss what questions would need to be answered in order to decide whether the prediction and the argument on which it is based are reasonable.Be sure to explain how the answers to these questions would help to evaluate the prediction.

爵士乐在Monroe非常流行:去年夏天10万多人参加了Monroe的爵士音乐节。而且Monroe获评价最高的广播节目是平时每天播出的Jazz Nightly。同时,若干知名爵士音乐家居住在Monroe。但是

当前,最近的爵士俱乐部也在65英里以外。鉴于目前M地区爵士音乐的流行以及一项全国性研究表明典型的爵士爱好者每年花费近1000元用于爵士娱乐。因此,在Monroe建立爵士乐俱乐部将会是非常盈利的产业。

164)

The following was written by a group of developers in the city of Monroe.

"A jazz music club in Monroe would be a tremendously profitable enterprise.At present,the nearest jazz club is over 60 miles away from Monroe;thus,our proposed club,the C Note,would have the local market all to itself.In addition,there is ample evidence of the popularity of jazz in Monroe:over 100,000 people attended Monroe's jazz festival last summer,several well-known jazz musicians live in Monroe,and the highest-rated radio program in Monroe is'Jazz Nightly.'Finally,a nationwide study indicates that the typical jazz fan spends close to$1,000 per year on jazz entertainment.We therefore predict that the C Note cannot help but make money."

Write a response in which you discuss what questions would need to be answered in order to decide whether the prediction and the argument on which it is based are reasonable.Be sure to explain how the answers to these questions would help to evaluate the prediction.

在Monroe建立爵士乐俱乐部将会是非常盈利的产业。当前,最近的爵士俱乐部也在65英里以外;因此,我们筹建的俱乐部C Note将会占有全部的本地市场。而且,爵士乐在Monroe非常流行:去年夏天10万多人参加了Monroe的爵士音乐节,若干知名爵士音乐家居住在Monroe,Monroe获评价最高的广播节目是平时每天播出的Jazz Nightly。最后,一项全国性研究表明典型的爵士爱好者每年花费近1000元用于爵士娱乐。显然C Note将会。

---------------------------------------------------------------------------------------------------------------------------------------------

27)

The following appeared in a letter to the editor of a local newspaper.

"Commuters complain that increased rush-hour traffic on Blue Highway between the suburbs and the city center has doubled their commuting time.The favored proposal of the motorists'lobby is to widen the highway,adding an additional lane of traffic.But last year's addition of a lane to the nearby Green Highway was followed by a worsening of traffic jams on it.A better alternative is to add a bicycle lane to Blue Highway.Many area residents are keen bicyclists.A bicycle lane would encourage them to use bicycles to commute,and so would reduce rush-hour traffic rather than fostering an increase."

Write a response in which you discuss what specific evidence is needed to evaluate the argument and explain how the evidence would weaken or strengthen the argument.

乘客抱怨在郊区与市中心的B高速公路的更长交通高峰期使他们在路上的时间多花了一倍。一个驾驶协会的不错建议是拓宽高速公路,修建一条附加机动车道。但去年但临近的G高速采用了相同的方法后交通状况更加恶化。一种更好的建议是在B高速上修一条自行车道。很多的当地人都是自行车爱好者。一条新的自行车道将会激励他们骑自行车外出,这样的方式照比驾驶协会的建议会减少高峰时间而不是延长。

29)

The following appeared in an editorial in a local newspaper.

"Commuters complain that increased rush-hour traffic on Blue Highway between the suburbs and the city center has doubled their commuting time.The favored proposal of the motorists'lobby is to widen the highway,adding an additional lane of traffic.Opponents note that last year's addition of a lane to the nearby Green Highway was followed by a worsening of traffic jams on it.Their

suggested alternative proposal is adding a bicycle lane to Blue Highway.Many area residents are keen bicyclists.A bicycle lane would encourage them to use bicycles to commute,it is

argued,thereby reducing rush-hour traffic."

Write a response in which you discuss what questions would need to be answered in order to decide whether the recommendation and the argument on which it is based are reasonable.Be sure to explain how the answers to these questions would help to evaluate the recommendation.

乘客抱怨在郊区与市中心的B高速公路的更长交通高峰期使他们在路上的时间多花了一倍。一个驾驶协会的不错建议是拓宽高速公路,修建一条附加机动车道。但反对者却说去年临近的G高速采用了相同的方法后交通状况更加恶化。他们建议一种更好的建议是在B高速上修一条自行车道。很多的当地人都是自行车爱好者。一条新的自行车道将会激励他们骑自行车外出,这样的方式会减少高峰时间的交通流量。 --------------------------------------------------------------------------------------------------------------------------------------------- 30)

The following appeared as a recommendation by a committee planning a ten-year budget for the city of Calatrava.

"The birthrate in our city is declining:in fact,last year's birthrate was only one-half that of five years ago.Thus the number of students enrolled in our public schools will soon decrease dramatically,and we can safely reduce the funds budgeted for education during the next decade.At the same time,we can reduce funding for athletic playing fields and other recreational facilities.As a result,we will have sufficient money to fund city facilities and programs used primarily by adults,since we can expect the adult population of the city to increase."

Write a response in which you discuss what specific evidence is needed to evaluate the argument and explain how the evidence would weaken or strengthen the argument.

我们市的出生率正在降低:事实上,去年的出生率只是5年前的一半。因此学生的入学人数将会急剧下降,我们可以放心的削减未来10年的教育经费。与此同时,我们也可以减少对操场和其他活动设施的经费。所以,我们可以有充足的费用去扶持城市设施及其他的成人项目,因为我们预测未来城市里面的成人人口将会增加。

--------------------------------------------------------------------------------------------------------------------------------------------- 31)

The following appeared in a letter to the editor of Parson City's local newspaper.

"In our region of Trillura,the majority of money spent on the schools that most students attend—the city-run public schools—comes from taxes that each city government collects.The region's cities differ,however,in the budgetary priority they give to public education.For example,both as a

proportion of its overall tax revenues and in absolute terms,Parson City has recently spent almost twice as much per year as Blue City has for its public schools—even though both cities have about the same number of residents.Clearly,Parson City residents place a higher value on providing a good education in public schools than Blue City residents do."

Write a response in which you discuss what specific evidence is needed to evaluate the argument and explain how the evidence would weaken or strengthen the argument.

在我们t地区,政府用于公立学校教育的开支大部分都是从各地市政府征收的税收而来的。然而,依据总体税收比率及相关政策来看,该地区不同城市对于公立教育的重视程度是不同的。例如,p城市用于公立学校的预算是b市的2倍,尽管两个城市居民的数量基本相同。所以,p城市百姓显然比b市居民更加关注公立教育。

--------------------------------------------------------------------------------------------------------------------------------------------- 32)

The following appeared in a memo from a vice president of Quiot Manufacturing.

"During the past year,Quiot Manufacturing had 30 percent more on-the-job accidents than at the nearby Panoply Industries plant,where the work shifts are one hour shorter than ours.Experts say

that significant contributing factors in many on-the-job accidents are fatigue and sleep deprivation among workers.Therefore,to reduce the number of on-the-job accidents at Quiot and thereby increase productivity,we should shorten each of our three work shifts by one hour so that employees will get adequate amounts of sleep."

Write a response in which you examine the stated and/or unstated assumptions of the

argument.Be sure to explain how the argument depends on these assumptions and what the implications are for the argument if the assumptions prove unwarranted.

在过去,Q工厂的工伤事故比邻近的Panoply工厂多30%,Panoply的每班工作时间比我们的短一个小时。专家相信导致很多工伤事故的重要因素就是工人的疲劳和睡眠不足。因此,为减少Q的工伤事故数量并提高生产效率,我们应该把我们的三个班次的工作时间每个都减少一小时,从而我们的雇员将有更充足的睡眠。

104)

The following appeared in a memo from a vice president of a manufacturing company.

"During the past year,workers at our newly opened factory reported 30 percent more on-the-job accidents than workers at nearby Panoply Industries.Panoply produces products very similar to those produced at our factory,but its work shifts are one hour shorter than ours.Experts say that fatigue and sleep deprivation among workers are significant contributing factors in many

on-the-job accidents.Panoply's superior safety record can therefore be attributed to its shorter work shifts,which allow its employees to get adequate amounts of rest."

Write a response in which you discuss one or more alternative explanations that could rival the proposed explanation and explain how your explanation(s)can plausibly account for the facts presented in the argument.

在过去,Q工厂的工伤事故比邻近的Panoply工厂多30%,P与Q都生产类似的产品,但Panoply的每班工作时间比我们的短一个小时。专家相信导致很多工伤事故的重要因素就是工人的疲劳和睡眠不足。因此,P的更高安全记录是因为其更短每班工作时间,从而使其雇员将有更充足的休息。 105)

The following appeared in a memo from the vice president of Butler Manufacturing.

"During the past year,workers at Butler Manufacturing reported 30 percent more on-the-job

accidents than workers at nearby Panoply Industries,where the work shifts are one hour shorter than ours.A recent government study reports that fatigue and sleep deprivation among workers are significant contributing factors in many on-the-job accidents.If we shorten each of our work shifts by one hour,we can improve Butler Manufacturing's safety record by ensuring that our employees are adequately rested."

Write a response in which you discuss what specific evidence is needed to evaluate the argument and explain how the evidence would weaken or strengthen the argument.

在过去,B工厂的工伤事故比邻近的Panoply工厂多30%,Panoply的每班工作时间比我们的短一个小时。最近一项政府调查显示很多工伤事故的重要因素就是工人的疲劳和睡眠不足。因此,如果我们把每个班次的工作时间每个都减少一小时,我们的雇员将有更充足的睡眠,这将会改善我们的安全生产状况。 106)

The following appeared in a memo from the Board of Directors of Butler Manufacturing.

"During the past year,workers at Butler Manufacturing reported 30 percent more on-the-job

accidents than workers at nearby Panoply Industries,where the work shifts are one hour shorter than ours.A recent government study reports that fatigue and sleep deprivation among workers are significant contributing factors in many on-the-job accidents.Therefore,we recommend that

Butler Manufacturing shorten each of its work shifts by one hour.Shorter shifts will allow Butler to improve its safety record by ensuring that its employees are adequately rested."

Write a response in which you discuss what questions would need to be answered in order to decide whether the recommendation is likely to have the predicted result.Be sure to explain how the answers to these questions would help to evaluate the recommendation.

在过去,B工厂的工伤事故比邻近的Panoply工厂多30%,Panoply的每班工作时间比我们的短一个小时。最近一项政府调查显示很多工伤事故的重要因素就是工人的疲劳和睡眠不足。因此,如果我们把每个班次的工作时间每个都减少一小时,我们的雇员将有更充足的睡眠,这将会改善我们的安全生产状况。

167)

The following appeared in a memo from a vice president of Alta Manufacturing.

"During the past year,Alta Manufacturing had thirty percent more on-the-job accidents than nearby Panoply Industries,where the work shifts are one hour shorter than ours.Experts believe that a significant contributing factor in many accidents is fatigue caused by sleep deprivation among workers.Therefore,to reduce the number of on-the-job accidents at Alta,we recommend shortening each of our three work shifts by one hour.If we do this,our employees will get adequate amounts of sleep."

Write a response in which you discuss what questions would need to be answered in order to decide whether the recommendation and the argument on which it is based are reasonable.Be sure to explain how the answers to these questions would help to evaluate the recommendation.

去年,A工厂的工伤事故比邻近的Panoply工厂多30%,Panoply的每班工作时间比我们的短一个小时。专家们认为示很多工伤事故的重要因素就是工人的疲劳和睡眠不足。因此,为了减少A的工伤事故,我们建议A把每个班次的工作时间每个都减少一小时,这样做会使我们的雇员有更充足的睡眠。

--------------------------------------------------------------------------------------------------------------------------------------------- 33)

The following appeared in a memorandum from the planning department of an electric power company.

"Several recent surveys indicate that home owners are increasingly eager to conserve energy.At the same time,manufacturers are now marketing many home appliances,such as refrigerators and air conditioners,that are almost twice as energy efficient as those sold a decade ago.Also,new technologies for better home insulation and passive solar heating are readily available to reduce the energy needed for home heating.Therefore,the total demand for electricity in our area will not increase—and may decline slightly.Since our three electric generating plants in operation for the past twenty years have always met our needs,construction of new generating plants will not be necessary."

Write a response in which you examine the stated and/or unstated assumptions of the

argument.Be sure to explain how the argument depends on these assumptions and what the implications are for the argument if the assumptions prove unwarranted.

一些最近的调查表明,房主越来越强烈地希望节省能源,并且生产商现在正在推出很多比十年前的电器几乎节能两倍的家用电器,比如冰箱和空调,这些电器要比10前出售的节能50%。而且,更好的房屋隔热和被动式太阳能采暖的新技术已经可以用于减少家庭采暖所需的能源。因此,我们预计我们地区的用电需求总量不会增加,而可能有轻微下降。由于我们的已经运作了20年的三座发电站总能够满足需求,我们无需建造新的发电厂。

--------------------------------------------------------------------------------------------------------------------------------------------- 34)

The vice president of human resources at Climpson Industries sent the following recommendation to the company's president.

"In an effort to improve our employees'productivity,we should implement electronic monitoring of employees'Internet use from their workstations.Employees who use the Internet from their workstations need to be identified and punished if we are to reduce the number of work hours spent on personal or recreational activities,such as shopping or playing games.By installing software to detect employees'Internet use on company computers,we can prevent employees from wasting time,foster a better work ethic at Climpson,and improve our overall profits."

Write a response in which you examine the stated and/or unstated assumptions of the

argument.Be sure to explain how the argument depends on these assumptions and what the implications are for the argument if the assumptions prove unwarranted.

为提高我们的员工的生产效率,我们应该在员工的电脑上加装电子监控来监视员工对互联网的使用。如果我们要减少用于私人以及娱乐活动,比如购物或玩游戏的工作时间,那些在电脑上不正当使用互联网的员工必须要被确认并且受到惩罚。通过在公司电脑上安装检测员工使用互联网的软件,我们可以防止员工浪费时间,并培养C公司更好的工作氛围,以及提高我们的整体利润。

58)

The vice president for human resources at Climpson Industries sent the following recommendation to the company's president.

"In an effort to improve our employees'productivity,we should implement electronic monitoring of employees'Internet use from their workstations.Employees who use the Internet inappropriately from their workstations need to be identified and punished if we are to reduce the number of work hours spent on personal or recreational activities,such as shopping or playing games.Installing software on company computers to detect employees'Internet use is the best way to prevent employees from wasting time on the job.It will foster a better work ethic at Climpson and improve our overall profits."

Write a response in which you discuss what specific evidence is needed to evaluate the argument and explain how the evidence would weaken or strengthen the argument.

为提高我们的员工的生产效率,我们应该在员工的电脑上加装电子监控来监视员工对互联网的使用。如果我们要减少用于私人以及娱乐活动,比如购物或玩游戏的工作时间,那么在电脑上不正当使用互联网的员工必须要被确认并且受到惩罚。通过在公司电脑上安装检测员工使用互联网的软件,我们可以防止员工浪费时间,并培养C公司更好的工作氛围,以及提高我们的整体利润。

94)

The vice president of human resources at Climpson Industries sent the following recommendation to the company's president.

"A recent national survey found that the majority of workers with access to the Internet at work had used company computers for personal or recreational activities,such as banking or playing games.In an effort to improve our employees'productivity,we should implement electronic monitoring of employees'Internet use from their workstations.Using electronic monitoring software is the best way to reduce the number of hours Climpson employees spend on personal or

recreational activities.We predict that installing software to monitor employees'Internet use will allow us to prevent employees from wasting time,thereby increasing productivity and improving overall profits."

Write a response in which you discuss what questions would need to be answered in order to decide whether the prediction and the argument on which it is based are reasonable.Be sure to explain how the answers to these questions would help to evaluate the prediction.

最近一项全国调查显示绝大部分员工在上班期间用单位电脑上网从事私人及娱乐活动。例如理财或玩游戏。为提高我们的员工的生产效率,我们应该在员工的电脑上加装电子监控来监视员工对互联网的使用。安装监视软件是最好的方法来减少用于私人以及娱乐活动。我们预计通过在公司电脑上安装检测员工使用互联网的软件,我们可以防止员工浪费时间,并培养C公司更好的工作氛围,以及提高我们的整体利润。

--------------------------------------------------------------------------------------------------------------------------------------------- 35)

The following appeared in a letter from the owner of the Sunnyside Towers apartment complex to its manager.

"One month ago,all the showerheads in the first three buildings of the Sunnyside Towers complex were modified to restrict maximum water flow to one-third of what it used to be.Although actual readings of water usage before and after the adjustment are not yet available,the change will obviously result in a considerable savings for Sunnyside Corporation,since the corporation must pay for water each month.Except for a few complaints about low water pressure,no problems with showers have been reported since the adjustment.I predict that modifying showerheads to restrict water flow throughout all twelve buildings in the Sunnyside Towers complex will increase our profits even more dramatically."

Write a response in which you discuss what questions would need to be answered in order to decide whether the prediction and the argument on which it is based are reasonable.Be sure to explain how the answers to these questions would help to evaluate the prediction.

一个月前,Sunnyside塔楼最低的五层的所有淋浴喷头被调节成水压只有以前的大约三分之一。尽管在调节之后用水量的确切读数还没有出来,但这种变革显然将为Sunnyside公司节省大量的花费,因为公司必须每月为所用的水付费。除了关于低水压的几起投诉,在调节之后没有发生关于淋浴喷头的问题的报告。我预计在Sunnyside塔楼所有的12层都限制水压将会增加我们未来的利润。 52)

The following appeared in a letter from the owner of the Sunnyside Towers apartment building to its manager.

"One month ago,all the showerheads on the first five floors of Sunnyside Towers were modified to restrict the water flow to approximately one-third of its original flow.Although actual readings of water usage before and after the adjustment are not yet available,the change will obviously result in a considerable savings for Sunnyside Corporation,since the corporation must pay for water

each month.Except for a few complaints about low water pressure,no problems with showers have been reported since the adjustment.Clearly,restricting water flow throughout all the twenty floors of Sunnyside Towers will increase our profits further."

Write a response in which you discuss what questions would need to be answered in order to decide whether the recommendation is likely to have the predicted result.Be sure to explain how the answers to these questions would help to evaluate the recommendation.

一个月前,Sunnyside塔楼最低的五层的所有淋浴喷头被调节成水压只有以前的大约三分之一。尽管在调节之后用水量的确切读数还没有出来,但这种变革显然将为Sunnyside公司节省大量的花费,因为公司必须每月为所用的水付费。除了关于低水压的几起投诉,在调节之后没有发生关于淋浴喷头的问题的报告。我预计在Sunnyside塔楼所有的12层都限制水压将会增加我们未来的利润。

128)

The following appeared in a letter from the owner of the Sunnyside Towers apartment complex to its manager.

"One month ago,all the showerheads in the first three buildings of the Sunnyside Towers complex

were modified to restrict maximum water flow to one-third of what it used to be.Although actual readings of water usage before and after the adjustment are not yet available,the change will obviously result in a considerable savings for Sunnyside Corporation,since the corporation must pay for water each month.Except for a few complaints about low water pressure,no problems with showers have been reported since the adjustment.Clearly,modifying showerheads to restrict water flow throughout all twelve buildings in the Sunnyside Towers complex will increase our profits further."

Write a response in which you discuss what specific evidence is needed to evaluate the argument and explain how the evidence would weaken or strengthen the argument.

一个月前,Sunnyside塔楼最低的五层的所有淋浴喷头被调节成水压只有以前的大约三分之一。尽管在调节之后用水量的确切读数还没有出来,但这种变革显然将为Sunnyside公司节省大量的花费,因为公司必须每月为所用的水付费。除了关于低水压的几起投诉,在调节之后没有发生关于淋浴喷头的问题的报告。我预计在Sunnyside塔楼所有的12层都限制水压将会增加我们未来的利润。

129)

The following appeared in a letter from the owner of the Sunnyside Towers apartment complex to its manager.

"Last week,all the showerheads in the first three buildings of the Sunnyside Towers complex were modified to restrict maximum water flow to one-third of what it used to be.Although actual readings of water usage before and after the adjustment are not yet available,the change will obviously result in a considerable savings for Sunnyside Corporation,since the corporation must pay for water each month.Except for a few complaints about low water pressure,no problems with

showers have been reported since the adjustment.Clearly,modifying showerheads to restrict water flow throughout all twelve buildings in the Sunnyside Towers complex will increase our profits further."

Write a response in which you examine the stated and/or unstated assumptions of the

argument.Be sure to explain how the argument depends on these assumptions and what the implications are for the argument if the assumptions prove unwarranted.

一个月前,Sunnyside塔楼最低的五层的所有淋浴喷头被调节成水压只有以前的大约三分之一。尽管在调节之后用水量的确切读数还没有出来,但这种变革显然将为Sunnyside公司节省大量的花费,因为公司必须每月为所用的水付费。除了关于低水压的几起投诉,在调节之后没有发生关于淋浴喷头的问题的报告。我预计在Sunnyside塔楼所有的12层都限制水压将会增加我们未来的利润。

--------------------------------------------------------------------------------------------------------------------------------------------- 36)

The following report appeared in the newsletter of the West Meria Public Health Council.

"An innovative treatment has come to our attention that promises to significantly reduce

absenteeism in our schools and workplaces.A study reports that in nearby East Meria,where fish consumption is very high,people visit the doctor only once or twice per year for the treatment of colds.Clearly,eating a substantial amount of fish can prevent colds.Since colds represent the most frequently given reason for absences from school and work,we recommend the daily use of

Ichthaid—a nutritional supplement derived from fish oil—as a good way to prevent colds and lower absenteeism."

Write a response in which you discuss what specific evidence is needed to evaluate the argument and explain how the evidence would weaken or strengthen the argument.

我们注意到一种新的可以保证显著减少学校和工作岗位缺席的疗法。一项研究显示在附近的EM,鱼

的消费量很高,那的百姓每年因为感冒去看医生的次数每年只有一两次。这表明吃大量的鱼很明显可以预防感冒。另外,由于感冒是缺课和缺席的最常见原因,我们建议每天服用从鱼油提炼出的营养物以来有效的预防感冒和减少缺席。

163)

The following memo appeared in the newsletter of the West Meria Public Health Council.

"An innovative treatment has come to our attention that promises to significantly reduce

absenteeism in our schools and workplaces.A study reports that in nearby East Meria,where

consumption of the plant beneficia is very high,people visit the doctor only once or twice per year for the treatment of colds.Clearly,eating a substantial amount of beneficia can prevent colds.Since colds are the reason most frequently given for absences from school and work,we recommend the daily use of nutritional supplements derived from beneficia.We predict this will dramatically reduce absenteeism in our schools and workplaces."

Write a response in which you discuss what questions would need to be answered in order to decide whether the recommendation is likely to have the predicted result.Be sure to explain how the answers to these questions would help to evaluate the recommendation.

我们注意到一种新的可以保证显著减少学校和工作岗位缺席的疗法。一项研究显示在附近的EM,鱼的消费量很高,那的百姓每年因为感冒去看医生的次数每年只有一两次。这表明吃大量的鱼很明显可以预防感冒。另外,由于感冒是缺课和缺席的最常见原因,所以我们建议每天服用从鱼油提炼出的营养物以来有效的预防感冒和减少缺席。我们预计出席率也将会提升。

166)

The following memo appeared in the newsletter of the West Meria Public Health Council.

"An innovative treatment has come to our attention that promises to significantly reduce

absenteeism in our schools and workplaces.A study reports that in nearby East Meria,where fish consumption is very high,people visit the doctor only once or twice per year for the treatment of colds.This shows that eating a substantial amount of fish can clearly prevent

colds.Furthermore,since colds are the reason most frequently given for absences from school and work,attendance levels will improve.Therefore,we recommend the daily use of a nutritional supplement derived from fish oil as a good way to prevent colds and lower absenteeism."

Write a response in which you discuss what questions would need to be answered in order to decide whether the recommendation and the argument on which it is based are reasonable.Be sure to explain how the answers to these questions would help to evaluate the recommendation.

我们注意到一种新的可以保证显著减少学校和工作岗位缺席的疗法。一项研究显示在附近的EM,鱼的消费量很高,那的百姓每年因为感冒去看医生的次数每年只有一两次。这表明吃大量的鱼很明显可以预防感冒。另外,由于感冒是缺课和缺席的最常见原因,所以出席率也将会提升。因此,我们建议每天服用从鱼油提炼出的营养物以来有效的预防感冒和减少缺席。

--------------------------------------------------------------------------------------------------------------------------------------------- 37)

The following appeared in a recommendation from the planning department of the city of Transopolis.

"Ten years ago,as part of a comprehensive urban renewal program,the city of Transopolis adapted for industrial use a large area of severely substandard housing near the

freeway.Subsequently,several factories were constructed there,crime rates in the area

declined,and property tax revenues for the entire city increased.To further revitalize the city,we should now take similar action in a declining residential area on the opposite side of the city.Since some houses and apartments in existing nearby neighborhoods are currently

unoccupied,alternate housing for those displaced by this action will be readily available."

Write a response in which you discuss what specific evidence is needed to evaluate the argument and explain how the evidence would weaken or strengthen the argument.

十年前,作为一项综合城市改造项目的一部分,Transopolis市为了工业用途在快速路旁边改造了一大片严重低于标准的住房。随后,那里建立了若干工厂,犯罪率降低了,全市资产税收入增加。为进一步使城市重新繁荣,我们应该在城市另一侧的正在衰落的居住区采取类似的行动。由于现有邻近社区的一些住宅和公寓无人居住,这一行动产生的搬迁户的住所将有所保障。 --------------------------------------------------------------------------------------------------------------------------------------------- 38)

The following appeared in a memo from the new vice president of Sartorian,a company that manufactures men's clothing.

"Five years ago,at a time when we had difficulties in obtaining reliable supplies of high quality wool fabric,we discontinued production of our alpaca overcoat.Now that we have a new fabric supplier,we should resume production.This coat should sell very well:since we have not offered an alpaca overcoat for five years and since our major competitor no longer makes an alpaca

overcoat,there will be pent-up customer demand.Also,since the price of most types of clothing has increased in each of the past five years,customers should be willing to pay significantly higher prices for alpaca overcoats than they did five years ago,and our company profits will increase."

Write a response in which you discuss what specific evidence is needed to evaluate the argument and explain how the evidence would weaken or strengthen the argument.

五年前,当我们在获取可靠的高质量羊毛毛料的渠道方面存在困难的时候,我们停止了高档羊毛外衣的生产。现在有了新的毛料供应商,我们应该重新开始生产。这种外衣应该会卖的很好:因为我们已经五年没有供应羊毛外衣了,而且由于我们的主要竞争对手已不再生产羊毛外衣,消费者将有很迫切的需求。而且,由于过去五年中多数种类的服装价格每年都在上涨,消费者应该愿意花比五年前高的多的价格购买羊毛外衣,从而我们公司的利润将会上升。

95)

The following appeared in a memo from the new vice president of Sartorian,a company that manufactures men's clothing.

"Five years ago,at a time when we had difficulty obtaining reliable supplies of high-quality wool fabric,we discontinued production of our popular alpaca overcoat.Now that we have a new fabric supplier,we should resume production.Given the outcry from our customers when we discontinued this product and the fact that none of our competitors offers a comparable product,we can expect pent-up consumer demand for our alpaca coats.This demand and the overall increase in clothing prices will make Sartorian's alpaca overcoats more profitable than ever before."

Write a response in which you examine the stated and/or unstated assumptions of the

argument.Be sure to explain how the argument depends on these assumptions and what the implications are for the argument if the assumptions prove unwarranted.

五年前,当我们在获取可靠的高质量羊毛毛料的渠道方面存在困难的时候,我们停止了高档羊毛外衣的生产。现在有了新的毛料供应商,我们应该重新开始生产。这种外衣应该会卖的很好:因为我们已经五年没有供应羊毛外衣了,而且由于我们的主要竞争对手已不再生产羊毛外衣,消费者将有很迫切的需求。而且,由于过去五年中多数种类的服装价格每年都在上涨,消费者应该愿意花比五年前高的多的价格购买羊毛外衣,从而我们S公司的利润将会上升。

96)

The following appeared in a memo from the new vice president of Sartorian,a company that manufactures men's clothing.

"Five years ago,at a time when we had difficulty obtaining reliable supplies of high-quality wool fabric,we discontinued production of our popular alpaca overcoat.Now that we have a new fabric

supplier,we should resume production.Given the outcry from our customers when we discontinued this product and the fact that none of our competitors offers a comparable product,we can expect pent-up consumer demand for our alpaca coats.Due to this demand and the overall increase in clothing prices,we can predict that Sartorian's alpaca overcoats will be more profitable than ever before."

Write a response in which you discuss what questions would need to be answered in order to decide whether the prediction and the argument on which it is based are reasonable.Be sure to explain how the answers to these questions would help to evaluate the prediction.

五年前,当我们在获取可靠的高质量羊毛毛料的渠道方面存在困难的时候,我们停止了高档羊毛外衣的生产。现在有了新的毛料供应商,我们应该重新开始生产。这种外衣应该会卖的很好:因为我们已经五年没有供应羊毛外衣了,而且由于我们的主要竞争对手已不再生产羊毛外衣,消费者将有很迫切的需求。而且,由于过去五年中多数种类的服装价格每年都在上涨,消费者应该愿意花比五年前高的多的价格购买羊毛外衣,从而我们S公司的利润将会上升。

--------------------------------------------------------------------------------------------------------------------------------------------- 39)

A recent sales study indicates that consumption of seafood dishes in Bay City restaurants has increased by 30 percent during the past five years.Yet there are no currently operating city restaurants whose specialty is seafood.Moreover,the majority of families in Bay City are

two-income families,and a nationwide study has shown that such families eat significantly fewer home-cooked meals than they did a decade ago but at the same time express more concern about healthful eating.Therefore,the new Captain Seafood restaurant that specializes in seafood should be quite popular and profitable.

Write a response in which you discuss what specific evidence is needed to evaluate the argument and explain how the evidence would weaken or strengthen the argument.

最近的销量调查显示Bay City餐馆的海鲜菜肴的消费量比过去五年增加了30%。而现在该市还没有专门经营海鲜菜的餐厅。而且,Bay City的大多数家庭是双收入家庭,一次国家调查显示这类家庭在家做饭的数量比十年前显著减少,同时他们更关注健康饮食。因此,在Bay City开设一家新的专营海鲜食品的C餐馆将会非常受欢迎而且有利可图。 174)

A recent sales study indicates that consumption of seafood dishes in Bay City restaurants has increased by 30 percent during the past five years.Yet there are no currently operating city restaurants whose specialty is seafood.Moreover,the majority of families in Bay City are two-income families,and a nationwide study has shown that such families eat significantly fewer home-cooked meals than they did a decade ago but at the same time express more concern about healthful eating.Therefore,the new Captain Seafood restaurant that specializes in seafood should be quite popular and profitable.

Write a response in which you discuss what questions would need to be addressed in order to decide whether the conclusion and the argument on which it is based are reasonable.Be sure to explain how the answers to the questions would help to evaluate the conclusion.

最近的销量调查显示Bay City餐馆的海鲜菜肴的消费量比过去五年增加了30%。而现在该市还没有专门经营海鲜菜的餐厅。而且,Bay City的大多数家庭是双收入家庭,一次国家调查显示这类家庭在家做饭的数量比十年前显著减少,同时他们更关注健康饮食。因此,在Bay City开设一家新的专营海鲜食品的C餐馆将会非常受欢迎而且有利可图。

---------------------------------------------------------------------------------------------------------------------------------------------

40)

Milk and dairy products are rich in vitamin D and calcium—substances essential for building and maintaining bones.Many people therefore say that a diet rich in dairy products can help prevent

osteoporosis,a disease that is linked to both environmental and genetic factors and that causes the bones to weaken significantly with age.But a long-term study of a large number of people

found that those who consistently consumed dairy products throughout the years of the study have a higher rate of bone fractures than any other participants in the study.Since bone fractures are symptomatic of osteoporosis,this study result shows that a diet rich in dairy products may actually increase,rather than decrease,the risk of osteoporosis.

Write a response in which you discuss what specific evidence is needed to evaluate the argument and explain how the evidence would weaken or strengthen the argument.

牛奶和奶制品富含维生素D和钙,这是骨骼生长和维持所必须的物质。因此很多人相信多吃奶制品的饮食可以帮助预防骨质疏松症,这是一种骨骼随年龄而显著弱化而且与环境和基因因素相关联的疾病。但是一项对大量人群的长期研究发现那些在研究期间经常食用奶制品的人骨折发病率比其他参加研究的人要高。由于骨折是骨质疏松症的症状之一,这一研究结果表明富含奶制品的饮食实际上会增加而不是减少患骨质疏松症的危险

--------------------------------------------------------------------------------------------------------------------------------------------- 41)

The following appeared in a health newsletter.

"A ten-year nationwide study of the effectiveness of wearing a helmet while bicycling indicates that ten years ago,approximately 35 percent of all bicyclists reported wearing helmets,whereas today that number is nearly 80 percent.Another study,however,suggests that during the same ten-year period,the number of bicycle-related accidents has increased 200 percent.These results demonstrate that bicyclists feel safer because they are wearing helmets,and they take more risks as a result.Thus,to reduce the number of serious injuries from bicycle accidents,the government should concentrate more on educating people about bicycle safety and less on encouraging or requiring bicyclists to wear helmets."

Write a response in which you examine the stated and/or unstated assumptions of the

argument.Be sure to explain how the argument depends on these assumptions and what the implications are for the argument if the assumptions prove unwarranted.

The following is a letter to the head of the tourism bureau on the island of Tria.

一项为期十年的对于在骑自行车时戴头盔的作用的研究显示,10年前,所有骑车人中大约有35%报告说他们戴头盔,现在这个比例接近80%。然而另外一项调查显示在相同的十年期间,因骑车引起的事故数量增加了200%。这些结果说明骑车人因为戴着头盔而感到更安全,因而导致他们面临更大的危险。因此,为减少自行车事故中严重伤害的数量,政府应该更多地对人们加强自行车安全的教育,并提供鼓励或要求骑车人戴头盔的课程。

123)

The following appeared in a health newsletter.

"A ten-year nationwide study of the effectiveness of wearing a helmet while bicycling indicates that ten years ago,approximately 35 percent of all bicyclists reported wearing helmets,whereas today that number is nearly 80 percent.Another study,however,suggests that during the same ten-year period,the number of accidents caused by bicycling has increased 200 percent.These results demonstrate that bicyclists feel safer because they are wearing helmets,and they take more risks as a result.Thus,there is clearly a call for the government to strive to reduce the number of serious injuries from bicycle accidents by launching an education program that concentrates on the factors other than helmet use that are necessary for bicycle safety."

Write a response in which you discuss what questions would need to be answered in order to decide whether the recommendation and the argument on which it is based are reasonable.Be sure to explain how the answers to these questions would help to evaluate the recommendation.

一项为期十年的对于在骑自行车时戴头盔的作用的研究显示,10年前,所有骑车人中大约有35%报告说他们戴头盔,现在这个比例接近80%。然而另外一项调查显示在相同的十年期间,因骑车引起的事故数量增加了200%。这些结果说明骑车人因为戴着头盔而感到更安全,因而导致他们面临更大的危险。因此,为减少自行车事故中严重伤害的数量,政府应该更多地对人们加强自行车安全的教育,并非要求骑车人戴头盔。

125)

The following appeared in a health newsletter.

"A ten-year nationwide study of the effectiveness of wearing a helmet while bicycling indicates that ten years ago,approximately 35 percent of all bicyclists reported wearing helmets,whereas today that number is nearly 80 percent.Another study,however,suggests that during the same ten-year period,the number of accidents caused by bicycling has increased 200 percent.These results demonstrate that bicyclists feel safer because they are wearing helmets,and they take more risks as a result.Thus there is clearly a call for the government to strive to reduce the number of serious injuries from bicycle accidents by launching an education program that concentrates on the factors other than helmet use that are necessary for bicycle safety."

Write a response in which you discuss what specific evidence is needed to evaluate the argument and explain how the evidence would weaken or strengthen the argument.

一项为期十年的对于在骑自行车时戴头盔的作用的研究显示,10年前,所有骑车人中大约有35%报告说他们戴头盔,现在这个比例接近80%。然而另外一项调查显示在相同的十年期间,因骑车引起的事故数量增加了200%。这些结果说明骑车人因为戴着头盔而感到更安全,因而导致他们面临更大的危险。因此,为减少自行车事故中严重伤害的数量,政府应该更多地对人们加强自行车安全的教育,并非要求骑车人戴头盔。

142)

Hospital statistics regarding people who go to the emergency room after roller-skating accidents indicate the need for more protective equipment.Within that group of people,75 percent of those who had accidents in streets or parking lots had not been wearing any protective

clothing(helmets,knee pads,etc.)or any light-reflecting material(clip-on lights,glow-in-the-dark wrist pads,etc.).Clearly,the statistics indicate that by investing in high-quality protective gear and reflective equipment,roller skaters will greatly reduce their risk of being severely injured in an accident.

Write a response in which you examine the stated and/or unstated assumptions of the

argument.Be sure to explain how the argument depends on these assumptions and what the implications are for the argument if the assumptions prove unwarranted.

医院对那些滑板事故后来医院急救的人的调查显示他们需要更具保护性的设备。在那群人中,75%在大街或停车场发生事故的人没有穿着任何防护衣服(头盔,护膝等等)或任何的反光物质(小手电,或者夜光护腕等等)。很明显,该数据表明投资高质量的防护工具及反光设备,滑板玩家们会大大减少其在事故中受伤的机率。 --------------------------------------------------------------------------------------------------------------------------------------------- 42)

"Erosion of beach sand along the shores of Tria Island is a serious threat to our island and our tourist industry.In order to stop the erosion,we should charge people for using the

beaches.Although this solution may annoy a few tourists in the short term,it will raise money for replenishing the sand.Replenishing the sand,as was done to protect buildings on the nearby island of Batia,will help protect buildings along our shores,thereby reducing these buildings'risk of additional damage from severe storms.And since beaches and buildings in the area will be preserved,Tria's tourist industry will improve over the long term."

Write a response in which you discuss what specific evidence is needed to evaluate the argument and explain how the evidence would weaken or strengthen the argument.

T岛海岸的E沙滩的侵蚀对于我们岛和我们的旅游业是个严重的威胁。为阻止侵蚀,我们应该对使用海滩的人收费。尽管这一解决方案会在短期内触怒少量游客,它将会减少使用海滩的人数并增加补充沙子的资金。象临近的Batia岛一样补充沙子将会有助于对我们沿岸建筑的保护,从而减少这些房屋在大风暴中受损的危险。由于房屋和海滩将会受到保护,所以T岛地区的旅游业将会在长远得到发展。 --------------------------------------------------------------------------------------------------------------------------------------------- 43)

The following appeared in a memorandum written by the chairperson of the West Egg Town Council.

"Two years ago,consultants predicted that West Egg's landfill,which is used for garbage

disposal,would be completely filled within five years.During the past two years,however,the town's residents have been recycling twice as much material as they did in previous years.Next month the amount of recycled material—which includes paper,plastic,and metal—should further

increase,since charges for pickup of other household garbage will double.Furthermore,over 90 percent of the respondents to a recent survey said that they would do more recycling in the future.Because of our town's strong commitment to recycling,the available space in our landfill should last for considerably longer than predicted."

Write a response in which you discuss what specific evidence is needed to evaluate the argument and explain how the evidence would weaken or strengthen the argument.

两年前,我们的顾问预言West Egg用于投放垃圾的填埋地将在五年内完全充满。然而在过去两年间,市对于铝和纸张的循环再生的数量比以前翻了一番。由于垃圾收集的收费在下个月将会加倍,循环再生的材料数量将进一步增加。而且,最近一次调查超过90%的回应者表示他们将会在未来做更多的循环再生工作。由于居对循环再生的有力支持,我们填埋地可利用空间的使用时间将比预期的长得多。

--------------------------------------------------------------------------------------------------------------------------------------------- 44)

The following appeared in a letter to the editor of a journal on environmental issues.

"Over the past year,the Crust Copper Company(CCC)has purchased over 10,000 square miles of land in the tropical nation of West Fredonia.Mining copper on this land will inevitably result in pollution and,since West Fredonia is the home of several endangered animal species,in environmental disaster.But such disasters can be prevented if consumers simply refuse to purchase products that are made with CCC's copper unless the company abandons its mining plans."

Write a response in which you examine the stated and/or unstated assumptions of the

argument.Be sure to explain how the argument depends on these assumptions and what the implications are for the argument if the assumptions prove unwarranted.

去年,CCC公司在热带国家West Fredonia购买了上万平方英里的土地。在这些地方采矿将会不可避免地导致污染和环境灾害,因为West Fredonia是很多濒危物种的栖居地。但如果消费者简单地拒绝购买用CCC所生产的铜而制造的产品,直到CCC放弃它的采矿计划就可以避免这种灾害。

--------------------------------------------------------------------------------------------------------------------------------------------- 45)

The following is part of a memorandum from the president of Humana University.

"Last year the number of students who enrolled in online degree programs offered by nearby Omni University increased by 50 percent.During the same year,Omni showed a significant decrease from prior years in expenditures for dormitory and classroom space,most likely because instruction in the online programs takes place via the Internet.In contrast,over the past three years,enrollment

at Humana University has failed to grow,and the cost of maintaining buildings has increased along with our budget deficit.To address these problems,Humana University will begin immediately to create and actively promote online degree programs like those at Omni.We predict that instituting these online degree programs will help Humana both increase its total enrollment and solve its budget problems."

Write a response in which you discuss what questions would need to be answered in order to decide whether the prediction and the argument on which it is based are reasonable.Be sure to explain how the answers to these questions would help to evaluate the prediction.

去年报名参加附近Omni大学远程教学系统的学生人数上升了50%。同年,Omni用于宿舍和教室的开支比前一年显著下降,很可能是因为远程教学通过互动电脑录像授课。比较而言,在过去三年中,Humana学院的报名人数下降了,而且用于房屋维护的费用和财政赤字都上升。因此,为解决这些问题,我们也应该开展并积极推广与Omni一样的远程教学系统。我们预计推出这样的在线课程不光可以增加我们的招生也同时可以解决目前的财政赤字危机。

49)

The following is part of a memorandum from the president of Humana University.

"Last year the number of students who enrolled in online degree programs offered by nearby Omni University increased by 50 percent.During the same year,Omni showed a significant decrease from prior years in expenditures for dormitory and classroom space,most likely because online instruction takes place via the Internet.In contrast,over the past three years,enrollment at Humana University has failed to grow and the cost of maintaining buildings has increased.Thus,to increase enrollment and solve the problem of budget deficits at Humana University,we should initiate and actively promote online degree programs like those at Omni."

Write a response in which you examine the stated and/or unstated assumptions of the

argument.Be sure to explain how the argument depends on these assumptions and what the implications are for the argument if the assumptions prove unwarranted.

去年报名参加Omni大学远程教学系统的学生人数上升了50%。同年,Omni用于宿舍和教室的开支比前一年显著下降,很可能是因为远程教学通过互动电脑录像授课。比较而言,在过去三年中,Humana学院的报名人数下降了,而且用于房屋维护的费用上升。因此,为增加报名数量并解决Humana学院财政赤字问题,我们也应该开展并积极推广与Omni一样的远程教学系统。

--------------------------------------------------------------------------------------------------------------------------------------------- 46)

The following appeared in a health magazine published in Corpora.

"Medical experts say that only one-quarter of Corpora's citizens meet the current standards for adequate physical fitness,even though twenty years ago,one-half of all of Corpora's citizens met the standards as then defined.But these experts are mistaken when they suggest that spending too much time using computers has caused a decline in fitness.Since overall fitness levels are highest in regions of Corpora where levels of computer ownership are also highest,it is clear that using computers has not made citizens less physically fit.Instead,as shown by this year's unusually low expenditures on fitness-related products and services,the recent decline in the economy is most likely the cause,and fitness levels will improve when the economy does."

Write a response in which you examine the stated and/or unstated assumptions of the

argument.Be sure to explain how the argument depends on these assumptions and what the implications are for the argument if the assumptions prove unwarranted.

医学专家指出C市只有四分之一的市民达到了目前的健康标准,尽管20年前,C的市有一半都达到了那时由国家制定的健康标准。同时专家们认为长时间的使用电脑可能是导致这一现象的原因,但这样的解释可能是错的。由于C市是电脑拥有量最高的地区也是总体健康水平最高的地区,显然使

用电脑并没有导致市民体质的下降。相反,今年用于健身产品和服务的花费低的异乎寻常,这表明最近经济的衰退最可能是原因,当经济复苏了健康水平也会随之提高。 --------------------------------------------------------------------------------------------------------------------------------------------- 47)

The following appeared in a memorandum from the owner of Movies Galore,a chain of

movie-rental stores.

"Because of declining profits,we must reduce operating expenses at Movies Galore's ten movie-rental stores.Raising prices is not a good option,since we are famous for our low prices.Instead,we should reduce our operating hours.Last month our store in downtown Marston reduced its hours by closing at 6:00 p.m.rather than 9:00 p.m.and reduced its overall inventory by no longer stocking any DVD released more than five years ago.Since we have received very few customer complaints about these new policies,we should now adopt them at all other Movies Galore stores as our best strategies for improving profits."

Write a response in which you discuss what specific evidence is needed to evaluate the argument and explain how the evidence would weaken or strengthen the argument.

由于利润下降,我们必须压缩G公司的十家录像租赁店的营业开支。涨价并不是一个好选择,因为我们就是因特价销售而知名的。因此我们应该缩短营业时间。上个月我们位于闹市区Marston的店营业时间从以前的9点缩减到下午6点,并通过不再保留发行5年以上的电影来减少总库存量。由于我们所收到的客户关于这些新政策的投诉很少,我们应该在所有G公司的商店实施这些政策作为增加利润的最好策略。

111)

The following appeared in a memorandum from the owner of Movies Galore,a chain of

movie-rental stores.

"In order to stop the recent decline in our profits,we must reduce operating expenses at Movies Galore's ten movie-rental stores.Since we are famous for our special bargains,raising our rental prices is not a viable way to improve profits.Last month our store in downtown Marston significantly decreased its operating expenses by closing at 6:00 P.M.rather than 9:00 P.M.and by reducing its stock by eliminating all movies released more than five years ago.By implementing similar changes in our other stores,Movies Galore can increase profits without jeopardizing our reputation for offering great movies at low prices."

Write a response in which you examine the stated and/or unstated assumptions of the

argument.Be sure to explain how the argument depends on these assumptions and what the implications are for the argument if the assumptions prove unwarranted.

为了遏制我们的利润下降,我们必须压缩G公司的十家录像租赁店的营业开支。涨价并不是一个好选择去提高利润,因为我们就是因特价销售而知名的。上个月我们位于闹市区Marston的店营业时间从以前的9点缩减到下午6点,并通过不再保留发行5年以上的电影来减少总库存量。我们应该在所有G公司的商店实施这些政策作,这必将会增加M的利润同时也没有损坏我们提供低价高质影片的声誉。

112) The following appeared in a memorandum from the owner of Movies Galore,a chain of

movie-rental stores.

"In order to reverse the recent decline in our profits,we must reduce operating expenses at Movies Galore's ten movie-rental stores.Since we are famous for our special bargains,raising our rental prices is not a viable way to improve profits.Last month our store in downtown Marston

significantly decreased its operating expenses by closing at 6:00 p.m.rather than 9:00 p.m.and by reducing its stock by eliminating all movies released more than five years ago.Therefore,in order

to increase profits without jeopardizing our reputation for offering great movies at low prices,we recommend implementing similar changes in our other nine Movies Galore stores."

Write a response in which you discuss what questions would need to be answered in order to decide whether the recommendation and the argument on which it is based are reasonable.Be sure to explain how the answers to these questions would help to evaluate the recommendation.

为了遏制我们的利润下降,我们必须压缩G公司的十家录像租赁店的营业开支。涨价并不是一个好选择去提高利润,因为我们就是因特价销售而知名的。上个月我们位于闹市区Marston的店营业时间从以前的9点缩减到下午6点,并通过不再保留发行5年以上的电影来减少总库存量。为了能增加M的利润同时不损坏我们提供低价高质影片的声誉,我们应该在G公司的其他9家商店实施这些政策作。

--------------------------------------------------------------------------------------------------------------------------------------------- 48)

The following appeared in a magazine article about planning for retirement.

"Clearview should be a top choice for anyone seeking a place to retire,because it has spectacular natural beauty and a consistent climate.Another advantage is that housing costs in Clearview have fallen significantly during the past year,and taxes remain lower than those in neighboring towns.Moreover,Clearview's mayor promises many new programs to improve schools,streets,and public services.And best of all,retirees in Clearview can also expect excellent health care as they grow older,since the number of physicians in the area is far greater than the national average."

Write a response in which you discuss what specific evidence is needed to evaluate the argument and explain how the evidence would weaken or strengthen the argument.

由于Clearview的天然景色和温和气候,它应该成为那些为退休后寻找生活地的人的首选。另一项好处是,Clearview的房价在去年显著下降,房地产税一直比邻近城市低。而且,Clearview的市长承诺了很多新方案来改学校、街道和公共服务。Clearview的退休人员也可以在老年时享受到出色的医疗服务,因为该地区的医生数量高于全国平均水平。

--------------------------------------------------------------------------------------------------------------------------------------------- 50)

An ancient,traditional remedy for insomnia—the scent of lavender flowers—has now been proved effective.In a recent study,30 volunteers with chronic insomnia slept each night for three weeks on lavender-scented pillows in a controlled room where their sleep was monitored

electronically.During the first week,volunteers continued to take their usual sleeping

medication.They slept soundly but wakened feeling tired.At the beginning of the second week,the volunteers discontinued their sleeping medication.During that week,they slept less soundly than the previous week and felt even more tired.During the third week,the volunteers slept longer and more soundly than in the previous two weeks.Therefore,the study proves that lavender cures insomnia within a short period of time.

Write a response in which you discuss what specific evidence is needed to evaluate the argument and explain how the evidence would weaken or strengthen the argument.

一种古老的传统治疗失眠的偏方--薰衣草花香,现在被证明是有效的。在一次最近的调查中,30名患有慢性失眠的志愿者在三周之内每晚都在一个受监视的控制室内睡在带薰衣草花香的枕头上。在第一周,志愿者继续服用他们常用的安眠药。他们睡得很沉但醒来时很累。在第二周,他们不服用药物。结果与前一周相比他们睡得不那么沉并且感觉更累。在第三周,他们睡得比前两周时间长而且更深。这表明薰衣草在短时间内治愈了失眠。

---------------------------------------------------------------------------------------------------------------------------------------------

53)

The following appeared in a health magazine.

"The citizens of Forsythe have adopted more healthful lifestyles.Their responses to a recent survey show that in their eating habits they conform more closely to government nutritional recommendations than they did ten years ago.Furthermore,there has been a fourfold increase in sales of food products containing kiran,a substance that a scientific study has shown reduces cholesterol.This trend is also evident in reduced sales of sulia,a food that few of the most healthy citizens regularly eat."

Write a response in which you discuss what specific evidence is needed to evaluate the argument and explain how the evidence would weaken or strengthen the argument.

Forsythe的居选择了更健康的生活方式。他们对于最近一项调查的回答显示,他们的饮食习惯比十年前更加贴近政府的营养建议。而且,含有kiran的食品销量增长了四倍,在一次科学研究中发现kiran是一种能够降低胆固醇水平的物质。这种趋势同样也被sulia的销量下降所证实,sulia是那些最健康的居极少经常食用的食品 144)

The citizens of Forsythe have adopted more healthful lifestyles.Their responses to a recent survey show that in their eating habits they conform more closely to government nutritional

recommendations than they did ten years ago.Furthermore,there has been a fourfold increase in sales of food products containing kiran,a substance that a scientific study has shown reduces cholesterol.This trend is also evident in reduced sales of sulia,a food that few of the healthiest citizens regularly eat.

Write a response in which you examine the stated and/or unstated assumptions of the

argument.Be sure to explain how the argument depends on these assumptions and what the implications are for the argument if the assumptions prove unwarranted.

Forsythe的居选择了更健康的生活方式。他们对于最近一项调查的回答显示,他们的饮食习惯比十年前更加贴近政府的营养建议。而且,含有kiran的食品销量增长了四倍,在一次科学研究中发现kiran是一种能够降低胆固醇水平的物质。这种趋势同样也被sulia的销量下降所证实,sulia是那些最健康的居极少经常食用的食品。 151)

Benton City residents have adopted healthier lifestyles.A recent survey of city residents shows that the eating habits of city residents conform more closely to government nutritional

recommendations than they did ten years ago.During those ten years,local sales of food products containing kiran,a substance that a scientific study has shown reduces cholesterol,have increased fourfold,while sales of sulia,a food rarely eaten by the healthiest residents,have declined

dramatically.Because of these positive changes in the eating habits of Benton City residents,we predict that the obesity rate in the city will soon be well below the national average.

Write a response in which you discuss what questions would need to be answered in order to decide whether the prediction and the argument on which it is based are reasonable.Be sure to explain how the answers to these questions would help to evaluate the prediction.

B市的居民选择了更健康的生活方式。他们对于最近一项调查的回答显示,他们的饮食习惯比十年前更加贴近政府的营养建议。而且,含有kiran的食品销量增长了四倍,在一次科学研究中发现kiran是一种能够降低胆固醇水平的物质。这种趋势同样也被sulia的销量下降所证实,sulia是那些最健康的居极少经常食用的食品。由于这些B市居民饮食习惯的积极改变,我们预计该市的肥胖率将很快低于全国平均水平。

--------------------------------------------------------------------------------------------------------------------------------------------- 54)

Humans arrived in the Kaliko Islands about 7,000 years ago,and within 3,000 years most of the large mammal species that had lived in the forests of the Kaliko Islands had become extinct.Yet humans cannot have been a factor in the species'extinctions,because there is no evidence that

the humans had any significant contact with the mammals.Further,archaeologists have discovered numerous sites where the bones of fish had been discarded,but they found no such areas containing the bones of large mammals,so the humans cannot have hunted the

mammals.Therefore,some climate change or other environmental factor must have caused the species'extinctions.

Write a response in which you examine the stated and/or unstated assumptions of the

argument.Be sure to explain how the argument depends on these assumptions and what the implications are for the argument if the assumptions prove unwarranted.

大约7000年前人类到达了Kaliko岛,在3000年内曾经生活在Kaliko岛的树林中的大型哺乳动物绝大多数已经灭绝了。然而人类并不是导致这些物种灭绝的因素,因为没有证据表明人类与这些哺乳动物有很多接触。而且,考古学家发现一些有大量鱼骨被抛弃的场所,而他们并没有发现存在大型哺乳动物骨头的类似场所,因而人类并没有猎杀这些哺乳动物。因此,一定是一些气候上的变化或其他环境因素导致了这些物种的灭绝。 165)

Humans arrived in the Kaliko Islands about 7,000 years ago,and within 3,000 years most of the large mammal species that had lived in the forests of the Kaliko Islands were extinct.Previous archaeological findings have suggested that early humans generally relied on both fishing and hunting for food;since archaeologists have discovered numerous sites in the Kaliko Islands where the bones of fish were discarded,it is likely that the humans also hunted the

mammals.Furthermore,researchers have uncovered simple tools,such as stone knives,that could be used for hunting.The only clear explanation is that humans caused the extinction of the various mammal species through excessive hunting.

Write a response in which you discuss one or more alternative explanations that could rival the proposed explanation and explain how your explanation(s)can plausibly account for the facts presented in the argument.

大约7000年前人类到达了Kaliko岛,在3000年内曾经生活在Kaliko岛的树林中的大型哺乳动物绝大多数已经灭绝了。早期的人类学家发现,早期人类主要以鱼类和打猎为生。然而考古学家发现一些有大量鱼骨被抛弃的场所,很有可能人类也扑杀哺乳动物。另外,研究者也发现了一些简单的捕猎工具,例如石刀。显然,哺乳动物灭绝现象的唯一解释就是人类的过度捕杀。

--------------------------------------------------------------------------------------------------------------------------------------------- 55)

The following appeared in an editorial in a business magazine.

"Although the sales of Whirlwind video games have declined over the past two years,a recent survey of video-game players suggests that this sales trend is about to be reversed.The survey asked video-game players what features they thought were most important in a video game.According to the survey,players prefer games that provide lifelike graphics,which require the most up-to-date computers.Whirlwind has just introduced several such games with an extensive advertising campaign directed at people ten to twenty-five years old,the age-group most likely to play video games.It follows,then,that the sales of Whirlwind video games are likely to increase dramatically in the next few months."

Write a response in which you examine the stated and/or unstated assumptions of the

argument.Be sure to explain how the argument depends on these assumptions and what the implications are for the argument if the assumptions prove unwarranted.

尽管Whirlwind游戏公司的游戏销量在过去两年中下降了,最近一次对于电子游戏玩家的调查显示这种销售趋势可能会逆转。该调查询问游戏玩家对于一个游戏来说他们所认为的最重要特征是什么。根据调查结果,玩家倾向于那些需要最先进的电脑的,具有活灵活现图象的游戏。Whirlwind刚刚面

向10-25岁的人群开展了大力的广告活动来推广若干此类游戏,10-25岁是最喜欢玩游戏的年龄层。这说明Whirlwind公司的游戏销量将会在未来几个月猛增。 --------------------------------------------------------------------------------------------------------------------------------------------- 56)

The following appeared in a memo from the vice president of marketing at Dura-Sock,Inc.

"A recent study of our customers suggests that our company is wasting the money it spends on its patented Endure manufacturing process,which ensures that our socks are strong enough to last for two years.We have always advertised our use of the Endure process,but the new study shows that despite our socks'durability,our average customer actually purchases new Dura-Socks every three months.Furthermore,our customers surveyed in our largest market,northeastern United States cities,say that they most value Dura-Socks'stylish appearance and availability in many colors.These findings suggest that we can increase our profits by discontinuing use of the Endure manufacturing process."

Write a response in which you examine the stated and/or unstated assumptions of the

argument.Be sure to explain how the argument depends on these assumptions and what the implications are for the argument if the assumptions prove unwarranted.

最近一次对于Dura-Sock使用者的调查表明我们公司用于其专利生产方式"Endure"上的钱是浪费的,这种方式使我们生产的袜子足以使用两年。Dura-Sock一直在做广告宣传它使用"Endure"方式,但这次新调查显示尽管我们的袜子十分耐穿,但Dura-Sock的消费者每三个月就买一双Dura-Sock袜子。而且,在位于美国北部城市我们最大的商场回应调查的Dura-Sock消费者说他们最欣赏Dura-Sock时尚的外观和众多颜色的选择。这些事实说明Dura-Sock可以通过停止使用"Endure"生产方式来增加盈利。

57)

The following appeared in a memo from the vice president of marketing at Dura-Sock,Inc.

"A recent study of our customers suggests that our company is wasting the money it spends on its patented Endure manufacturing process,which ensures that our socks are strong enough to last for two years.We have always advertised our use of the Endure process,but the new study shows that despite our socks'durability,our average customer actually purchases new Dura-Socks every three months.Furthermore,our customers surveyed in our largest market,northeastern United States cities,say that they most value Dura-Socks'stylish appearance and availability in many colors.These findings suggest that we can increase our profits by discontinuing use of the Endure manufacturing process."

Write a response in which you discuss what specific evidence is needed to evaluate the argument and explain how the evidence would weaken or strengthen the argument.

最近一次对于Dura-Sock使用者的调查表明我们公司用于其专利生产方式"Endure"上的钱是浪费的,这种方式使我们生产的袜子足以使用两年。Dura-Sock一直在做广告宣传它使用"Endure"方式,但这次新调查显示尽管我们的袜子十分耐穿,Dura-Sock的消费者每三个月就买一双Dura-Sock袜子。而且,在位于美国北部城市我们最大的商场回应调查的Dura-Sock消费者说他们最欣赏Dura-Sock时尚的外观和众多颜色的选择。这些事实说明Dura-Sock可以通过停止使用"Endure"生产方式来增加盈利。 82)

The following appeared in a memo from the vice president of marketing at Dura-Socks,Inc.

"A recent study of Dura-Socks customers suggests that our company is wasting the money it spends on its patented Endure manufacturing process,which ensures that our socks are strong enough to last for two years.We have always advertised our use of the Endure process,but the new study shows that despite the socks'durability,our customers,on average,actually purchase new Dura-Socks every three months.Furthermore,customers surveyed in our largest

market—northeastern United States cities—say that they most value Dura-Socks'stylish

appearance and availability in many colors.These findings suggest that we can increase our profits by discontinuing use of the Endure manufacturing process."

Write a response in which you discuss what questions would need to be answered in order to decide whether the recommendation and the argument on which it is based are reasonable.Be sure to explain how the answers to these questions would help to evaluate the recommendation.

最近一次对于Dura-Sock使用者的调查表明我们公司用于其专利生产方式"Endure"上的钱是浪费的,这种方式使我们生产的袜子足以使用两年。Dura-Sock一直在做广告宣传它使用"Endure"方式,但这次新调查显示尽管我们的袜子十分耐穿,但Dura-Sock的消费者每三个月就买一双Dura-Sock袜子。而且,在位于美国北部城市我们最大的商场回应调查的Dura-Sock消费者说他们最欣赏Dura-Sock时尚的外观和众多颜色的选择。这些事实说明Dura-Sock可以通过停止使用"Endure"生产方式来增加盈利。

---------------------------------------------------------------------------------------------------------------------------------------------

59)

The following appeared in a memo from the president of Bower Builders,a company that constructs new homes.

"A nationwide survey reveals that the two most-desired home features are a large family room and a large,well-appointed kitchen.A number of homes in our area built by our competitor Domus

Construction have such features and have sold much faster and at significantly higher prices than the national average.To boost sales and profits,we should increase the size of the family rooms and kitchens in all the homes we build and should make state-of-the-art kitchens a standard feature.Moreover,our larger family rooms and kitchens can come at the expense of the dining room,since many of our recent buyers say they do not need a separate dining room for family meals."

Write a response in which you examine the stated and/or unstated assumptions of the

argument.Be sure to explain how the argument depends on these assumptions and what the implications are for the argument if the assumptions prove unwarranted.

一项全国性调查发现人们最想拥有的两项家庭设施就是具备冲浪浴缸的浴室和大厨房。我们的竞争对手Domus Construction在附近开发的新住宅安装了冲浪浴缸,这些住宅的销售速度和售价显著高于平均水平。为增加我们的销售和利润,我们应该在所有新住宅中设置冲浪浴缸和大型厨房作为标准配置。由于我们的新客户没有提出对小型庭院的任何投诉,我们也可以通过减小庭院的尺寸来增加盈利。

--------------------------------------------------------------------------------------------------------------------------------------------- 60) The following appeared in a letter from a firm providing investment advice for a client.

"Most homes in the northeastern United States,where winters are typically cold,have traditionally used oil as their major fuel for heating.Last heating season that region experienced 90 days with below-normal temperatures,and climate forecasters predict that this weather pattern will continue for several more years.Furthermore,many new homes are being built in the region in response to recent population growth.Because of these trends,we predict an increased demand for heating oil and recommend investment in Consolidated Industries,one of whose major business operations is the retail sale of home heating oil."

Write a response in which you examine the stated and/or unstated assumptions of the

argument.Be sure to explain how the argument depends on these assumptions and what the implications are for the argument if the assumptions prove unwarranted.

在冬季非常寒冷的美国东北部的房屋,一直都使用传统的油料作为采暖的主要燃料。去年该地区经

历了90天低于往年平均气温的严寒,并且气象预测家们预测这种天气会持续未来数年。而且,去年由于人口增长,本地建造了很多的房屋。按此趋势,我们预测对于油料的需求将会增加,同时也建议对CI公司投资,该公司的不要业务之一就是家用采暖油料的零售。

1) 今后几年不一定用油作为主要燃料,新建的房屋也不一定使用油做燃料

2) 天气预报不一定准,可能会越来越热

3) below-normal temperatures没有具体说明低了多少,如果低的很少的话完全可以不用考虑

4) 没有证据表明去年温度降低后油料的使用量增加了

5) 去年人口增加造了很多房屋,之后人口可能减少

6) 即使油料需求增加也不一定投资ci公司,应该投资规模较大的公司,而不是“零售”型公司 7) 人口的增长使油料需求增加了多少,若果太少的话是否还值得投资

145)

The following appeared in a memo to the board of directors of a company that specializes in the delivery of heating oil.

"Most homes in the northeastern United States,where winters are typically cold,have traditionally used oil as their major fuel for heating.Last heating season,that region experienced 90 days with below-normal temperatures,and climate forecasters predict that this weather pattern will continue for several more years.Furthermore,many new homes are being built in the region in response to recent population growth.Because of these trends,we can safely predict that this region will experience an increased demand for heating oil during the next five years."

Write a response in which you discuss what questions would need to be answered in order to decide whether the prediction and the argument on which it is based are reasonable.Be sure to explain how the answers to these questions would help to evaluate the prediction.

在冬季非常寒冷的美国东北部的房屋,一直都使用传统的油料作为采暖的主要燃料。去年该地区经历了90天低于往年平均气温的严寒,并且气象预测家们预测这种天气会持续未来数年。而且,去年由于人口增长,本地建造了很多的房屋。按此趋势,我们预测在未来五年,本地对于取暖油料的需求将会增加.

146)

The following appeared in a memo to the board of directors of a company that specializes in the delivery of heating oil.

"Most homes in the northeastern United States,where winters are typically cold,have traditionally used oil as their major fuel for heating.Last heating season,that region experienced 90 days with below-normal temperatures,and climate forecasters predict that this weather pattern will continue for several more years.Furthermore,many new homes are being built in the region in response to recent population growth.Because of these trends,we can safely predict that this region will experience an increased demand for heating oil during the next five years."

Write a response in which you discuss what specific evidence is needed to evaluate the argument and explain how the evidence would weaken or strengthen the argument.

在冬季非常寒冷的美国东北部的房屋,一直都使用传统的油料作为采暖的主要燃料。去年该地区经历了90天低于往年平均气温的严寒,并且气象预测家们预测这种天气会持续未来数年。而且,去年由于人口增长,本地建造了很多的房屋。按此趋势,我们预测在未来五年,本地对于取暖油料的需求将会增加.

150)

The following appeared in a letter from a firm providing investment advice to a client.

"Homes in the northeastern United States,where winters are typically cold,have traditionally used oil as their major fuel for heating.Last year that region experienced 90 days with below-average

temperatures,and climate forecasters at Waymarsh University predict that this weather pattern will continue for several more years.Furthermore,many new homes have been built in this region during the past year.Because these developments will certainly result in an increased demand for heating oil,we recommend investment in Consolidated Industries,one of whose major business operations is the retail sale of home heating oil."

Write a response in which you discuss what questions would need to be answered in order to decide whether the recommendation and the argument on which it is based are reasonable.Be sure to explain how the answers to these questions would help to evaluate the recommendation.

在冬季非常寒冷的美国东北部的房屋,一直都使用传统的油料作为采暖的主要燃料。去年该地区经历了90天低于往年平均气温的严寒,并且气象预测家们预测这种天气会持续未来数年。而且,去年由于人口增长,本地建造了很多的房屋。按此趋势,我们预测对于油料的需求将会增加,同时也建议对CI公司投资,该公司的不要业务之一就是家用采暖油料的零售。

154)

The following appeared in a letter from a firm providing investment advice to a client.

"Homes in the northeastern United States,where winters are typically cold,have traditionally used oil as their major fuel for heating.Last year that region experienced twenty days with

below-average temperatures,and local weather forecasters throughout the region predict that this weather pattern will continue for several more years.Furthermore,many new homes have been built in this region during the past year.Based on these developments,we predict a large increase in the demand for heating oil.Therefore,we recommend investment in Consolidated Industries,one of whose major business operations is the retail sale of home heating oil."

Write a response in which you discuss what questions would need to be answered in order to decide whether the recommendation and the argument on which it is based are reasonable.Be sure to explain how the answers to these questions would help to evaluate the recommendation.

在冬季非常寒冷的美国东北部的房屋,一直都使用传统的油料作为采暖的主要燃料。去年该地区经历了90天低于往年平均气温的严寒,并且气象预测家们预测这种天气会持续未来数年。而且,去年本地建造了很多的房屋。按此趋势,我们预测对于油料的需求将会增加.所以我们建议对CI公司投资,该公司的主要业务之一就是家用采暖油料的零售。

155)

The following appeared in a letter from a firm providing investment advice to a client.

"Homes in the northeastern United States,where winters are typically cold,have traditionally used oil as their major fuel for heating.Last year that region experienced twenty days with below-average temperatures,and local weather forecasters throughout the region predict that this weather pattern will continue for several more years.Furthermore,many new homes have been built in this region during the past year.Because of these developments,we predict an increased demand for heating oil and recommend investment in Consolidated Industries,one of whose major business operations is the retail sale of home heating oil."

Write a response in which you discuss what specific evidence is needed to evaluate the argument and explain how the evidence would weaken or strengthen the argument.

在冬季非常寒冷的美国东北部的房屋,一直都使用传统的油料作为采暖的主要燃料。去年该地区经历了90天低于往年平均气温的严寒,并且气象预测家们预测这种天气会持续未来数年。而且,去年本地建造了很多的房屋。按此趋势,我们预测对于油料的需求将会增加.所以我们建议对CI公司投资,该公司的主要业务之一就是家用采暖油料的零售。

--------------------------------------------------------------------------------------------------------------------------------------------- 61)

The following appeared in an article in the Grandview Beacon.

"For many years the city of Grandview has provided annual funding for the Grandview

Symphony.Last year,however,private contributions to the symphony increased by 200 percent and attendance at the symphony's concerts-in-the-park series doubled.The symphony has also announced an increase in ticket prices for next year.Given such developments,some city

commissioners argue that the symphony can now be fully self-supporting,and they recommend that funding for the symphony be eliminated from next year's budget."

Write a response in which you discuss what questions would need to be answered in order to decide whether the recommendation and the argument on which it is based are reasonable.Be sure to explain how the answers to these questions would help to evaluate the recommendation.

这么多年来G市政府每年都资助G的交响乐团。但是去年,个人对乐团的资助增加了200%,同时乐团公园音乐会的人数翻了一番。另外乐团还宣布明年的票价将会提高。鉴于目前局面,一些G市的官员们认为G的交响乐团可以独立维持生存,可以取消明年对该乐团的政府资助。

1)200%的基数不知道,太少的话即使翻倍也没意义

2)doubled同1)

3)票价提高会导致听众数量的减少,总收入不一定增加

4)只有last year一年的数据不足以说明问题,很可能是个特例

5)去年的收入具体增加了多少?增加的部分是否可以抵消政府的资助?

6)即使乐队可以自我维持,政府的资助还可以用来扩大规模、更新设备或者增加宣传等等 139)

The following appeared in a memo from a budget planner for the city of Grandview.

"Our citizens are well aware of the fact that while the Grandview Symphony Orchestra was struggling to succeed,our city government promised annual funding to help support its

programs.Last year,however,private contributions to the symphony increased by 200 percent,and attendance at the symphony's concerts-in-the-park series doubled.The symphony has also announced an increase in ticket prices for next year.Such developments indicate that the symphony can now succeed without funding from city government and we can eliminate that

expense from next year's budget.Therefore,we recommend that the city of Grandview eliminate its funding for the Grandview Symphony from next year's budget.By doing so,we can prevent a city budget deficit without threatening the success of the symphony."

Write a response in which you discuss what questions would need to be answered in order to decide whether the recommendation is likely to have the predicted result.Be sure to explain how the answers to these questions would help to evaluate the recommendation.

我们的居民意识到就在G管弦乐队为其维持做斗争的时候,我们市政府承诺每年提供资助来支持乐队的计划。

然而去年,对乐队的个人资助增加了200%,乐队的公园音乐会的听众人数翻了一番。乐队还宣布明年的票价将会提高。这些事实说明G乐队可以不用市政府的资助就能维持。因此我们建议G市政府取消来年预算中对G乐团的支出。这一行动肯定能够防止财政赤字同时也不会影响G乐团的成功。 141)

The following appeared in a memo to the board of the Grandview Symphony.

"The city of Grandview has provided annual funding for the Grandview Symphony since the symphony's inception ten years ago.Last year the symphony hired an internationally known conductor,who has been able to attract high-profile guest musicians to perform with the

symphony.Since then,private contributions to the symphony have doubled and attendance at the symphony's concerts-in-the-park series has reached new highs.Now that the Grandview Symphony is an established success,it can raise ticket prices.Increased revenue from larger

audiences and higher ticket prices will enable the symphony to succeed without funding from the city government."

Write a response in which you discuss what specific evidence is needed to evaluate the argument and explain how the evidence would weaken or strengthen the argument.

G市政府自打10年前G乐团成立以来每年都给予其资助。去年G交响乐团聘用了一个可以吸引一些高水平的客座音乐家来助阵演出的国际知名指挥家。至此后,对乐队的个人资助增加了200%,乐队的公园音乐会的听众人数也创了新高。现在乐队已经确立了其成功地为,明年的票价将会提高。大批观众的票务收入以及更贵的价格将会让G乐团在没有G市政府资助下依然会取得成功。 143)

The following appeared in a memo from a budget planner for the city of Grandview.

"When the Grandview Symphony was established ten years ago,the city of Grandview agreed to provide the symphony with annual funding until the symphony became self-sustaining.Two years ago,the symphony hired an internationally known conductor,who has been able to attract high-profile guest musicians to perform with the symphony.Since then,private contributions to the symphony have tripled and attendance at the symphony's outdoor summer concert series has reached record highs.Now that the symphony has succeeded in finding an audience,the city can eliminate its funding of the symphony."

Write a response in which you examine the stated and/or unstated assumptions of the

argument.Be sure to explain how the argument depends on these assumptions and what the implications are for the argument if the assumptions prove unwarranted.

自打10年前G乐团成立以来,G市政府就同意每年都给予其资助直至其能自我维持为止。2年前,G交响乐团聘用了一个可以吸引一些高水平的客座音乐家来助阵演出的国际知名指挥家。至此后,对乐队的个人资助增加了200%,乐队的夏季户外音乐会的听众人数也创了新高。既然G乐团现在可以成功的吸引如此大量观众,那么G市可以停止对G乐团的资助。

162)

The following appeared in a memo from a budget planner for the city of Grandview.

"It is time for the city of Grandview to stop funding the Grandview Symphony Orchestra.It is true that the symphony struggled financially for many years,but last year private contributions to the symphony increased by 200 percent and attendance at the symphony's concerts-in-the-park series doubled.In addition,the symphony has just announced an increase in ticket prices for next year.For these reasons,we recommend that the city eliminate funding for the Grandview

Symphony Orchestra from next year's budget.We predict that the symphony will flourish in the years to come even without funding from the city."

Write a response in which you discuss what questions would need to be answered in order to decide whether the recommendation is likely to have the predicted result.Be sure to explain how the answers to these questions would help to evaluate the recommendation.

G市政府应该停止对G交响乐团的资助了。事实上G乐团已经挣扎了多年,但去年对乐队的个人资助增加了200%,乐队的公园音乐会的听众人数也翻了一番。另外,乐队也对外宣布明年的票价将会提高。因此,我们建议G市应该停止从明年的预算中继续对G乐团的拨款。我们预计G乐团在没有G市政府资助下依然会取得成功。

--------------------------------------------------------------------------------------------------------------------------------------------- 62)

The following appeared in a memo from the director of a large group of hospitals.

"In a laboratory study of liquid antibacterial hand soaps,a concentrated solution of UltraClean produced a 40 percent greater reduction in the bacteria population than did the liquid hand soaps

currently used in our hospitals.During a subsequent test of UltraClean at our hospital in

Workby,that hospital reported significantly fewer cases of patient infection than did any of the other hospitals in our group.Therefore,to prevent serious patient infections,we should supply UltraClean at all hand-washing stations throughout our hospital system."

Write a response in which you examine the stated and/or unstated assumptions of the

argument.Be sure to explain how the argument depends on these assumptions and what the implications are for the argument if the assumptions prove unwarranted.

再一次对于抗菌洗手液的实验研究中,UC浓缩液比我们医院目前使用的洗手液多杀40%的细菌。我们对在W的医院做的随后测试中发现,该医院上报的患者感染数量明显少于我们集团的其他医院。因此,为防止严重的患者感染,我们应该在我们的医院系统内的所有洗手站提供UC洗手液。

1. 实验室研究不代表现实。比例不代表数量

2. 由个体推知一般

3. Infection还可能由其他的很多事情导致。

4. 信息不完整:实验室的结果不证明在现实使用中效果一样显著,因为UC有可能受到现实环境中其它化学和物理因素的影响,而使其失效

5. 横向比较与纵向比较混淆:作者说我们医院比其他医院感染的少,但是与不使用UC的时候比较呢?如果不使用UC的时候我们医院的感染的也比其他医院要少,就不能证明是UC的原因。

6. 信息不完整:UC杀死的细菌是否能够引起严重的感染?如果UC杀死的仅仅是较弱的细菌,不能够组织严重感染。

121)

The following appeared in a memo from the director of a large group of hospitals.

"In a controlled laboratory study of liquid hand soaps,a concentrated solution of extra strength UltraClean hand soap produced a 40 percent greater reduction in harmful bacteria than did the liquid hand soaps currently used in our hospitals.During our recent test of regular-strength UltraClean with doctors,nurses,and visitors at our hospital in Worktown,the hospital reported significantly fewer cases of patient infection(a 20 percent reduction)than did any of the other

hospitals in our group.Therefore,to prevent serious patient infections,we should supply UltraClean at all hand-washing stations,including those used by visitors,throughout our hospital system."

Write a response in which you examine the stated and/or unstated assumptions of the

argument.Be sure to explain how the argument depends on these assumptions and what the implications are for the argument if the assumptions prove unwarranted.

再一次对于抗菌洗手液的实验研究中,超强UC浓缩液比我们医院目前使用的洗手液多杀40%的有害细菌。我们对在W的医院的使用常规UC洗手液的医生,护士以及访客做的随后测试中发现,该医院上报的患者感染数量(低20%)明显少于我们集团的其他医院。因此,为防止严重的患者感染,我们应该在我们的医院系统内的所有洗手站提供UC洗手液。

122)

The following appeared in a memo from the director of a large group of hospitals.

"In a controlled laboratory study of liquid hand soaps,a concentrated solution of extra strength UltraClean hand soap produced a 40 percent greater reduction in harmful bacteria than did the liquid hand soaps currently used in our hospitals.During our recent test of regular-strength UltraClean with doctors,nurses,and visitors at our hospital in Worktown,the hospital reported significantly fewer cases of patient infection(a 20 percent reduction)than did any of the other

hospitals in our group.The explanation for the 20 percent reduction in patient infections is the use of UltraClean soap."

Write a response in which you discuss one or more alternative explanations that could rival the proposed explanation and explain how your explanation(s)can plausibly account for the facts presented in the argument.

在一次对于抗菌洗手液的实验研究中,超强UC浓缩液比我们医院目前使用的洗手液多杀40%的有害细菌。我们对在W的医院的使用常规UC洗手液的医生,护士以及访客做的随后测试中发现,该医院上报的患者感染数量(低20%)明显少于我们集团的其他医院。很明显这减少的20%的患者是由于使用了UC洗手液。

124)

The following appeared in a memo from the director of a large group of hospitals.

"In a controlled laboratory study of liquid hand soaps,a concentrated solution of extra strength UltraClean hand soap produced a 40 percent greater reduction in harmful bacteria than did the liquid hand soaps currently used in our hospitals.During our recent test of regular-strength UltraClean with doctors,nurses,and visitors at our hospital in Worktown,the hospital reported significantly fewer cases of patient infection(a 20 percent reduction)than did any of the other

hospitals in our group.Therefore,to prevent serious patient infections,we should supply UltraClean at all hand-washing stations,including those used by visitors,throughout our hospital system."

Write a response in which you discuss what specific evidence is needed to evaluate the argument and explain how the evidence would weaken or strengthen the argument.

再一次对于抗菌洗手液的实验研究中,超强UC浓缩液比我们医院目前使用的洗手液多杀40%的有害细菌。我们对在W的医院的使用常规UC洗手液的医生,护士以及访客做的随后测试中发现,该医院上报的患者感染数量(低20%)明显少于我们集团的其他医院。因此,为了防止严重患者感染,我们应该在我们所有的医院的洗手站,包括访客用的洗手站提供UC洗手液。

--------------------------------------------------------------------------------------------------------------------------------------------- 63)

The following appeared in a letter to the editor of the Parkville Daily newspaper.

"Throughout the country last year,as more and more children below the age of nine participated in youth-league sports,over 40,000 of these young players suffered injuries.When interviewed for a recent study,youth-league soccer players in several major cities also reported psychological pressure exerted by coaches and parents to win games.Furthermore,education experts say that long practice sessions for these sports take away time that could be used for academic

activities.Since the disadvantages outweigh any advantages,we in Parkville should discontinue organized athletic competition for children under nine."

Write a response in which you examine the stated and/or unstated assumptions of the

argument.Be sure to explain how the argument depends on these assumptions and what the implications are for the argument if the assumptions prove unwarranted.

去年在全国范围内,随着越来越多的9岁以下儿童参加少年体育比赛,有超过40000的少年选手受伤。在最近一次研究所作的采访中,很多大城市的少年足球选手报告说存在来自教练和家长要求赢得比赛的心理压力。而且,教育专家指出这些运动项目长时间的训练占据了本应用于学习的时间。由于不利因素明显超过了有利因素,我们Parkville应该停止组织9岁以下儿童的体育比赛。 --------------------------------------------------------------------------------------------------------------------------------------------- 64)

Collectors prize the ancient life-size clay statues of human figures made on Kali Island but have long wondered how Kalinese artists were able to depict bodies with such realistic precision.Since archaeologists have recently discovered molds of human heads and hands on Kali,we can now conclude that the ancient Kalinese artists used molds of actual bodies,not sculpting tools and techniques,to create these statues.This discovery explains why Kalinese miniature statues were abstract and entirely different in style:molds could be used only for life-size sculptures.It also

explains why few ancient Kalinese sculpting tools have been found.In light of this

discovery,collectors predict that the life-size sculptures will decrease in value while the miniatures increase in value.

Write a response in which you discuss what questions would need to be answered in order to decide whether the prediction and the argument on which it is based are reasonable.Be sure to explain how the answers to these questions would help to evaluate the prediction.

收藏家很欣赏Kali岛出产的古代等身泥土雕像,但长期以来不清楚Kali的艺术家如何能够以如此高的精确度刻画人体。由于考古学家最近在Kali发现了人类头部和手的模具,我们现在可以得出结论:古代Kali艺术家使用真人的模子,而不是雕刻工具和技艺来塑造这些雕像。这一发现解释了为什么Kali的缩微雕像是抽象的而且风格迥异:模子只能被用于等身雕像。它同样也解释了为什么很少发现Kali的雕刻工具。由于有了这一研究进展,收藏家应预见到等身雕像的贬值以及缩微雕像的升值。 --------------------------------------------------------------------------------------------------------------------------------------------- 65)

When Stanley Park first opened,it was the largest,most heavily used public park in town.It is still the largest park,but it is no longer heavily used.Video cameras mounted in the park's parking lots last month revealed the park's drop in popularity:the recordings showed an average of only 50 cars per day.In contrast,tiny Carlton Park in the heart of the business district is visited by more than 150 people on a typical weekday.An obvious difference is that Carlton Park,unlike Stanley Park,provides ample seating.Thus,if Stanley Park is ever to be as popular with our citizens as Carlton Park,the town will obviously need to provide more benches,thereby converting some of the unused open areas into spaces suitable for socializing.

Write a response in which you examine the stated and/or unstated assumptions of the

argument.Be sure to explain how the argument depends on these assumptions and what the implications are for the argument if the assumptions prove unwarranted.

当Stanley公园第一次开放的时候,它是本市最大,使用频率最高的公园。现在它仍是最大的,但使用频率已经不再高了。上个月在公园停车场架设的摄像机发现公园的受欢迎度下降了:录像显示平均每天只有50辆车。与之相比,位于商业区中心地带的Carlton小公园在工作日每天游客超过150人。一个很明显的差异就是Carlton公园与Stanley公园不同,它提供充足的座椅。因此,如果Stanley公园想要和Carlton获得同等的受欢迎度的话,显然应该提供更多的长椅,从而把一些未加利用的开阔地转化成适于人们交往的空间。

--------------------------------------------------------------------------------------------------------------------------------------------- 66)

The following appeared in a memo from the owner of a chain of cheese stores located throughout the United States.

"For many years all the stores in our chain have stocked a wide variety of both domestic and

imported cheeses.Last year,however,all of the five best-selling cheeses at our newest store were domestic cheddar cheeses from Wisconsin.Furthermore,a recent survey by Cheeses of the World magazine indicates an increasing preference for domestic cheeses among its

subscribers.Since our company can reduce expenses by limiting inventory,the best way to improve profits in all of our stores is to discontinue stocking many of our varieties of imported cheese and concentrate primarily on domestic cheeses."

Write a response in which you discuss what questions would need to be answered in order to decide whether the recommendation is likely to have the predicted result.Be sure to explain how the answers to these questions would help to evaluate the recommendation.

很多年来我们的所有连锁店都储备了很多种类的国产奶酪和进口奶酪。然而去年,我们的最新店里五种销量最高的奶酪都是威斯康星出产的cheddar奶酪。而且,最近一次由Cheese of the World杂志所举行的调查显示,其订阅者对于国产奶酪的倾向性越来越高。由于我们公司可以通过限制库存来

减少开支,在我们所有连锁店增加盈利的最好方式就是停止贮备很多进口奶酪而主要集中于国产奶酪。

107)

The following appeared in a memo from the business manager of a chain of cheese stores located throughout the United States.

"For many years all the stores in our chain have stocked a wide variety of both domestic and

imported cheeses.Last year,however,all of the five best-selling cheeses at our newest store were domestic cheddar cheeses from Wisconsin.Furthermore,a recent survey by Cheeses of the World magazine indicates an increasing preference for domestic cheeses among its

subscribers.Since our company can reduce expenses by limiting inventory,the best way to improve profits in all of our stores is to discontinue stocking many of our varieties of imported cheese and concentrate primarily on domestic cheeses."

Write a response in which you examine the stated and/or unstated assumptions of the

argument.Be sure to explain how the argument depends on these assumptions and what the implications are for the argument if the assumptions prove unwarranted.

很多年来我们的所有连锁店都储备了很多种类的国产奶酪和进口奶酪。然而去年,我们的最新店里五种销量最高的奶酪都是威斯康星出产的cheddar奶酪。而且,最近一次由Cheese of the World杂志所举行的调查显示,其订阅者对于国产奶酪的倾向性越来越高。由于我们公司可以通过限制库存来减少开支,在我们所有连锁店增加盈利的最好方式就是停止贮备很多进口奶酪而主要集中于国产奶酪。

108)

The following appeared in a memo from the owner of a chain of cheese stores located throughout the United States.

"For many years all the stores in our chain have stocked a wide variety of both domestic and

imported cheeses.Last year,however,all of the five best-selling cheeses at our newest store were domestic cheddar cheeses from Wisconsin.Furthermore,a recent survey by Cheeses of the World magazine indicates an increasing preference for domestic cheeses among its

subscribers.Since our company can reduce expenses by limiting inventory,the best way to improve profits in all of our stores is to discontinue stocking many of our varieties of imported cheese and concentrate primarily on domestic cheeses."

Write a response in which you discuss what specific evidence is needed to evaluate the argument and explain how the evidence would weaken or strengthen the argument.

很多年来我们的所有连锁店都储备了很多种类的国产奶酪和进口奶酪。然而去年,我们的最新店里五种销量最高的奶酪都是威斯康星出产的cheddar奶酪。而且,最近一次由Cheese of the World杂志所举行的调查显示,其订阅者对于国产奶酪的倾向性越来越高。由于我们公司可以通过限制库存来减少开支,在我们所有连锁店增加盈利的最好方式就是停止贮备很多进口奶酪而主要集中于国产奶酪。

--------------------------------------------------------------------------------------------------------------------------------------------- 67)

The following appeared as part of a business plan developed by the manager of the Rialto Movie Theater.

"Despite its downtown location,the Rialto Movie Theater,a local institution for five decades,must make big changes or close its doors forever.It should follow the example of the new Apex Theater in the mall outside of town.When the Apex opened last year,it featured a video arcade,plush carpeting and seats,and a state-of-the-art sound system.Furthermore,in a recent survey,over 85 percent of respondents reported that the high price of newly released movies prevents them from

going to the movies more than five times per year.Thus,if the Rialto intends to hold on to its share of a decreasing pool of moviegoers,it must offer the same features as Apex."

Write a response in which you discuss what questions would need to be answered in order to decide whether the recommendation is likely to have the predicted result.Be sure to explain how the answers to these questions would help to evaluate the recommendation.

尽管已成立50余年的当地机构Rialto剧院地处闹市,它必须采取重大变革,否则就将面临永久停业。它应该借鉴城外商业街上新的Apex剧院的例子。Apex去年开业的时候,它拥有视听走廊,豪华地毯和座椅,以及最先进的音响系统。而且,最近一次调查中,超过85%的被访者报告说新发行影片的过高票价致使他们看电影的次数每年至少减少五次。因此,如果Rialto想在电影观众减少的情况下保持市场份额,它必须提供和Apex相同的设施和服务。

--------------------------------------------------------------------------------------------------------------------------------------------- 68)

A recent study reported that pet owners have longer,healthier lives on average than do people who own no pets.Specifically,dog owners tend to have a lower incidence of heart disease.In light of these findings,Sherwood Hospital should form a partnership with Sherwood Animal Shelter to institute an adopt-a-dog program.The program would encourage dog ownership for patients recovering from heart disease,which should reduce these patients'chance of experiencing continuing heart problems and also reduce their need for ongoing treatment.As a further benefit,the publicity about the program would encourage more people to adopt pets from the shelter.And that will reduce the incidence of heart disease in the general population.

Write a response in which you examine the stated and/or unstated assumptions of the

argument.Be sure to explain how the argument depends on these assumptions and what the implications are for the argument if the assumptions prove unwarranted.

最近一项研究报告说养宠物的人平均而言比不养宠物的人活得更长更健康。特别是,养狗的人心脏病发病率更低。根据这些发现,Sherwood医院应该和Sherwood动物收养所合作建立一个"收养狗"的计划。这一计划将会鼓励正在治疗心脏病的患者养狗,这将通过减少这些患者接受治疗的次数而减少医疗费用。而且,对于这一计划的宣传将鼓励更多的人从收养所领养宠物,这将减少整体人群患心脏病的危险。

--------------------------------------------------------------------------------------------------------------------------------------------- 69)

The following appeared in a memo from a vice president of a large,highly diversified company.

"Ten years ago our company had two new office buildings constructed as regional headquarters for two regions.The buildings were erected by different construction companies—Alpha and Zeta.Although the two buildings had identical floor plans,the building constructed by Zeta cost 30 percent more to build.However,that building's expenses for maintenance last year were only half those of Alpha's.In addition,the energy consumption of the Zeta building has been lower than that of the Alpha building every year since its construction.Given these data,plus the fact that Zeta has a stable workforce with little employee turnover,we recommend using Zeta rather than Alpha for our new building project,even though Alpha's bid promises lower construction costs."

Write a response in which you discuss what questions would need to be answered in order to decide whether the recommendation and the argument on which it is based are reasonable.Be sure to explain how the answers to these questions would help to evaluate the recommendation.

10年以前我们公司在两个不同地区拥有两座地方办公建筑。它们由两家建筑公司--Alpha和Zeta分别建造。尽管两座建筑的平面布局基本相同,由Zeta所建造的建筑造价高出了30%,但去年的维护费用是由Alpha建的楼的50%。而且,Z大楼建成以来的能耗每年都比Alpha大楼要少。这些数据,再加上Z公司拥有稳定的员工队伍且雇员流动性很小的事实,表明我们应该使用Z建筑公司而不是A公司来建造未来的建筑工程,尽管A的标书承诺造价会更低。

70)

The following appeared in a memo from a vice president of a large,highly diversified company.

"Ten years ago our company had two new office buildings constructed as regional headquarters for two regions.The buildings were erected by different construction companies—Alpha and

Zeta.Although the two buildings had identical floor plans,the building constructed by Zeta cost 30 percent more to build.However,that building's expenses for maintenance last year were only half those of Alpha's.Furthermore,the energy consumption of the Zeta building has been lower than that of the Alpha building every year since its construction.Such data indicate that we should use Zeta rather than Alpha for our contemplated new building project,even though Alpha's bid promises lower construction costs."

Write a response in which you discuss what specific evidence is needed to evaluate the argument and explain how the evidence would weaken or strengthen the argument.

10年以前我们公司在两个不同地区拥有两座地方办公建筑。它们由两家建筑公司--Alpha和Zeta分别建造。尽管两座建筑的平面布局基本相同,由Zeta所建造的建筑造价高出了30%,但去年的维护费用是由Alpha建的楼的50%。而且,Z大楼建成以来的能耗每年都比Alpha大楼要少。这些数据表明我们应该使用Z建筑公司而不是A公司来建造未来的建筑工程,尽管A的标书承诺造价会更低。 115)

The following appeared in a memo from a vice president of a large,highly diversified company.

"Ten years ago our company had two new office buildings constructed as regional headquarters for two different regions.The buildings were erected by two different construction

companies—Alpha and Zeta.Even though the two buildings had identical floor plans,the building constructed by Zeta cost 30 percent more to build,and its expenses for maintenance last year were twice those of the building constructed by Alpha.Furthermore,the energy consumption of the Zeta building has been higher than that of the Alpha building every year since its

construction.Such data,plus the fact that Alpha has a stable workforce with little employee turnover,indicate that we should use Alpha rather than Zeta for our contemplated new building project."

Write a response in which you examine the stated and/or unstated assumptions of the

argument.Be sure to explain how the argument depends on these assumptions and what the implications are for the argument if the assumptions prove unwarranted.

10年以前我们公司在两个不同地区拥有两座地方办公建筑。它们由两家建筑公司--Alpha和Zeta分别建造。尽管两座建筑的平面布局基本相同,由Zeta所建造的建筑造价高出了30%,去年的维护费用是由Alpha建的楼的两倍。而且,Zeta大楼建成以来的能耗每年都比Alpha大楼要多。这些数据,再加上Alpha公司拥有稳定的员工队伍且雇员流动性很小的事实,表明我们应该使用Alpha建筑公司而不是Zeta来建造未来所有的建筑工程。

--------------------------------------------------------------------------------------------------------------------------------------------- 71) The following is a letter to the editor of the Waymarsh Times.

"Traffic here in Waymarsh is becoming a problem.Although just three years ago a state traffic survey showed that the typical driving commuter took 20 minutes to get to work,the commute now takes closer to 40 minutes,according to the survey just completed.Members of the town council already have suggested more road building to address the problem,but as well as being

expensive,the new construction will surely disrupt some of our residential neighborhoods.It would be better to follow the example of the nearby city of Garville.Last year Garville implemented a policy that rewards people who share rides to work,giving them coupons for free gas.Pollution levels in Garville have dropped since the policy was implemented,and people from Garville tell me

that commuting times have fallen considerably.There is no reason why a policy like Garville's shouldn't work equally well in Waymarsh."

Write a response in which you discuss what specific evidence is needed to evaluate the argument and explain how the evidence would weaken or strengthen the argument.

W的交通已经成为一个大问题。尽管3年前一个全国的交通调查显示开车上班族需20分钟,而最近调查显示现在却要40分钟。市议会已经建议修筑更多的公路来解决这一问题,但这需要很多钱,同时工程也给本地百姓带了一些不便。我们应该效仿临近的G市,去年G市实施一项政策鼓励人们合伙搭车,并给他们免费的加油卷。政策实施后,G的污染减轻了,一些G的人们告诉我在路上的时间明显的快了很多。因此,我们W采用相同的政策也一定会达到相同的效果。

---------------------------------------------------------------------------------------------------------------------------------------------

72)

The following appeared as a letter to the editor of a national newspaper.

"Your recent article on corporate downsizing*in Elthyria maintains that the majority of competent workers who have lost jobs as a result of downsizing face serious economic hardship,often for years,before finding other suitable employment.But this claim is undermined by a recent report on the Elthyrian economy,which found that since 1999 far more jobs have been created than have been eliminated,bringing the unemployment rate in Elthyria to its lowest level in

decades.Moreover,two-thirds of these newly created jobs have been in industries that tend to pay above-average wages,and the vast majority of these jobs are full-time."

*Downsizing is the process whereby corporations deliberately make themselves smaller,reducing the number of their employees.

Write a response in which you discuss what specific evidence is needed to evaluate the argument and explain how the evidence would weaken or strengthen the argument.

你们最近关于E国集体裁员的文章里称很多在企业面临严重经济困难的时候裁员而导致失业的有能力的工人通常要用几年的时间找到另一份合适的工作。但这种观点与最近一次关于E国经济的报告相矛盾,报告发现自19xx年以来新增的就业机会数量远超过被取消的岗位数量。,并且使E的失业率达到了近几十年的最低水平。另外,新增就业机会中有三分之二是那些提供高于平均水平薪酬的企业提供的,而且这些岗位绝大多数是全职工作。

--------------------------------------------------------------------------------------------------------------------------------------------- 73) The following appeared on the Mozart School of Music Web site.

"The Mozart School of Music should be the first choice for parents considering enrolling their child in music lessons.First of all,the Mozart School welcomes youngsters at all ability and age levels;there is no audition to attend the school.Second,the school offers instruction in nearly all musical instruments as well a wide range of styles and genres from classical to rock.Third,the faculty includes some of the most distinguished musicians in the area.Finally,many Mozart graduates have gone on to become well-known and highly paid professional musicians."

Write a response in which you examine the stated and/or unstated assumptions of the

argument.Be sure to explain how the argument depends on these assumptions and what the implications are for the argument if the assumptions prove unwarranted.

Mozart音乐学校显然应该是所有学音乐学生家长的第一选择。首先,Mozart学校面向所有年龄段和能力的孩子们进行招生,并且没有任何的面试。其次,学校提供几乎所有的乐器以及范围广泛的曲

风和流派学习课程,从古典到摇滚。第三,学校员工包括一些本地最著名的音乐家。最后,很多Mozart的毕业生已经成为著名而且较高收入的音乐家。

--------------------------------------------------------------------------------------------------------------------------------------------- 74)

The president of Grove College has recommended that the college abandon its century-old

tradition of all-female education and begin admitting men.Pointing to other all-female colleges that experienced an increase in applications after adopting coeducation,the president argues that coeducation would lead to a significant increase in applications and enrollment.However,the

director of the alumnae association opposes the plan.Arguing that all-female education is essential to the very identity of the college,the director cites annual surveys of incoming students in which these students say that the school's all-female status was the primary reason they selected Grove.The director also points to a survey of Grove alumnae in which a majority of respondents strongly favored keeping the college all female.

Write a response in which you discuss what questions would need to be answered in order to decide whether the recommendation and the argument on which it is based are reasonable.Be sure to explain how the answers to these questions would help to evaluate the recommendation.

G学院院长建议Grove学院放弃其已有百年历史的女校传统,允许录取男性。鉴于其他女校在实行男女同校后,学生申请数量增加。院长认为G实行男女同校一定会在申请和入学人数上都有大幅提升。但校友会的主席却反对该提议,并认为G的女校形式已经成为深入人心。在每年一度的新生调查中显示女校的性质是她们报考G的主要原因。另外在G校友中的调查显示绝大多数的回访者强烈建议保留女校传统。

1) survey的规模和真实性

2) 是否有其他学生择校时更看重的因素

3) 反对招男生的校友和出钱资助的校友是一批人么?

4) Other all-female schools的经验对grove同样适用吗?

5) 校友的资助有多少?是否申请的增加会带来更多的收益?

147) The following recommendation was made by the president and administrative staff of Grove College,a private institution,to the college's governing committee.

"We recommend that Grove College preserve its century-old tradition of all-female education rather than admit men into its programs.It is true that a majority of faculty members voted in favor of coeducation,arguing that it would encourage more students to apply to Grove.But 80 percent of the students responding to a survey conducted by the student government wanted the school to remain all female,and over half of the alumnae who answered a separate survey also opposed coeducation.Keeping the college all female will improve morale among students and convince alumnae to keep supporting the college financially."

Write a response in which you discuss what specific evidence is needed to evaluate the argument and explain how the evidence would weaken or strengthen the argument.

我们建议Grove学院保留其已有百年历史的女校传统,而不是允许录取男性。确实有大部分员工投票赞成男女同校,认为这会使更多的学生申请Grove。但由学生会所组织的一次调查中,有80%的被访学生要求学校维持女校形式,并且在回应了一次单独调查的校友中超过一半的人也反对男女同校。因此,维持女校形式将促进学生的精神状态并且让校友继续对学院进行财政资助。

148)

The following recommendation was made by the president and administrative staff of Grove College,a private institution,to the college's governing committee.

"We recommend that Grove College preserve its century-old tradition of all-female education rather than admit men into its programs.It is true that a majority of faculty members voted in favor

of coeducation,arguing that it would encourage more students to apply to Grove.But 80 percent of the students responding to a survey conducted by the student government wanted the school to remain all female,and over half of the alumnae who answered a separate survey also opposed coeducation.Keeping the college all female will improve morale among students and convince alumnae to keep supporting the college financially."

Write a response in which you examine the stated and/or unstated assumptions of the

argument.Be sure to explain how the argument depends on these assumptions and what the implications are for the argument if the assumptions prove unwarranted.

我们建议Grove学院保留其已有百年历史的女校传统,而不是允许录取男性。确实有大部分员工投票赞成男女同校,认为这会使更多的学生申请Grove。但由学生会所组织的一次调查中,有80%的被访学生要求学校维持女校形式,并且在回应了一次单独调查的校友中超过一半的人也反对男女同校。因此,维持女校形式将促进学生的精神状态并且让校友继续对学院进行财政资助。

149)

The following recommendation was made by the president and administrative staff of Grove College,a private institution,to the college's governing committee.

"We recommend that Grove College preserve its century-old tradition of all-female education rather than admit men into its programs.It is true that a majority of faculty members voted in favor of coeducation,arguing that it would encourage more students to apply to Grove.But 80 percent of the students responding to a survey conducted by the student government wanted the school to remain all female,and over half of the alumnae who answered a separate survey also opposed coeducation.Keeping the college all female will improve morale among students and convince alumnae to keep supporting the college financially."

Write a response in which you discuss what questions would need to be answered in order to decide whether the recommendation is likely to have the predicted result.Be sure to explain how the answers to these questions would help to evaluate the recommendation.

我们建议Grove学院保留其已有百年历史的女校传统,而不是允许录取男性。确实有大部分员工投票赞成男女同校,认为这会使更多的学生申请Grove。但由学生会所组织的一次调查中,有80%的被访学生要求学校维持女校形式,并且在回应了一次单独调查的校友中超过一半的人也反对男女同校。因此,维持女校形式将促进学生的精神状态并且让校友继续对学院进行财政资助。

156)

The following recommendation was made by the president and administrative staff of Grove College,a private institution,to the college's governing committee.

"Recently,there have been discussions about ending Grove College's century-old tradition of all-female education by admitting male students into our programs.At a recent faculty meeting,a majority of faculty members voted in favor of coeducation,arguing that it would encourage more students to apply to Grove.However,Grove students,both past and present,are against the idea of coeducation.Eighty percent of the students responding to a survey conducted by the student

government wanted the school to remain all female,and over half of the alumnae who answered a separate survey also opposed coeducation.Therefore,we recommend maintaining Grove College's tradition of all-female education.We predict that keeping the college all-female will

improve morale among students and convince alumnae to keep supporting the college financially."

Write a response in which you discuss what questions would need to be answered in order to decide whether the recommendation is likely to have the predicted result.Be sure to explain how the answers to these questions would help to evaluate the recommendation.

最近,人们一直讨论关于结束G大学一个世纪以来的女校传统,开始接受男学生。在最近的一次学院会议上,有大部分员工投票赞成男女同校,认为这会使更多的学生申请Grove。但是G的学生,

不论现在还是已经毕业的,都反对同校教育。由学生会所组织的一次调查中,有80%的被访学生要求学校维持女校形式,并且在回应了一次单独调查的校友中超过一半的人也反对男女同校。因此,我们建议继续维持G的女校形式。我们预计保持女校的形式将促进学生的精神状态并且让校友继续对学院进行财政资助。 --------------------------------------------------------------------------------------------------------------------------------------------- 75)

The following appeared in a letter to the editor of a Batavia newspaper.

"The department of agriculture in Batavia reports that the number of dairy farms throughout the country is now 25 percent greater than it was 10 years ago.During this same time

period,however,the price of milk at the local Excello Food Market has increased from$1.50 to over$3.00 per gallon.To prevent farmers from continuing to receive excessive profits on an apparently increased supply of milk,the Batavia government should begin to regulate retail milk prices.Such regulation is necessary to ensure fair prices for consumers."

Write a response in which you discuss what questions would need to be answered in order to decide whether the recommendation is likely to have the predicted result.Be sure to explain how the answers to these questions would help to evaluate the recommendation.

Batavia的农业部门报告说全国奶牛农庄的数量比10年前增加了25%。然而就在同一时期,当地Excello Food Market牛奶的价格从每加仑$1.5上涨到了$3.0。为防止农场主在牛奶供应量明显增加的情况下获取过多的利润,Batavia的政府应限制牛奶的零售价。这种规定对于稳定物价是必需的。 --------------------------------------------------------------------------------------------------------------------------------------------- 76)

The following appeared in a newsletter offering advice to investors.

"Over 80 percent of the respondents to a recent survey indicated a desire to reduce their intake of foods containing fats and cholesterol,and today low-fat products abound in many food

stores.Since many of the food products currently marketed by Old Dairy Industries are high in fat and cholesterol,the company's sales are likely to diminish greatly and company profits will no doubt decrease.We therefore advise Old Dairy stockholders to sell their shares,and other investors not to purchase stock in this company."

Write a response in which you discuss what questions would need to be answered in order to decide whether the advice and the argument on which it is based are reasonable.Be sure to explain how the answers to these questions would help to evaluate the advice.

最近一次调查中超过80%的回应者表达了他们想减少含脂肪和胆固醇的食品的愿望,现在很多食品商店都提供丰富的低脂肪食品。由于Old Dairy Industries当前生产的很多产品都有很高的脂肪和胆固醇含量,该公司的销量有可能严重下降,其盈利无疑会减少。因而我们建议Old Dairy的贡持有者抛出他们所持的股份,其他投资者也不应购买该公司的股份。

--------------------------------------------------------------------------------------------------------------------------------------------- 77)

The following recommendation appeared in a memo from the mayor of the town of Hopewell.

"Two years ago,the nearby town of Ocean View built a new municipal golf course and resort hotel.During the past two years,tourism in Ocean View has increased,new businesses have

opened there,and Ocean View's tax revenues have risen by 30 percent.Therefore,the best way to improve Hopewell's economy—and generate additional tax revenues—is to build a golf course and resort hotel similar to those in Ocean View."

Write a response in which you examine the stated and/or unstated assumptions of the

argument.Be sure to explain how the argument depends on these assumptions and what the implications are for the argument if the assumptions prove unwarranted.

两年前Ocean View建造了一个新的市立高尔夫球场和度假旅馆。过去两年中,Ocean View的旅客增加了,开设了很多新的商业,而且税收增加了30%。改Hopewell的经济,产生更多税收的最好途径就是建立一个和Ocean View类似的高尔夫球场和度假旅馆。

169)

The following appeared in a memo from the mayor of Brindleburg to the city council.

"Two years ago,the town of Seaside Vista opened a new municipal golf course and resort hotel.Since then,the Seaside Vista Tourism Board has reported a 20%increase in visitors.In addition,local banks reported a steep rise in the number of new business loan applications they received this year.The amount of tax money collected by Seaside Vista has also

increased,allowing the town to announce plans to improve Seaside Vista's roads and bridges.We recommend building a similar golf course and resort hotel in Brindleburg.We predict that this project will generate additional tax revenue that the city can use to fund much-needed public improvements."

Write a response in which you discuss what questions would need to be answered in order to decide whether the recommendation is likely to have the predicted result.Be sure to explain how the answers to these questions would help to evaluate the recommendation.

两年前SV建造了一个新的市立高尔夫球场和度假旅馆。此后,SV旅游局报告说旅客增加了20%,另外当地银行报告说他们今年受到的新的商业贷款申请大幅提升,同时SV的税收也增加了,并使SV的当局宣布要计划改善当地的道路和桥梁。我们建议B也应该建立一个和SV类似的高尔夫球场和度假旅馆。我们预计该项目会提升我们的税收以来去扶持当地公共设施的改善。

--------------------------------------------------------------------------------------------------------------------------------------------- 78)

The following appeared in a memo from the vice president of a food distribution company with food storage warehouses in several cities.

"Recently,we signed a contract with the Fly-Away Pest Control Company to provide pest control services at our fast-food warehouse in Palm City,but last month we discovered that over$20,000 worth of food there had been destroyed by pest damage.Meanwhile,the Buzzoff Pest Control Company,which we have used for many years,continued to service our warehouse in

Wintervale,and last month only$10,000 worth of the food stored there had been destroyed by pest damage.Even though the price charged by Fly-Away is considerably lower,our best means of saving money is to return to Buzzoff for all our pest control services."

Write a response in which you discuss what specific evidence is needed to evaluate the argument and explain how the evidence would weaken or strengthen the argument.

最近我们和Fly-Away Pest-Control公司签订了一项合同来为我们在Palm City的快餐食品仓库提供杀虫服务,但上个月我们发现,那里有价值超过$20000的食品被害虫破坏。同时,我们使用多年的Buzzoff Pest-Control公司继续在Wintervale的仓库服务,上个月那里只有价值$10000的食品被害虫破坏。尽管Fly-Away的收费低廉的多,我们节省收费的最好方式就是重新使用Buzzoff公司来提供我们所有的杀虫服务。 114)

The following appeared in a memo from the vice president of a food distribution company with food storage warehouses in several cities.

"Recently,we signed a contract with the Fly-Away Pest Control Company to provide pest control services at our warehouse in Palm City,but last month we discovered that over$20,000 worth of food there had been destroyed by pest damage.Meanwhile,the Buzzoff Pest Control

Company,which we have used for many years in Palm City,continued to service our warehouse in Wintervale,and last month only$10,000 worth of the food stored there had been destroyed by pest

damage.Even though the price charged by Fly-Away is considerably lower,our best means of saving money is to return to Buzzoff for all our pest control services."

Write a response in which you discuss what questions would need to be answered in order to decide whether the recommendation and the argument on which it is based are reasonable.Be sure to explain how the answers to these questions would help to evaluate the recommendation.

最近我们和Fly-Away Pest-Control公司签订了一项合同来为我们在Palm City的快餐食品仓库提供杀虫服务,但上个月我们发现,那里有价值超过$20000的食品被害虫破坏。同时,我们使用多年的Buzzoff Pest-Control公司继续在Wintervale的仓库服务,上个月那里只有价值$10000的食品被害虫破坏。尽管Fly-Away的收费低廉的多,我们节省收费的最好方式就是重新使用Buzzoff公司来提供我们所有的杀虫服务。

116)

The following appeared in a memo from the vice president of a food distribution company with food storage warehouses in several cities.

"Recently,we signed a contract with the Fly-Away Pest Control Company to provide pest control services at our warehouse in Palm City,but last month we discovered that over$20,000 worth of food there had been destroyed by pest damage.Meanwhile,the Buzzoff Pest Control Company,which we have used for many years in Palm City,continued to service our warehouse in Wintervale,and last month only$10,000 worth of the food stored there had been destroyed by pest damage.This difference in pest damage is best explained by the negligence of Fly-Away."

Write a response in which you discuss one or more alternative explanations that could rival the proposed explanation and explain how your explanation(s)can plausibly account for the facts presented in the argument.

最近我们和Fly-Away Pest-Control公司签订了一项合同来为我们在Palm City的快餐食品仓库提供杀虫服务,但上个月我们发现,那里有价值超过$20000的食品被害虫破坏。同时,我们使用多年的Buzzoff Pest-Control公司继续在Wintervale的仓库服务,上个月那里只有价值$10000的食品被害虫破坏。两个公司的杀虫区别只能解释为Fly公司的粗心大意。

117)

The following appeared in a memo from the vice president of a food distribution company with food storage warehouses in several cities.

"Recently,we signed a contract with the Fly-Away Pest Control Company to provide pest control services at our warehouse in Palm City,but last month we discovered that over$20,000 worth of food there had been destroyed by pest damage.Meanwhile,the Buzzoff Pest Control Company,which we have used for many years in Palm City,continued to service our warehouse in Wintervale,and last month only$10,000 worth of the food stored there had been destroyed by pest damage.Even though the price charged by Fly-Away is considerably lower,our best means of saving money is to return to Buzzoff for all our pest control services."

Write a response in which you examine the stated and/or unstated assumptions of the

argument.Be sure to explain how the argument depends on these assumptions and what the implications are for the argument if the assumptions prove unwarranted.

最近我们和Fly-Away Pest-Control公司签订了一项合同来为我们在Palm City的快餐食品仓库提供杀虫服务,但上个月我们发现,那里有价值超过$20000的食品被害虫破坏。同时,我们使用多年的Buzzoff Pest-Control公司继续在Wintervale的仓库服务,上个月那里只有价值$10000的食品被害虫破坏。尽管Fly-Away的收费低廉的多,我们节省收费的最好方式就是重新使用Buzzoff公司来提供我们所有的杀虫服务。

--------------------------------------------------------------------------------------------------------------------------------------------- 79)

Since those issues of Newsbeat magazine that featured political news on their front cover were the poorest-selling issues over the past three years,the publisher of Newsbeat has recommended that the magazine curtail its emphasis on politics to focus more exclusively on economics and personal finance.She points to a recent survey of readers of general interest magazines that indicates greater reader interest in economic issues than in political ones.Newsbeat's

editor,however,opposes the proposed shift in editorial policy,pointing out that very few magazines offer extensive political coverage anymore.

Write a response in which you discuss what questions would need to be answered in order to decide whether the recommendation and the argument on which it is based are reasonable.Be sure to explain how the answers to these questions would help to evaluate the recommendation.

鉴于过去三年中N报纸销量最低的几期杂志是在封面上刊登了国际新闻故事的那几期,N报纸出版商建议该杂志减少对政治的关注而应更多的转向经济和个人理财方面。她指出在最近的一次大众感兴趣的杂志调查中显示大部分读者对经济问题的关注大于对政治方面的关注。然后N报纸的编辑却反对这样的变革,他指出很少有杂志会大量的报道政治内容。 --------------------------------------------------------------------------------------------------------------------------------------------- 80)

The following is taken from a memo from the advertising director of the Super Screen Movie Production Company.

"According to a recent report from our marketing department,during the past year,fewer people attended Super Screen-produced movies than in any other year.And yet the percentage of

positive reviews by movie reviewers about specific Super Screen movies actually increased during the past year.Clearly,the contents of these reviews are not reaching enough of our prospective viewers.Thus,the problem lies not with the quality of our movies but with the public's lack of awareness that movies of good quality are available.Super Screen should therefore allocate a greater share of its budget next year to reaching the public through advertising."

Write a response in which you discuss what questions would need to be answered in order to decide whether the recommendation and the argument on which it is based are reasonable.Be sure to explain how the answers to these questions would help to evaluate the recommendation.

最近我们的市场部门调查显示,尽管观众们对S的正面评价增加,但去年来S影院的人数为近几年最低。很明显,这些评价还未能深入我们未来的观众的心理。因此,目前的问题不在于我们的影片质量而是大众根本没有意识到我们拥有如此高质量的影片。因此,S影院应该将明年更多的预算资金用于广告以来让更多人认识到我们的影片。 --------------------------------------------------------------------------------------------------------------------------------------------- 81)

The following appeared in a business magazine.

"As a result of numerous complaints of dizziness and nausea on the part of consumers of Promofoods tuna,the company requested that eight million cans of its tuna be returned for testing.Promofoods concluded that the canned tuna did not,after all,pose a health risk.This conclusion is based on tests performed on samples of the recalled cans by chemists from Promofoods;the chemists found that of the eight food chemicals most commonly blamed for causing symptoms of dizziness and nausea,five were not found in any of the tested cans.The chemists did find small amounts of the three remaining suspected chemicals but pointed out that these occur naturally in all canned foods."

Write a response in which you discuss what questions would need to be addressed in order to decide whether the conclusion and the argument on which it is based are reasonable.Be sure to explain how the answers to the questions would help to evaluate the conclusion.

去年由于有大量消费者投诉说产生了眩晕和恶心,Promofoods要求800万罐金枪鱼回收检测。Promofoods下结论说这些罐头根本不含任何有健康危害的化学物质。这一结论基于这样的事实,即Promofoods的化学家测试了回收的罐头样本,并发现8种最常见的导致眩晕和恶心症状的化学物质中,有五种都没有在任何被测试的罐头中被发现。化学家确实发现剩下三种受怀疑的化学物质在所有其他罐头食品中都存在,并指出这在所有的关头中都存在,属于正常现象。 --------------------------------------------------------------------------------------------------------------------------------------------- 83)

The following is a letter to the editor of an environmental magazine.

"In 1975 a wildlife census found that there were seven species of amphibians in Xanadu National Park,with abundant numbers of each species.However,in 2002 only four species of amphibians were observed in the park,and the numbers of each species were drastically reduced.There has been a substantial decline in the numbers of amphibians worldwide,and global pollution of water and air is clearly implicated.The decline of amphibians in Xanadu National Park,however,almost certainly has a different cause:in 1975,trout—which are known to eat amphibian eggs—were introduced into the park."

Write a response in which you discuss what specific evidence is needed to evaluate the argument and explain how the evidence would weaken or strengthen the argument.

19xx年国家公园野生动物调查显示其有7中两栖动物,每种的数量都很丰富。然而到了202年在公园中只观察到4种两栖动物,并且每种动物的数量都显著下降。同时全球两栖动物数量也在持续的下降,有可能是由于全球空气和水质的污染。但X公园的下降有几乎确定是另外一种原因导致的:19xx年的在公园水域引入鲑鱼的行为(鲑鱼捕食两栖动物的卵)。

84)The following is a letter to the editor of an environmental magazine.

"Two studies of amphibians in Xanadu National Park confirm a significant decline in the numbers of amphibians.In 1975 there were seven species of amphibians in the park,and there were abundant numbers of each species.However,in 2002 only four species of amphibians were observed in the park,and the numbers of each species were drastically reduced.One proposed explanation is that the decline was caused by the introduction of trout into the park's waters,which began in 1975.(Trout are known to eat amphibian eggs.)"

Write a response in which you discuss one or more alternative explanations that could rival the proposed explanation and explain how your explanation(s)can plausibly account for the facts presented in the argument.

在X国家公园对于两栖动物所进行的两次研究证实了全球两栖动物数量的下降。19xx年公园中有7中两栖动物,每种的数量都很丰富。然而到了202年在公园中只观察到4种两栖动物,并且每种动物的数量都显著下降。一种解释就是两栖动物数量的下降曾被归因于始于19xx年的在公园水域引入鲑鱼的行为(鲑鱼捕食两栖动物的卵)。

--------------------------------------------------------------------------------------------------------------------------------------------- 85)

In a study of the reading habits of Waymarsh citizens conducted by the University of

Waymarsh,most respondents said that they preferred literary classics as reading

material.However,a second study conducted by the same researchers found that the type of book most frequently checked out of each of the public libraries in Waymarsh was the mystery

novel.Therefore,it can be concluded that the respondents in the first study had misrepresented their reading habits.

Write a response in which you discuss what specific evidence is needed to evaluate the argument and explain how the evidence would weaken or strengthen the argument.

在一次由W大学所举行的关于W居阅读习惯的调查中,多数被访者说他们倾向于阅读古典文学。

然而,由相同的研究人员随后进行的调查发现W所有公共图书馆中最经常被借阅的书是神秘小说。因此,我们可以得出结论,第一次调查的被访者错误地表达了他们的阅读习惯 87)

In a study of the reading habits of Waymarsh citizens conducted by the University of

Waymarsh,most respondents said they preferred literary classics as reading material.However,a second study conducted by the same researchers found that the type of book most frequently checked out of each of the public libraries in Waymarsh was the mystery novel.Therefore,it can be concluded that the respondents in the first study had misrepresented their reading preferences.

Write a response in which you examine the stated and/or unstated assumptions of the

argument.Be sure to explain how the argument depends on these assumptions and what the implications are for the argument if the assumptions prove unwarranted.

在一次由W大学所举行的关于W居阅读习惯的调查中,多数被访者说他们倾向于阅读古典文学。然而,由相同的研究人员随后进行的调查发现W所有公共图书馆中最经常被借阅的书是神秘小说。因此,我们可以得出结论,第一次调查的被访者错误地表达了他们的阅读习惯.

--------------------------------------------------------------------------------------------------------------------------------------------- 86)

The following appeared in a memo at XYZ company.

"When XYZ lays off employees,it pays Delany Personnel Firm to offer those employees assistance in creating résumés and developing interviewing skills,if they so desire.Laid-off

employees have benefited greatly from Delany's services:last year those who used Delany found jobs much more quickly than did those who did not.Recently,it has been proposed that we use the less expensive Walsh Personnel Firm in place of Delany.This would be a mistake because eight years ago,when XYZ was using Walsh,only half of the workers we laid off at that time found jobs within a year.Moreover,Delany is clearly superior,as evidenced by its bigger staff and larger

number of branch offices.After all,last year Delany's clients took an average of six months to find jobs,whereas Walsh's clients took nine."

Write a response in which you discuss what specific evidence is needed to evaluate the argument and explain how the evidence would weaken or strengthen the argument.

当XYZ裁员的时候,它雇佣了Delany人事公司在这些下岗员工有需要的时候为他们在写简历和提高面试技巧方面提供帮助。下岗员工从Delany的服务中极大受益:去年获得了Delany帮助的员工找工作比没有找Delany的员工快的多。最近,有提案提出用收费较少的Walsh人事公司代替Delany。这将是一个错误,因为在8年前,XYZ使用的就是Walsh,我们当时裁减的员工只有一半在一年之内找到了工作。而且,Delany显然更好,理由是它的员工更多,分支机构也更多。不论如何,去年Delany的客户平均用六个月找到工作,而Walsh的客户平均用了9个月。

89)

The following appeared in a memo at XYZ company.

"When XYZ lays off employees,it pays Delany Personnel Firm to offer those employees assistance in creating résumés and developing interviewing skills,if they so desire.Laid-off

employees have benefited greatly from Delany's services:last year those who used Delany found jobs much more quickly than did those who did not.Recently,it has been proposed that we use the less expensive Walsh Personnel Firm in place of Delany.This would be a mistake because eight years ago,when XYZ was using Walsh,only half of the workers we laid off at that time found jobs within a year.Moreover,Delany is clearly superior,as evidenced by its bigger staff and larger

number of branch offices.After all,last year Delany's clients took an average of six months to find jobs,whereas Walsh's clients took nine."

Write a response in which you examine the stated and/or unstated assumptions of the

argument.Be sure to explain how the argument depends on these assumptions and what the implications are for the argument if the assumptions prove unwarranted.

当XYZ裁员的时候,它雇佣了Delany人事公司在这些下岗员工有需要的时候为他们在写简历和提高面试技巧方面提供帮助。下岗员工从Delany的服务中极大受益:去年获得了Delany帮助的员工找工作比没有找Delany的员工快的多。最近,有提案提出用收费较少的Walsh人事公司代替Delany。这将是一个错误,因为在8年前,XYZ使用的就是Walsh,我们当时裁减的员工只有一半在一年之内找到了工作。而且,Delany显然更好,理由是它的员工更多,分支机构也更多。不论如何,去年Delany的客户平均用六个月找到工作,而Walsh的客户平均用了9个月。 --------------------------------------------------------------------------------------------------------------------------------------------- 88)

The following appeared in a memorandum written by the vice president of Health Naturally,a small but expanding chain of stores selling health food and other health-related products.

"Our previous experience has been that our stores are most profitable in areas where residents are highly concerned with leading healthy lives.We should therefore build one of our new stores in Plainsville,which clearly has many such residents.Plainsville merchants report that sales of running shoes and exercise equipment are at all-time highs.The local health club,which nearly closed five years ago due to lack of business,has more members than ever,and the weight-training and aerobics classes are always full.We can even anticipate a new generation of

customers:Plainsville's schoolchildren are required to participate in a program called Fitness for Life,which emphasizes the benefits of regular exercise at an early age."

Write a response in which you discuss what specific evidence is needed to evaluate the argument and explain how the evidence would weaken or strengthen the argument.

以前的经验显示,我们的商店在那些居对健康生活高度关注的地区是盈利最多的。因此我们应该把下一家连锁店开设在Plainsville,那里有很多这样的居。Plainsville的商家报告说运动鞋和运动衣的销售处于历史高点。当地一家五年前因缺乏客源而濒临倒闭的康体俱乐部现在的会员比以往任何时候都多,减肥训练和体操班总是满员。我们还可以预见到新生代的顾客群:Plainsville的在校学生被要求参加一个叫做"终生健康"的项目,它强调从小开始经常锻炼的好处。

90)

The following appeared in a memorandum written by the vice president of Health Naturally,a small but expanding chain of stores selling health food and other health-related products.

"Our previous experience has been that our stores are most profitable in areas where residents are highly concerned with leading healthy lives.We should therefore build one of our new stores in Plainsville,which clearly has many such residents.Plainsville merchants report that sales of running shoes and exercise equipment are at all-time highs.The local health club,which nearly closed five years ago due to lack of business,has more members than ever,and the weight-training and aerobics classes are always full.We can even anticipate a new generation of

customers:Plainsville's schoolchildren are required to participate in a program called Fitness for Life,which emphasizes the benefits of regular exercise at an early age."

Write a response in which you examine the stated and/or unstated assumptions of the

argument.Be sure to explain how the argument depends on these assumptions and what the implications are for the argument if the assumptions prove unwarranted.

以前的经验显示,我们的商店在那些居对健康生活高度关注的地区是盈利最多的。因此我们应该把下一家连锁店开设在Plainsville,那里有很多这样的居。Plainsville的商家报告说运动鞋和运动衣的销售处于历史高点。当地一家五年前因缺乏客源而濒临倒闭的康体俱乐部现在的会员比以往任何时候都多,减肥训练和体操班总是满员。我们还可以预见到新生代的顾客群:Plainsville的在校学生被要

求参加一个叫做"终生健康"的项目,它强调从小开始经常锻炼的好处。

--------------------------------------------------------------------------------------------------------------------------------------------- 91)

Three years ago,because of flooding at the Western Palean Wildlife Preserve,100 lions and 100 western gazelles were moved to the East Palean Preserve,an area that is home to most of the same species that are found in the western preserve,though in larger numbers,and to the eastern gazelle,a close relative of the western gazelle.The only difference in climate is that the eastern preserve typically has slightly less rainfall.Unfortunately,after three years in the eastern

preserve,the imported western gazelle population has been virtually eliminated.Since the slight reduction in rainfall cannot be the cause of the virtual elimination of western gazelle,their

disappearance must have been caused by the larger number of predators in the eastern preserve.

Write a response in which you discuss what specific evidence is needed to evaluate the argument and explain how the evidence would weaken or strengthen the argument.

3年前,由于P西部野生动物保护区的洪水,100只狮子和100只西部羚羊被转移到了东部,这里也是很多与西部物种属相同种族的物种的家,尽管数量更大,同时也是西部羚羊的近亲-东部羚羊的家。气候上唯一的区别就是东部雨量微少一些。不幸的是,3年后,东部保护区的外来西部羚羊几乎绝迹。因为雨量的微小差别不可能是导致西部羚羊灭绝的原因,其灭绝原因一定是因为东部保护区内大量的食肉动物的存在。

--------------------------------------------------------------------------------------------------------------------------------------------- 92) Workers in the small town of Leeville take fewer sick days than workers in the large city of Masonton,50 miles away.Moreover,relative to population size,the diagnosis of stress-related illness is proportionally much lower in Leeville than in Masonton.According to the Leeville

Chamber of Commerce,these facts can be attributed to the health benefits of the relatively relaxed pace of life in Leeville.

Write a response in which you discuss one or more alternative explanations that could rival the proposed explanation and explain how your explanation(s)can plausibly account for the facts presented in the argument.

小城镇L的的员工病假天数比50英里外的大城市Mason要少。而且,相对于人口数量,L的抑郁类诊断人数也远远少于M。根据L的商业部门总结,这样的健康状态归结于L相对比较悠闲的生活节奏。 101)

There is now evidence that the relaxed pace of life in small towns promotes better health and greater longevity than does the hectic pace of life in big cities.Businesses in the small town of Leeville report fewer days of sick leave taken by individual workers than do businesses in the nearby large city of Masonton.Furthermore,Leeville has only one physician for its one thousand residents,but in Masonton the proportion of physicians to residents is five times as high.Finally,the average age of Leeville residents is significantly higher than that of Masonton residents.These findings suggest that people seeking longer and healthier lives should consider moving to small communities.

Write a response in which you examine the stated and/or unstated assumptions of the

argument.Be sure to explain how the argument depends on these assumptions and what the implications are for the argument if the assumptions prove unwarranted.

现有证据表明,在小城市的悠闲生活方式比在大城市快节奏的生活更有利于健康和长寿。小城镇Leeville的商业行业所上报的员工病假天数比邻近大城市Mason要少。而且,Leeville每千人只有一名医生,而Mason市医生的比例是Leeville的五倍。Leeville居的平均年龄显著高于Mason居。这些数据表明寻求更健康更长寿生活的人应该考虑移居到小型社区。

103)

There is now evidence that the relaxed pace of life in small towns promotes better health and greater longevity than does the hectic pace of life in big cities.Businesses in the small town of Leeville report fewer days of sick leave taken by individual workers than do businesses in the nearby large city of Masonton.Furthermore,Leeville has only one physician for its one thousand residents,but in Masonton the proportion of physicians to residents is five times as high.Finally,the average age of Leeville residents is significantly higher than that of Masonton residents.These findings suggest that the relaxed pace of life in Leeville allows residents to live longer,healthier lives.

Write a response in which you discuss one or more alternative explanations that could rival the proposed explanation and explain how your explanation(s)can plausibly account for the facts presented in the argument.

现有证据表明,在小城市的悠闲生活方式比在大城市快节奏的生活更有利于健康和长寿。小城镇Leeville的商业行业所上报的员工病假天数比邻近大城市Mason要少。而且,Leeville每千人只有一名医生,而Mason市医生的比例是Leeville的五倍。Leeville居的平均年龄显著高于Mason居。这些数据表明寻L的休闲节奏会让百姓活得更健康更长寿。

--------------------------------------------------------------------------------------------------------------------------------------------- 97)

The following appeared in an e-mail sent by the marketing director of the Classical Shakespeare Theatre of Bardville.

"Over the past ten years,there has been a 20 percent decline in the size of the average audience at Classical Shakespeare Theatre productions.In spite of increased advertising,we are attracting fewer and fewer people to our shows,causing our profits to decrease significantly.We must take action to attract new audience members.The best way to do so is by instituting a'Shakespeare in the Park'program this summer.Two years ago the nearby Avon Repertory Company started a'Free Plays in the Park'program,and its profits have increased 10 percent since then.If we start a'Shakespeare in the Park'program,we can predict that our profits will increase,too."

Write a response in which you discuss what questions would need to be answered in order to decide whether the recommendation is likely to have the predicted result.Be sure to explain how the answers to these questions would help to evaluate the recommendation.

在过去的10年间,CS剧院的平均观众人数下降了20%。尽管广告的力度加大,但却越来越难以吸引观众来看我们的演出,结果导致我们的利润明显下滑。我们必须要采取行动来吸引新的观众,最好的方式就是在今年夏天举办一个“Shakespeare in the park”活动。因为2年前,临近的A公司开展了一次“free play in the park”的活动后,利润上涨了10%。如果我们开展一次“Shakespeare in the park”活动,我们预计我们的利润也会上涨。

--------------------------------------------------------------------------------------------------------------------------------------------- 113)

The following is a recommendation from the personnel director to the president of Acme Publishing Company.

"Many other companies have recently stated that having their employees take the Easy Read Speed-Reading Course has greatly improved productivity.One graduate of the course was able to read a 500-page report in only two hours;another graduate rose from an assistant manager to vice president of the company in under a year.Obviously,the faster you can read,the more information you can absorb in a single workday.Moreover,Easy Read would cost Acme only$500 per employee—a small price to pay when you consider the benefits.Included in this fee is a three-week seminar in Spruce City and a lifelong subscription to the Easy Read

newsletter.Clearly,to improve productivity,Acme should require all of our employees to take the Easy Read course."

Write a response in which you discuss what questions would need to be answered in order to decide whether the advice and the argument on which it is based are reasonable.Be sure to explain how the answers to these questions would help to evaluate the advice.

很多其他公司最近指出他们的员工参加了Easy Read的速读课程之后生产效率显著提高。这个课程的一名毕业生能够在两个小时之内读完长达500页的报告,另一名毕业生在一年内从助理经理上升到了副总裁。显然,你的阅读速度越快,在一天之内所能吸收的信息就越多。而且,Easy Read的学费只有每人500元,当考虑到它给A公司带来的效益的时候这就是个小数目。这个费用包括在S市举行的为期三周的研讨会和Easy Read新闻刊物的终生赠阅。显然,A公司通过要求所有员工参加Easy Read的课程将会极大地受益。

132)

The following appeared in a letter to the school board in the town of Centerville.

"All students should be required to take the driver's education course at Centerville High School.In the past two years,several accidents in and around Centerville have involved teenage

drivers.Since a number of parents in Centerville have complained that they are too busy to teach their teenagers to drive,some other instruction is necessary to ensure that these teenagers are safe drivers.Although there are two driving schools in Centerville,parents on a tight budget cannot afford to pay for driving instruction.Therefore an effective and mandatory program sponsored by the high school is the only solution to this serious problem."

Write a response in which you examine the stated and/or unstated assumptions of the

argument.Be sure to explain how the argument depends on these assumptions and what the implications are for the argument if the assumptions prove unwarranted.

Centerville高中的所有学生都应该被要求参加驾驶员教育课程。在过去两年中,Centerville周围有几起交通事故涉及了青少年驾驶者。由于Centerville的一些家长抱怨说他们太忙没有时间教他们的孩子驾驶,必须有一些其他的课程来保证这些孩子成为安全的驾驶者。尽管在Centerville已经有两所驾校,手头不宽裕的家长无法负担驾校学费。因此由学校组织的有效的强制性课程是解决这一严重问题的唯一方案。

134)

The following appeared in a letter to the school board in the town of Centerville.

"All students should be required to take the driver's education course at Centerville High School.In the past two years,several accidents in and around Centerville have involved teenage drivers.Since a number of parents in Centerville have complained that they are too busy to teach their teenagers to drive,some other instruction is necessary to ensure that these teenagers are safe drivers.Although there are two driving schools in Centerville,parents on a tight budget cannot afford to pay for driving instruction.Therefore an effective and mandatory program sponsored by the high school is the only solution to this serious problem."

Write a response in which you discuss what specific evidence is needed to evaluate the argument and explain how the evidence would weaken or strengthen the argument.

Centerville高中的所有学生都应该被要求参加驾驶员教育课程。在过去两年中,Centerville周围有几起交通事故涉及了青少年驾驶者。由于Centerville的一些家长抱怨说他们太忙没有时间教他们的孩子驾驶,必须有一些其他的课程来保证这些孩子成为安全的驾驶者。尽管在Centerville已经有两所驾校,手头不宽裕的家长无法负担驾校学费。因此由学校组织的有效的强制性课程是解决这一严重问题的唯一方案。

136)

The following appeared in a letter to the school board in the town of Centerville.

"All students should be required to take the driver's education course at Centerville High School.In the past two years,several accidents in and around Centerville have involved teenage

drivers.Since a number of parents in Centerville have complained that they are too busy to teach their teenagers to drive,some other instruction is necessary to ensure that these teenagers are safe drivers.Although there are two driving schools in Centerville,parents on a tight budget cannot afford to pay for driving instruction.Therefore an effective and mandatory program sponsored by the high school is the only solution to this serious problem."

Write a response in which you discuss what questions would need to be answered in order to decide whether the recommendation and the argument on which it is based are reasonable.Be sure to explain how the answers to these questions would help to evaluate the recommendation.

Centerville高中的所有学生都应该被要求参加驾驶员教育课程。在过去两年中,Centerville周围有几起交通事故涉及了青少年驾驶者。由于Centerville的一些家长抱怨说他们太忙没有时间教他们的孩子驾驶,必须有一些其他的课程来保证这些孩子成为安全的驾驶者。尽管在Centerville已经有两所驾校,手头不宽裕的家长无法负担驾校学费。因此由学校组织的有效的强制性课程是解决这一严重问题的唯一方案。

--------------------------------------------------------------------------------------------------------------------------------------------- 126)

The following is a recommendation from the personnel director to the president of Acme Publishing Company.

"Many other companies have recently stated that having their employees take the Easy Read Speed-Reading Course has greatly improved productivity.One graduate of the course was able to read a 500-page report in only two hours;another graduate rose from an assistant manager to vice president of the company in under a year.Obviously,the faster you can read,the more information you can absorb in a single workday.Moreover,Easy Read would cost Acme only$500 per employee—a small price to pay when you consider the benefits.Included in this fee is a three-week seminar in Spruce City and a lifelong subscription to the Easy Read

newsletter.Clearly,Acme would benefit greatly by requiring all of our employees to take the Easy Read course."

Write a response in which you discuss what specific evidence is needed to evaluate the argument and explain how the evidence would weaken or strengthen the argument.

很多其他公司最近指出他们的员工参加了Easy Read的速读课程之后生产效率显著提高。这个课程的一名毕业生能够在两个小时之内读完长达500页的报告,另一名毕业生在一年内从助理经理上升到了副总裁。显然,你的阅读速度越快,在一天之内所能吸收的信息就越多。而且,Easy Read的学费只有每人500元,当考虑到它给A公司带来的效益的时候这就是个小数目。这个费用包括在S市举行的为期三周的研讨会和Easy Read新闻刊物的终生赠阅。显然,A公司通过要求所有员工参加Easy Read的课程将会极大地受益。

127)

The following is a recommendation from the personnel director to the president of Acme Publishing Company.

"Many other companies have recently stated that having their employees take the Easy Read Speed-Reading Course has greatly improved productivity.One graduate of the course was able to read a 500-page report in only two hours;another graduate rose from an assistant manager to vice president of the company in under a year.Obviously,the faster you can read,the more information you can absorb in a single workday.Moreover,Easy Read would cost Acme only$500 per employee—a small price to pay when you consider the benefits.Included in this fee is a three-week seminar in Spruce City and a lifelong subscription to the Easy Read

newsletter.Clearly,to improve overall productivity,Acme should require all of our employees to take the Easy Read course."

Write a response in which you discuss what questions would need to be answered in order to decide whether the recommendation and the argument on which it is based are reasonable.Be sure to explain how the answers to these questions would help to evaluate the recommendation.

很多其他公司最近指出他们的员工参加了Easy Read的速读课程之后生产效率显著提高。这个课程的一名毕业生能够在两个小时之内读完长达500页的报告,另一名毕业生在一年内从助理经理上升到了副总裁。显然,你的阅读速度越快,在一天之内所能吸收的信息就越多。而且,Easy Read的学费只有每人500元,当考虑到它给A公司带来的效益的时候这就是个小数目。这个费用包括在S市举行的为期三周的研讨会和Easy Read新闻刊物的终生赠阅。显然,为了提高A公司的整体生产效率,它要求所有员工参加Easy Read的课程。

161)

The following is a recommendation from the personnel director to the president of Acme Publishing Company.

"Many other companies have recently stated that having their employees take the Easy Read Speed-Reading Course has greatly improved productivity.One graduate of the course was able to read a 500-page report in only two hours;another graduate rose from an assistant manager to vice president of the company in under a year.Obviously,the faster you can read,the more information you can absorb in a single workday.Moreover,Easy Read would cost Acme only$500 per employee—a small price to pay when you consider the benefits.Included in this fee is a three-week seminar in Spruce City and a lifelong subscription to the Easy Read

newsletter.Clearly,Acme would benefit greatly by requiring all of our employees to take the Easy Read course."

Write a response in which you examine the stated and/or unstated assumptions of the

argument.Be sure to explain how the argument depends on these assumptions and what the implications are for the argument if the assumptions prove unwarranted.

很多其他公司最近指出他们的员工参加了Easy Read的速读课程之后生产效率显著提高。这个课程的一名毕业生能够在两个小时之内读完长达500页的报告,另一名毕业生在一年内从助理经理上升到了副总裁。显然,你的阅读速度越快,在一天之内所能吸收的信息就越多。而且,Easy Read的学费只有每人500元,当考虑到它给A公司带来的效益的时候这就是个小数目。这个费用包括在S市举行的为期三周的研讨会和Easy Read新闻刊物的终生赠阅。显然,为了提高A公司的整体生产效率,它要求所有员工参加Easy Read的课程。

--------------------------------------------------------------------------------------------------------------------------------------------- 135)

The data from a survey of high school math and science teachers show that in the district of Sanlee many of these teachers reported assigning daily homework,whereas in the district of

Marlee,most science and math teachers reported assigning homework no more than two or three days per week.Despite receiving less frequent homework assignments,Marlee students earn

better grades overall and are less likely to be required to repeat a year of school than are students in Sanlee.These results call into question the usefulness of frequent homework assignments.Most likely the Marlee students have more time to concentrate on individual assignments than do the Sanlee students who have homework every day.Therefore teachers in our high schools should assign homework no more than twice a week.

Write a response in which you discuss what specific evidence is needed to evaluate the argument and explain how the evidence would weaken or strengthen the argument.

Attra州的教育部建议高中学生应该每天都被布置家庭作业。然而最近一项对于全州的数学和理化高中教师的调查却对每天布置家庭作业的作用提出了疑问。在Sanlee地区,86%的教师报告说他们每周布置3-5次家庭作业,而在Marlee地区,只有少于25%的教师报告说每周布置3-5次作业。而

Marlee地区的学生综合成绩更好,而且比起Sanlee的学生更不容易留级。因此,我们高中的所有教师每周布置作业都不应该超过两次。 137)

While the Department of Education in the state of Attra recommends that high school students be assigned homework every day,the data from a recent statewide survey of high school math and science teachers give us reason to question the usefulness of daily homework.In the district of Sanlee,86 percent of the teachers reported assigning homework three to five times a

week,whereas in the district of Marlee,less than 25 percent of the teachers reported assigning homework three to five times a week.Yet the students in Marlee earn better grades overall and are less likely to be required to repeat a year of school than are the students in Sanlee.Therefore,all teachers in our high schools should assign homework no more than twice a week.

Write a response in which you examine the stated and/or unstated assumptions of the

argument.Be sure to explain how the argument depends on these assumptions and what the implications are for the argument if the assumptions prove unwarranted.

Attra州的教育部建议高中学生应该每天都被布置家庭作业。然而最近一项对于全州的数学和理化高中教师的调查却对每天布置家庭作业的作用提出了疑问。在Sanlee地区,86%的教师报告说他们每周布置3-5次家庭作业,而在Marlee地区,只有少于25%的教师报告说每周布置3-5次作业。而Marlee地区的学生综合成绩更好,而且比起Sanlee的学生更不容易留级。因此,我们高中的所有教师每周布置作业都不应该超过两次。 140)

While the Department of Education in the state of Attra suggests that high school students be assigned homework every day,the data from a recent statewide survey of high school math and science teachers give us reason to question the usefulness of daily homework.In the district of Sanlee,86 percent of the teachers reported assigning homework three to five times a

week,whereas in the district of Marlee,less than 25 percent of the teachers reported assigning homework three to five times a week.Yet the students in Marlee earn better grades overall and are less likely to be required to repeat a year of school than are the students in Sanlee.Therefore,we recommend that all teachers in our high schools should assign homework no more than twice a week.

Write a response in which you discuss what questions would need to be answered in order to decide whether the recommendation and the argument on which it is based are reasonable.Be sure to explain how the answers to these questions would help to evaluate the recommendation.

Attra州的教育部建议高中学生应该每天都被布置家庭作业。然而最近一项对于全州的数学和理化高中教师的调查却对每天布置家庭作业的作用提出了疑问。在Sanlee地区,86%的教师报告说他们每周布置3-5次家庭作业,而在Marlee地区,只有少于25%的教师报告说每周布置3-5次作业。而Marlee地区的学生综合成绩更好,而且比起Sanlee的学生更不容易留级。因此,我们高中的所有教师每周布置作业都不应该超过两次。

--------------------------------------------------------------------------------------------------------------------------------------------- 152)

The following appeared in a memo to the board of directors of Bargain Brand Cereals.

"One year ago we introduced our first product,Bargain Brand breakfast cereal.Our very low prices quickly drew many customers away from the top-selling cereal companies.Although the

companies producing the top brands have since tried to compete with us by lowering their prices and although several plan to introduce their own budget brands,not once have we needed to raise our prices to continue making a profit.Given our success in selling cereal,we recommend that Bargain Brand now expand its business and begin marketing other low-priced food products as quickly as possible."

Write a response in which you discuss what questions would need to be answered in order to decide whether the recommendation and the argument on which it is based are reasonable.Be sure to explain how the answers to these questions would help to evaluate the recommendation.

一年以前我们推出了第一种产品Bargain Brand早餐麦片。我们低廉的价格迅速从一些销量最大的麦片厂商那里吸引了大量客户。尽管这些厂商曾经试图通过降低价格来与我们竞争,并且尽管有一些公司打算推出他们自己的廉价品牌,但我们从未需要通过涨价来持续盈利。基于我们销售麦片的成,Bargain Brand应该扩展业务,并且尽快开始推出其他的廉价食品。

153)

The following appeared in a memo to the board of directors of Bargain Brand Cereals.

"One year ago we introduced our first product,Bargain Brand breakfast cereal.Our very low prices quickly drew many customers away from the top-selling cereal companies.Although the

companies producing the top brands have since tried to compete with us by lowering their prices and although several plan to introduce their own budget brands,not once have we needed to raise our prices to continue making a profit.Given our success in selling cereal,we recommend that Bargain Brand now expand its business and begin marketing other low-priced food products as quickly as possible."

Write a response in which you examine the stated and/or unstated assumptions of the

argument.Be sure to explain how the argument depends on these assumptions and what the implications are for the argument if the assumptions prove unwarranted.

一年以前我们推出了第一种产品Bargain Brand早餐麦片。我们低廉的价格迅速从一些销量最大的麦片厂商那里吸引了大量客户。尽管这些厂商曾经试图通过降低价格来与我们竞争,并且尽管有一些公司打算推出他们自己的廉价品牌,但我们从未需要通过涨价来持续盈利。基于我们销售麦片的成,Bargain Brand应该扩展业务,并且尽快开始推出其他的廉价食品。

--------------------------------------------------------------------------------------------------------------------------------------------- 157)

The following appeared in a memo from the marketing director of Top Dog Pet Stores.

"Five years ago Fish Emporium started advertising in the magazine Exotic Pets Monthly.Their stores saw sales increase by 15 percent after their ads began appearing in the magazine.The three Fish Emporium stores in Gulf City saw an even greater increase than that.Because Top Dog Pet Stores is based in Gulf City,it seems clear that we should start placing our own ads inExotic Pets Monthly.If we do so,we will be sure to reverse the recent trend of declining sales and start making a profit again."

Write a response in which you examine the stated and/or unstated assumptions of the

argument.Be sure to explain how the argument depends on these assumptions and what the implications are for the argument if the assumptions prove unwarranted.

五年前,F商店开始在EP月刊杂志上做广告,此后其销售额上涨了15%。G市的三家F商店增长更为明显。因为TD宠物商店就座落在G市,看起来很明显我们应该开始在EP月刊上做广告了。如果这样做,我们一定会扭转最近的销售颓势并且在此盈利。

158)

The following appeared in a memo from the marketing director of Top Dog Pet Stores.

"Five years ago,Fish Emporium started advertising in the magazine Exotic Pets Monthly.Their stores saw sales increase by 15 percent.The three Fish Emporium stores in Gulf City saw an even greater increase than that.Because Top Dog has some of its largest stores in Gulf City,it seems clear that we should start placing our own ads in Exotic Pets Monthly.If we do so,we will be sure to reverse the recent trend of declining sales and start making a profit again."

Write a response in which you discuss what specific evidence is needed to evaluate the argument and explain how the evidence would weaken or strengthen the argument.

五年前,F商店开始在EP月刊杂志上做广告,此后其销售额上涨了15%。G市的三家F商店增长更为明显。因为TD一些最大宠物商店就座落在G市,看起来很明显我们应该开始在EP月刊上做广告了。如果这样做,我们一定会扭转最近的销售颓势并且在此盈利。 --------------------------------------------------------------------------------------------------------------------------------------------- 160)

The following appeared in a recommendation from the President of the Amburg Chamber of Commerce.

"Last October,the city of Belleville installed high-intensity lighting in its central business district,and vandalism there declined almost immediately.The city of Amburg,on the other hand,recently instituted police patrols on bicycles in its business district.However,the rate of vandalism here remains constant.Since high-intensity lighting is clearly the most effective way to combat crime,we recommend using the money that is currently being spent on bicycle patrols to install such lighting throughout Amburg.If we install this high-intensity lighting,we will significantly reduce crime rates in Amburg."

Write a response in which you discuss what questions would need to be answered in order to decide whether the recommendation is likely to have the predicted result.Be sure to explain how the answers to these questions would help to evaluate the recommendation.

去年10月Belleville市在其中央商业区安装了高照度灯光,那里的破坏公物的行为几乎立即减少。Amburg市最近开始在其中央商业区安排警察骑自行车巡逻,但破坏公物的发生率并没有变化。由于高照度灯光显然是震慑犯罪的最有效途径,我们建议应该把原来用于巡逻的开销用来在Amburg全市安装这种灯光。如果我们安装这样的高照度灯,那么Amburg的犯罪率会大幅降低。

172)

The following appeared in a recommendation from the president of Amburg's Chamber of Commerce.

"Last October the city of Belleville installed high-intensity lighting in its central business district,and vandalism there declined within a month.The city of Amburg has recently begun police patrols on bicycles in its business district,but the rate of vandalism there remains constant.We should install high-intensity lighting throughout Amburg,then,because doing so is a more effective way to

combat crime.By reducing crime in this way,we can revitalize the declining neighborhoods in our city."

Write a response in which you discuss what specific evidence is needed to evaluate the argument and explain how the evidence would weaken or strengthen the argument.

去年10月Belleville市在其中央商业区安装了高照度灯光,那里的破坏公物的行为在一个月内减少。Amburg市最近开始在其中央商业区安排警察骑自行车巡逻,但破坏公物的发生率并没有变化。由于高照度灯光显然是震慑犯罪的有效途径,我们建议应该在Amburg全市安装这种灯光。通过犯罪率大幅的降低,我们可以重新恢复那些萎靡社区的活力。

--------------------------------------------------------------------------------------------------------------------------------------------- 168) The following is a letter that recently appeared in the Oak City Gazette,a local newspaper.

"The primary function of the Committee for a Better Oak City is to advise the city government on how to make the best use of the city's limited budget.However,at some of our recent meetings we failed to make important decisions because of the foolish objections raised by committee members who are not even residents of Oak City.People who work in Oak City but who live elsewhere cannot fully understand the business and politics of the city.After all,only Oak City

residents pay city taxes,and therefore only residents understand how that money could best be used to improve the city.We recommend,then,that the Committee for a Better Oak City vote to restrict its membership to city residents only.We predict that,without the interference of

non-residents,the committee will be able to make Oak City a better place in which to live and work."

Write a response in which you discuss what questions would need to be answered in order to decide whether the recommendation is likely to have the predicted result.Be sure to explain how the answers to these questions would help to evaluate the recommendation.

O城市改善委员会的主要责任就是在如果更好使用有限的预算方面给予其建议。但是,在最近的会议上我们没有做出任何的重要的决定,是因为在会议上一些并非O市民的委员们提出的愚蠢的反对意见。那些只在O工作而不居住的人是根本不可能完全了解这里的政治和经济的。总而言之,只有O的市民缴税,因此只有他们才知道如何使用这笔钱更好的改善本市。所以我们的建议是城市改善委员会的委员们只有O市市民才可以投票。我们预计没有了外地人的干预,这个委员会会使我们O市变成更适宜工作和居住的城市。

--------------------------------------------------------------------------------------------------------------------------------------------- 170)

The following appeared in a memo from the vice president of a company that builds shopping malls around the country.

"The surface of a section of Route 101,paved just two years ago by Good Intentions Roadways,is now badly cracked with a number of dangerous potholes.In another part of the state,a section of Route 40,paved by Appian Roadways more than four years ago,is still in good condition.In a demonstration of their continuing commitment to quality,Appian Roadways recently purchased state-of-the-art paving machinery and hired a new quality-control manager.Therefore,I

recommend hiring Appian Roadways to construct the access roads for all our new shopping malls.I predict that our Appian access roads will not have to be repaired for at least four years."

Write a response in which you discuss what questions would need to be answered in order to decide whether the recommendation is likely to have the predicted result.Be sure to explain how the answers to these questions would help to evaluate the recommendation.

两年前由G筑路公司修筑的101公路的一段,表面现已严重开裂,并存在危险的坑洼。在同一州的另一个地区,四年多以前由Appian筑路公司修筑的66公路的一段现在依然状况良好。Appian公司最近购买了最先进的筑路机械,并雇佣了一名新的质量监控经理。由于Appian公司更优秀的工作和质量保证,我们应该和他们而不是G公司签订合同修筑我们新的商业街前面的通道。我们预计这条有A修筑的通道最少4年不用维修。